Вы находитесь на странице: 1из 132

FSICA

CONCEPTO 2. Magnitudes Derivadas


Desde que la palabra Fsica proviene Son aquellas que pueden ser expresadas
del trmino Physis, que significa en funcin de las magnitudes
Naturaleza, en sus inicios, ms o fundamentales.
menos hasta principios del siglo XIX, la
Fsica se consider como una Ciencia que II. SEGUN SU NATURALEZA
estudiara todos los fenmenos 1. Magnitudes Escalares:
naturales. Pero a partir del siglo XIX, se Son aquellas que quedan perfectamente
redujo su campo, limitndola al estudio definidas mediante un nmero real y su
de los llamados Fenmenos Fsicos, el correspondiente unidad de medida.
resto de fenmenos pasaron a formar Ejemplo: -10C; 5kg; etc.
parte de otras ciencias naturales.
La fsica es una ciencia natural 2. Magnitudes Vectoriales
encargada de estudiar los fenmenos Son aquellas que adems de conocer su
fsicos que ocurren en la naturaleza, valor, se requiere de su direccin y
sistematizndolos a travs de leyes sentido para quedar perfectamente
fsicas determinadas. definidas.

Fenmeno Fsico: Ejemplo:


Es todo cambio y/o transformacin que La Velocidad
experimentan ciertos cuerpos sin alterar La Aceleracin
su estructura ntima. Es decir, son La Fuerza, etc.
cambios reversibles.
Por ejemplo: SISTEMA INTERNACIONAL DE
Los cambios de estado UNIDADES (S.I.)
El movimiento de los cuerpos
La dilatacin de los cuerpos, Considera siete magnitudes
etc. fundamentales y dos auxiliares.

Anlisis Dimensional Magnitud Smb. Unidad Abreviatura


Longitud L Metro m
Magnitud Fsica Masa M Kilogramo Kg
Es todo aquello que puede ser medido Tiempo T Segundo s
con cierto grado de precisin usando Intensidad
para ello una unidad de medida patrn de Corriente I Ampere A
convencionalmente establecida. Elctrica
Las magnitudes fsicas, se clasifican en: Temperatura Kelvin K
Intensidad J Candela cd
I. SEGN SU ORIGEN Luminosa
1. Magnitudes Fundamentales Cantidad de N Mol mol
Son aquellas magnitudes que sirven de Sustancia
base para fijar las unidades y en funcin
de las cuales se expresan las dems
magnitudes.

O
FSICA
Ecuacin Dimensional Ejemplo:
Es aquella igualdad matemtica que [Cos 74] = 1 [ 5] = 1
sirve para relacionar las dimensiones de
[2] = 1
las magnitudes fsicas fundamentales,

3 2 = 1
para obtener las magnitudes derivadas
y fijar as sus unidades, adems permite
verificar si una frmula o ley fsica, es o
no correcta, dimensionalmente. 2 Slo se podr sumar o restar
magnitudes de la misma especie y
Notacin: el resultado de dicha operacin
Se usa un par de corchetes, as: ser igual a la misma magnitud.
Ejm.:
[ ] se lee Ecuacin Dimensional De 3m + 2m = 5m
[3m] + [2m] = [5m]
Ejemplo:
L+L=L
[B] : Ecuacin dimensional de la
magnitud fsica B Ejemplo:

ECUACIONES DIMENSIONALES MAS 8S 5S = 3S


CONOCIDAS [85] - [5S] = [3S]
1. [AREA] = L
2. [VOLUMEN] = L3 TT=T
3. [VELOCIDAD] = LT-1
4. [ACELERACION] = LT-2 3 Si una frmula fsica es
5. [FUERZA] = MLT-2 dimensionalmente correcta u
6. [TRABAJO] = MLT-2 homognea, todos los trminos de
7. [POTENCIA] = ML2T-3 dicha ecuacin deben ser
8. [PRESION] = ML-1T-2 dimensionalmente iguales.
9. [CALOR] = MLT-2
10. [ENERGIA] = MLT-2 As: sea la frmula fsica:
11. [TORQUE] = MLT-2
12. [MOMENTUM LINEAL] = MLT-1 P+Q=RS
13. [IMPULSO] = MLT-1
14. [CAUDAL] = L3T-1 [P] = [Q] = [R] = [S]
15. [VELOCIDAD ANGULAR] = T-1
16. [ACELERACION ANGULAR]= T-2 Ejemplos de Aplicacin
17. [CARGA ELECTRICA] = IT
18. [RESISTENCIA ELECTRICA] 1. Si: x = 8mg log 12
= MLT-3I-2 Donde
19. [POTENCIAL ELCTRICO] m: masa
= MLT-3I-1 g: aceleracin de la gravedad
20. [CAPACIDAD ELCTRICA] Qu dimensiones tendr x?
=M-1L-2T4I
Solucin:
PROPIEDADES DE LAS ECUACIONES [x] = [8mg log 12]
DIMENSIONALES Recordemos que:
[8] = 1 [log 12] = 1
1 Todo nmero expresado en Luego, tendremos:
cualquiera de sus formas tiene [x] = [mg]
como dimensin a la unidad.
[x] = MLT-2
O
FSICA
Observacin Importante
2. Si:
1 A Los exponentes de una magnitud
X= siempre son nmeros
2 vt cos
Ejemplos:
Donde:
* Son correctas:
A = rea; t = perodo; h; F2t-4; t5; Lcos 30
v = volumen. * No son correctas:
hm; Fq, Mt gF; n
Hallar las dimensiones de x * Las siguientes expresiones podran
ser correctas, siempre y cuando
Solucin: x sea un nmero
- M3x
A F4xL; ser correcta si XL es
[x ] = 1
-
2 vt. cos un nmero

En ste caso se cumple:


Recuerde: 1
[XL] = 1 [x] = = L-1
L
1
2 = 1 [] = 1 Luego: M2xL = M

4. Halle las dimensiones de K en la
[cos ] = 1 Luego: siguiente ecuacin dimensionalmente
A L2 correcta.
[x] = = 3
A .f
vt L .T 3AK = h g
. cos . v
L
[x] = 3
= LL 3T 1 [x] = L-2T-1 Donde:
LT
h : altura ; f : frecuencia
3. Si: g : gravedad; v : velocidad

3 (3a a ) 2 Solucin:
P=
( v + 6v) log5
* Analizamos el exponente
f g
A. = 1 [A ] =
Donde:
a = aceleracin; v = velocidad g f
Hallar las dimensiones de P
2
Solucin: [A] = LT 1
= LT 1
T
De la 2 propiedad:
[3a - a] = [a] = LT-2 Luego, en la expresin inicial:
[6v - v] = [v] = LT-1
Ak = h-1 . v
Luego:
a 2 LT 2
[P] = =
(=
)
2
L2T 4
LT-1 [K] = L-1 . LT-1

v LT 1 LT 1 [K] = L-1
[P] = LT-3

O
FSICA
Completamos el primer miembro
PROBLEMAS RESUELTOS para tener las mismas magnitudes
del segundo miembro, as:
1. Hallar [x] y [z] en la siguiente LT-1 = L 2 x -y T2y
2

ecuacin D.C.
( w + w log 2) + z 3 Igualamos exponentes:
tg = De T : 2y = -1
(g + gsen) x Y=-
De L :
Donde: -2x - y = 0 - 2x = y
w : peso; g = gravedad - 2x = -
x =
Solucin x=
Luego
Aplicamos la 1 propiedad: 1
(w + w ) + z w + z x y = -
1= = 2
(g + g ) x gx
(x - y) = 1
Luego:
gx = w + z
3. La ecuacin mostrada es D.C.
[gx] = [w] = [z] Hallar (x + y)

(1)
g = Vtx (4 + k y-x
)
De (1):
[z] = MLT-2 Donde:
t = tiempo; v = velocidad
Adems : g = gravedad
Solucin
[gx] = [w]
Como es D.C., tenemos:
w MLT 2 [4] = [Ky-x] = 1
[x] = =
g
LT 2 Es decir: y x = 0 y = x

[x] = M Entonces:
[g] = [ Vtx]
2. Qu valor tiene (x-y), si la LT-2 = LT-1 Tx = LTx-1
siguiente ecuacin es D.C.?
Igualando exponentes:
x 1 = -2 x = -1
f = k 2 x .g y
2
Luego y = -1

Donde: (x + y) = -2
: longitud; g: gravedad
k : constante numrica 4. Hallar si la ecuacin mostrada
es D.C.
Solucin
y = (x + 3 ) y sen
[f] = [ k 2 x .g y ]
2 v 1
ta a
T-1 = 1 . (L )
2 x 2
. (LT-2)-y
x
Donde:
T-1 = L 2 x . L-y T2y
2

t = tiempo; v = velocidad;
T-1 = L 2 x = aceleracin angular
2
-y
. T2y
O
FSICA

Solucin OPERACIONES BASICAS CON LOS


* [x] = [3 ] = T -2
VECTORES
v LT 1
* = [y] [ y] = 2 Debemos tener presente que para
x T
realizar operaciones con vectores, estos
deben ser de la misma naturaleza.
[y] = LT
I. Suma de Vectores
Luego, en la expresin original: Consiste en reemplazar a un conjunto de
ta a y = ()-1 y sen
vectores por uno solo llamado vector
1
a resultante ( R ).
Ta y = (T-2)-1 y sen
1
a Cmo determinamos la resultante de
Ta y = T2 ysen dos vectores?

Igualando exponentes: Rpta. Se debe tener en cuenta los


1 siguientes casos:
a=2; = sen
2
1. Para dos vectores con el mismo
sentido:
= 30
La resultante se obtiene sumando
los mdulos de los vectores
ANLISIS VECTORIAL
Ejemplo:
Vector: Es un ente matemtico que se
caracteriza porque tiene mdulo, A = 4u R = 7u

direccin y sentido. Un vector sirve para
representar a las magnitudes fsicas
vectoriales. B = 3u

Los vectores se pueden representar A esta resultante se le conoce


como Resultante Mxima (Rmax)
grficamente mediante un segmento de
recta orientado. As:
R=A+B
y

2. Para dos vectores con sentidos


Lnea de accin
opuestos
vI
:I A = 4u R = 1u
ulo
od
M
v Sentido
B = 3u

Direccin
R=A-B
x

Notacin: En este caso se obtiene restando


los mdulos de los vectores
* v : se lee vector v * A esta resultante se le conoce
* v : se lee mdulo del vector v como RESULTANTE MINIMA
(RMIN)

O
FSICA
3. Para dos vectores 2. El vector resultante ( R ) se
perpendiculares: obtiene uniendo el origen del
primer vector con el extremo del
ltimo vector
A = 3u R
Por ejemplo:
Para los vectores dados, halle el
mdulo de la resultante.
B = 4u
R= A +B
2 2

c=6

10
R= 32 + 42

A=
B=2
37
R = 5u
Solucin
Colocamos los vectores uno a
En este caso la resultante se
continuacin de otro.
obtiene aplicando el teorema de
Pitgoras. B=2

A 2 + B2

0
R=
=1
A C=6
4. Para dos vectores que forman un
37
ngulo cualquiera 6 2
R

A R El vector resultante se obtiene


uniendo el origen del primer
vector con el extremo del ltimo
vector. Luego:
B
Observe que en este caso se R=8
trazan paralelas a los vectores por Diferencia de dos Vectores
sus extremos. La unin del origen Los vectores que se van a restar
de los vectores con la interseccin se unen en un origen comn,
de las paralelas es el vector luego el vector diferencia se
resultante. obtiene uniendo los extremos de
El mdulo de ste vector los vectores. El vector diferencia
resultante se obtiene as: seala hacia el minuendo.

R= A 2 + B2 + 2AB Cos A D

A
Mtodo del Polgono
Nos permite determinar la resultante de
B B
varios vectores:
D = AB
Procedimiento
1. Trasladamos los vectores y los Su mdulo:
colocamos uno a continuacin de
otro (extremo de un vector en el D = A 2 + B2 2AB cos
origen del otro)
O
FSICA
Ejemplos de Aplicacin B C
1. La resultante mxima de dos
vectores de mdulos iguales es 20.
A D
Hallar la nueva resultante cuando dichos
vectores estn formando 120 entre s.
F E
Solucin: Luego; sumamos: AC + CD = AD
Sea los vectores ayb AE + ED = AD
R = 2 (AD)
Tales que: a = b =m Pero AD = 4u
Luego, Rmax = a + b
Luego R = 8u
Rmax = 2m
Por dato: 2m = 20 3. Dados los vectores mostrados,
m = 10
determinar P 2Q
Luego, cuando forman 120:

5
R

P=
Q=3
120 68 15
10
Solucin.
10
Unimos los vectores por sus orgenes.

R= 102 + 102 + 2(10)(10) cos120


=5

53 6
P

1 Q=
R= 10 2 + 10 2 + 2(10) 2 15 2
2
R = 10 D= 52 + 62 2(5)(6) Cos53

Conclusin D= 25 + 36 36 D=5
Dos vectores de igual mdulo que
formen 120 entre si originan una DESCOMPOSICION RECTANGULAR
resultante de igual mdulo que los DE UN VECTOR
vectores.
Consiste en reemplazar un vector por
2. La figura mostrada es un otros dos, de tal forma que stos sean
hexgono regular de lado 2u. Halle mutuamente perpendiculares.
el mdulo del vector resultante. y y
v
B C v
vy

x x
A D vx

F E Vx = V cos Vx = V Cos

Solucin Vy = V sen Vy = V sen


Trasladamos los vectores hacia los
lados que son paralelos a dichos Adems: Tag= Vy
vectores, as: Vx

O
FSICA
Ejemplos de Aplicacin 2. Halle la medida del ngulo
para que la resultante se
1. Hallar el mdulo de la resultante. encuentre en el eje x

120 10
90

53 6 30
37
16

Solucin:

120 Sen 53 Solucin


90 sen 37
37 53 10
10 sen
90 Cos 37 120 Cos 53
6
* Hallamos RH 16 cos 60 10 cos
60
RH = 120 cos 53 - 90 cos 37
3 4
RH = 120 x - 90 x
5 5 16 sen 60

RH = 0
Como la resultante est ubicada
sobre el eje x, entonces en el eje
* Hallamos RV vertical, la resultante debe ser
igual a cero:
RV = 90 Sen 37 + 120 sen 53
3 4 Luego:
RV = 90 x + 120 x
5 5 Ry = 0
10 sen - 16 cos 60 = 0
RV = 150
5 sen = 8 cos 60
5 sen = 8 x = 4
Luego la resultante total se
4
obtiene as: sen = = 53
5
R= R 2H + R 2v

R= 0 2 + 150 2 R = 150

O
FSICA

OBJETIVO * d = Desplazamiento
Describir geomtrica y matemticamente
el movimiento mecnico y conocer sus * d = rf ro (cambio de posicin)
leyes y propiedades; pero sin considerar * d = d : distancia: mdulo de
a las causas que lo determinan. En el
estudio de la cinemtica estableceremos desplzamiento
la relacin que existe entre las * e: Recorrido (Longitud de la
magnitudes tales como; desplazamiento, trayectoria)
velocidad y aceleracin.
VELOCIDAD ( V )
MOVIMIENTO MECNICO:
Se define como el cambio continuo de Es una magnitud fsica vectorial que nos
posicin que experimenta un cuerpo expresa la rapidez con la cual un mvil
respecto de otro tomado como cambia de posicin.
referencia. El cambio de posicin se puede dar en
As, por ejemplo: un intervalo de tiempo o en un instante
de tiempo.
B Unidad en el S.I.: (m/s)
A
C
- Velocidad Media ( Vm )
Se evala entre dos puntos de una
trayectoria y se define como la razn
Para A: C, experimenta movimiento
mecnico. entre el desplazamiento del cuerpo ( d ) y
Para B: C, no experimenta movimiento el intervalo de tiempo transcurrido (t).
mecnico.
De esto podemos concluir que el y
movimiento mecnico no es absoluto,
sino que es relativo, pues depende del
sistema de referencia t >>o
t
vm

ro d
ELEMENTOS DEL MOVIMIENTO
MECANICO
Y
mvil

trayectoria rf
d
x
d
Vm =
ro
e
rf
t
Note que la Vm y d con codirigidos.
(Colineales y tienen la misma direccin)

X - Velocidad Instantnea ( V )
Observador

Es una magnitud vectorial que


* ro = Posicin inicial caracteriza el movimiento mecnico de
* rf = Posicin final un punto, en un instante de tiempo t.

O
FSICA
El vector velocidad instantnea se grafica
tangente a la trayectoria y nos indica la Aplicacin 01:
direccin del movimiento. Determine el mdulo de la velocidad
media de cierto mvil que recorre el
y trayecto ABC con una rapidez constante
v de 5 m/s
dT
c

m
d

15
120

x A 20 m B
Cuando t 0, el desplazamiento es
tangente a la trayectoria.
Solucin:
d C
V = lim
t
20
t o t AB = = 4s
d 5
15 m
15
t BC = = 3s
5
0
Rapidez V
12

A 20 m B t = 7s
Es el mdulo de la velocidad instantnea
Ejemplo:
Ley de Cosenos
VB d= 20 2 + 15 2 2(20)(15)(cos 120 )
VA
B
C 1
d= 400 + 225 2(300)
A 2
VC
d= 925 d = 5 37 m

V = 5 m/s () Luego:
sentido
rapidez d 5 37 m
Vm = =
t 7 s

Movimiento con Velocidad Constante

Si V es constante, entonces su
mdulo (rapidez) y su direccin es
constante. Luego, esto implica que la
trayectoria del mvil necesariamente
ser Rectilnea. A este movimiento se

O
FSICA
le denomina MOVIMIENTO RECTILINEO La grfica es una recta paralela
UNIFORME (M.R.U.) al eje de los tiempos.
El rea bajo la grfica nos da el
En todo M.R.U. se cumple que: espacio recorrido.

d=Vxt Ao t = eot

Grfica x vs t
Ejemplo:
x (m)
Supongamos un mvil que se desplaza
horizontalmente con velocidad constante
y rapidez 4 m/s
xf
xf - x o
t = o t = 1s t = 2s xo
1s 1s t t(s)
Obs. t
7m 4m 4m La grfica es una recta
4m inclinada respecto de la
horizontal.
Xo = 7 m 4m
La tangente del ngulo de
X1 = 11 m inclinacin nos indica la
X2 = 15 m velocidad constante del mvil
xf xo
Tg =
t
Como: d=Vxt x = v.t
tg = V
tg = pendiente de la recta
xf x0 = V x t
Aplicaciones
x f = x 0 + V.t 1. En el instante t = 0, la posicin de
un mvil es xo=-4m y cuando
t=2s, X1 = 8m.Si el movimiento es
con velocidad constante; calcular
Ecuacin del M.R.U. la velocidad.
Solucin:
GRAFICAS EN EL M.R.U.
Grfica V vs t t = 0S t = 2S
............. .........
V (m/s) x
-4 x=0 +8

v Xo = - 4m Xf = + 8 m

Recordemos que:
A
xf = x0 + V x t
8 = -4 + V x 2
t (s)
0 1 2 t
V = 6 m/s ()

O
FSICA
2. Un ciclista durante 4 segundos
recorre con rapidez constante de ...........
5m/s hacia la derecha,
seguidamente regresa hacia la LT LOMN
izquierda con velocidad de 3m/s
durante 5s. Hallar el espacio
recorrido y el desplazamiento. dRECORRIDA

Solucin: dRECORRIDA = V x t
(LTUNEL + LOMNIBUS) = VOMN x t
30 + Lo = (35) (10)
X1 = 20 m
A
B Lo = 5m
5 m/s
3 m/s
4. Dos mviles estn separados
C inicialmente 700 m y parten al
encuentro con velocidades de 30
m/s y 40 m/s simultneamente.
X2 = - 15 m Calcular el tiempo que tardan en
d estar juntos

* e = x 1 + x 2 = 35m Solucin:

30 m/s t t 40 m/s
* d = x1 x 2
A A B B

* d = 20m 15 m
700 m

En este caso, aplicamos tiempo de


* d = 5 m() encuentro (te)
d
t = te =
VA + VB
3. Un mnibus tarda 10 segundos en
700m
pasar un tnel de longitud 30 m t= t = 10s
con una velocidad constante de 30m / s + 40m / s
3.5 m/s. Calcular la longitud del
mnibus ACELERACIN
Es una magnitud fsica vectorial que nos
Solucin; indica la rapidez con la que cambia la
* El mnibus ingresa al tnel velocidad de un mvil.
Tiene como unidad: (m/s)
LOMN LT
Aceleracin Media ( a m )
Mide la rapidez de cambio de velocidad
........... en un intervalo de tiempo

V Vf Vi
* El mnibus atravesar al tnel am = =
cuando salga completamente t t

O
FSICA
Solucin:
y V1
4 m/s
V1
am
t
8 m/s
V2 V2 V = V2 V1
v
x
o

La a m y V tienen la misma V = 82 + 42
direccin V = 4 5 m / s

Aceleracin Instantnea ( a ) Luego:


Mide la rapidez de cambio de velocidad m
en un instante de tiempo. V 4 5 s
am = =
t 2 s
y
am = 2 5 m/s
v

MOVIMIENTOS CON ACELERACION


a
CONSTANTE

I. Movimiento Rectilneo con


x Aceleracin Constante
La a apunta hacia la
concavidad de la trayectoria Primero, analicemos: Qu significa
a=5m/s?
Si : t 0 a = lim am Rpta. Significa que el mvil en cada
t o segundo cambia su rapidez en 5m/s

Dado que la rapidez puede aumentar o


Ejemplo de Aplicacin
Determine el mdulo de la disminuir, entonces se tiene que:
aceleracin media entre A y B, si se
emplea un tiempo de 2 segundos. Movimiento Acelerado

4 m/s a
A
v

Movimiento Desacelerado

B a
8 m/s
v

O
FSICA

Supongamos una pelota que se Aplicaciones


desplaza con rapidez inicial de 1. Un mvil parte de la posicin
4m/s y acelera con 2m/s Xo = -20m con una velocidad de
constante. 5m/s. Hallar la posicin y espacio
recorrido luego de 5 segundos, si
1s 1s 1s su aceleracin es 4m/s.

4 m/s 6 m/s 8 m/s Solucin


2 m/s 2 m/s 2 m/s 10 m/s Recordando la ecuacin de la posicin:
A D
d1 = 5m B d2 = 7m C d3 = 9m
xf = x0 + d
dTOTAL = 21m at 2
xf = xo + Vot +
2
Observe que:
La trayectoria es rectilnea 4x5
Los cambios en la velocidad xf = -20 + 5(5) +
2
son uniformes, por esto se
llama Movimiento Rectilneo
d
Uniformemente Variado
(M.R.U.V.)
xf = +55 m
La V es D.P. al tiempo
transcurrido.
Luego, el espacio recorrido ser:
Del Grfico:
Tramo AB : t = 1s V = 2m/s e = d = 75m
Tramo AC : t = 2s V = 4m/s
2. Una esferita inicia su movimiento
Tramo AD : t = 3s V = 6m/s con aceleracin constante
recorriendo en el segundo
Note, adems que los recorridos en segundo 3m. En cunto tiempo
segundos consecutivos se diferencian en habr recorrido los primeros 16m?
el valor de la aceleracin.
Solucin

Ecuaciones del M.R.U.V. Para calcular el tiempo, aplicamos:


at 2
1. Vf = Vo + at d = Vot +
2. Vf = Vo+ 2ad
2
at 2 at 2
3. d = Vot + 16 = ..............(1)
2 2
Luego, calcular la aceleracin a partir de
V + Vf la distancia en el 2 segundo:
4. d = o . t
2 a
d2s = Vo + (2 x 2 - 1)
a 2
5. dn.seg = Vo + ( 2 x n 1)
2 a
3= x 3 a = 2 m/s
2
Nota:
- Use signo (+) si V aumenta
- Use signo (-) si V disminuye En 1:

O
FSICA
t = 4s Ejm:

Grficas en el M.R.U.V.
V (m/s)
1. Posicin vs tiempo ( x - t)

a
-5 m/s
X (m)
Parbola
tg = (+)
t(s)
X1 A 0

V(m/s)
-5

X0
a 10 m/s
t(s)
o t1 10

tg = (-)
VA = tg
t(s)

2. Velocidad vs tiempo ( v -t) Sea la grfica siguiente:

V (m/s)
V (m/s)
8
Vf

A1
2 3
A t(s)
V0 A2
t(s) -4
o t1

A1 : Recorrido hacia la derecha.


a = tg
A2 : Recorrido hacia la izquierda

eT : A 1 + A 2 (Recorrido)
e=A

d : A 1 A 2 (Distancia)

O
FSICA
3. Aceleracin vs tiempo (a-t) Solucin:

m/s V (m/s)
a

5
A1
a
10
t (s)
A 6
A2
t(s) -10
0 t1

V = A
d = A1 A 2

d = 30 40
V = Vf Vo

d = 10 m
Aplicaciones

1. Se muestra la grfica (V - t) de
una partcula que se mueve sobre
el eje x. Halle el mdulo del
vector desplazamiento.

V (m/s)

t (s)
o 6 10

O
FSICA

CONCEPTO que emplea en bajar entre los


Es un movimiento ideal, que se verifica mismos puntos.
en las inmediaciones de la superficie
terrestre. Durante este movimiento de tsub = tbaj
cada libre, la nica fuerza que acta
sobre el cuerpo, es la fuerza de gravedad El cuerpo alcanza su altura
o peso del cuerpo. mxima cuando la velocidad
En este movimiento todos los cuerpos, con que se lanz sea igual a
experimentan una aceleracin constante cero. Es decir, en el punto ms
llamada aceleracin de la gravedad (g). alto su velocidad es igual a
Valor promedio = 9.8 m/s cero.
Valor prctico = 10 m/s
Se usar las mismas
Ejemplo: ecuaciones del M.R.U.V.
Consideremos el lanzamiento de una
esfera verticalmente hacia arriba a) Forma escalar:
(g=10m/s)
V=0 - Vf = Vi gt
gt 2
- h = Vit
1s 2
1s - Vf = Vi 2 gh
10 m/s
g h Vi + Vf
- =
10 m/s t 2
1s 20 m/s 1s Donde:

20 m/s (+) V aumenta


(-) V disminuye
1s
30 m/s b) Forma vectorial:

- Vf = Vi + gt
Observamos que:
Se trata de un M.R.U.V. con
gt 2
trayectoria vertical
- h = Vi t +
La velocidad de subida (VS) y la 2
velocidad de bajada (VB) para
puntos que estn al mismo
Vf = Vi + 2g.h
2 2
nivel, tiene igual valor. -

VSUB = VBAJ V +V
- h = o f . t
El tiempo que emplea en subir 2
entre dos puntos es igual al

O
FSICA
En este caso deber tener en cuenta - Luego el tiempo: tAB
el sentido de la magnitud que va a Vf = Vo gtAB
reemplazar. As: 30
tAB = tAB = 3g
10
(+) ; (-)
Analizamos el tramo BD:
EJEMPLOS DE APLICACIN Para este tramo utiliza un tiempo
de 7s. (tAB + tBD = 10s)
1. Hallar h si el tiempo total de
vuelo es de 10 segundos. Luego:
(g=10m/s) gt 2 BD
hBD = vEtBD +
2

10(7) 2
hBD = h BD = 245m
g 2
Vo = 30m/s
Por lo tanto:

h = hBD hAB
h

h = 200 m

Solucin: Forma Vectorial:

Forma Escalar: El objeto se lanza en a y llega al punto


* Analizamos el tramo AB: C, luego experimenta el
- Recuerda que en B V = 0 desplazamiento h AC ,
- Calculamos hAB

B B

30m/s
Vo = 30m/s
A C
A

h
hAC
D
C

Vf = Vo - 2 g hAB
0 = 30 - 2(10) hAB

hAB = 45m

O
FSICA
Luego
gt 2 Solucin:
h AC = VA .t + El tiempo que permanece en el
2
aire es equivalente al tiempo que
(10)(10) 2 tarda en subir hasta el punto ms
- h = 30(10) +
2 alto y el tiempo que tarda en
- h = 300 - 500 regresar.
- h = -200

hAC = 200 m

2. Se lanza un objeto verticalmente


hacia abajo desde cierta altura con
una velocidad Vo. Si luego de 5
ts tb
segundos impacta en el suelo con
70 m/s. Calcular con qu velocidad
se lanz dicho objeto. (g = 10 40 m/s
m/s)
Solucin:

g Vo t(aire) = ts + tb .... 1

5s En la subida
Vf = Vo gts

40
ts = ts = 4s
10

Adems:
70 m/s ts = tb = 4s
Vf = Vo + gt
70 = Vo + (10) (5) Reemplazamos en 1
t(aire) = 4s + 4s
Vo = 20 m/s

t(aire) = 8s
3. Halle el tiempo que la esferita
permanece en el aire. (g=10m/s)
Formula prctica:

Vo
tsub =
g
luego:

Vo = 40 m/s

O
FSICA
2Vo Al proyectar se observa que:
tTOTAL = t(aire) = 2ts =
g
1. En el eje x:
No existe aceleracin, entonces en
MOVIMIENTO PARABLICO DE
CADA LIBRE esta direccin la velocidad Vox
se mantiene constante, por lo
Si consideramos el caso de una pelotita tanto el mvil desarrolla un M.R.U.
que es lanzada de la siguiente manera:
2. En el eje y:

En esta direccin la velocidad Vy


experimenta cambios de manera
uniforme debido a la aceleracin
de la gravedad g, por lo tanto el
mvil experimenta en sta
proyeccin un M.V.C.L.
Se observa que dicha pelotita describe
como trayectoria una lnea curva. Pero al Observacin:
despreciar la accin del aire, tal
trayectoria es una parbola y por ello al Si bien el anlisis se hace
movimiento se le llama parablico. independientemente en cada eje,
Adems durante el desarrollo de este esto ocurre simultneamente, es
movimiento, sobre la pelotita acta decir, los intervalos de tiempo que
nicamente la fuerza de gravedad Fg = transcurren para cada direccin
mg y por ello tal movimiento es de son iguales.
cada libre, en consecuencia el
movimiento descrito es un movimiento De la figura se puede obtener la
parablico de cada libre (M.P.C.L.) siguiente relacin:

Para analizar el M.P.C.L. se proyecta tal


movimiento en la direccin vertical y en t(vuelo) = tproyeccin = tproyeccin
(ABC) Horizontal Vertical
la direccin horizontal. As:
(AMC) (ts + tb)

y
B M.P.C.L. M.R.U. M.V.C.L.
V1 Vx = Vox
Vox
Vox
Voy Vo HMAX
V1
Vox
C
A
Vox M X

x Voy

d : Alcance Horizontal

O
FSICA
EJEMPLOS DE APLICACION
X = 20m
1. De la parte superior de un edificio Observacin:
de 20 m de altura, se lanza Si quisiramos determinar la
horizontalmente una pelota con rapidez de la pelota despus de
una rapidez de 10 m/s ser lanzada, tendra que usarse el
Determine el alcance horizontal teorema de pitgoras.
que logra la pelota cuando Por ejemplo, en el punto P, Vx
impacta en el piso. (g = 10m/s) y Vy son respectivamente
perpendiculares, luego:
Solucin:
1. Graficamos

A Vx = 10 m/s Vp = Vx2 + Vy2

P Vx = 10 m/s
2. Desde la azotea de un edificio se
lanza horizontalmente un cuerpo
H = 20 m
Vy con una rapidez de 5m/s.
Determine su alcance horizontal y
la altura que desciende 2
segundos despus de su
B
lanzamiento.
M
x
Nos piden x Solucin:

2. Recordemos 1. Graficamos:

tAB = tAM = tMB = t A Vx = 5 m/s


Esto significa que si determinamos
el tiempo en el eje y lo hacemos
tambin en el eje x. Segn los h t = 2s
datos, conviene analizar el eje y
para determinar el tiempo. M B
x
3. Eje y: (A M) Voy = 0

gt 2
h = Voy t +
2
2
10 t Nos pide x y h
20 = 0 +
2
t = 2s 2. Eje x: (M B)
dMB = Vx . t
4. Eje x: (M B) x = (5) (2)
Usamos M.R.U.
Luego:
dMB = Vx . t x = 10 m

x = 10(2)
3. Eje y (A M)
(Contine Ud. la solucin)

O
FSICA
MOVIMIENTO CIRCUNFERENCIAL

Qu es el movimiento
circunferencial?
Para responder, analicemos lo que
ocurre cuando una piedra atada a una
cuerda gira en un plano vertical. Se
observa:
(C)

O R
(B)

R
L
(A)

1. Respecto al centro (0) la piedra


cambia continuamente de posicin
(A,B,C,....). Si unimos todas las
posiciones por las que pasa la
piedra obtenemos una lnea curva
denominada circunferencia.

2. El vector que parte del centro O


y ubica a la piedra en todo
instante se denomina radio vector
( R ) el que describe un ngulo
central () y una superficie
denominado crculo. Si slo
consideramos la trayectoria que
describe la piedra diremos que
sta desarrolla un MOVIMIENTO
CIRCUNFERENCIAL.

Por lo anterior, se dice lo


siguiente:

O
FSICA
El MOVIMIENTO CIRCUNFERENCIAL t = 1s = (/6) rad
es un fenmeno fsico que se manifiesta
cuando simultneamente un cuerpo t = 2s = 2(/6) rad
cambia de posicin y de ngulo central
t = 3s = 3(/6) rad
respecto de un punto fijo denominado
centro, permitindole describir una Se observa que el ngulo es
circunferencia como trayectoria. directamente proporcional al tiempo
transcurrido.
Para medir la longitud entre 2 posiciones
se utiliza una magnitud denominada
es D.P. a t. Ello implica que:
longitud de arco o recorrido lineal (L), la

cual est relacionado con el ngulo = cte. donde la constante es la rapidez
t
barrido () y el radio de giro (R)
angular (), la cual es el mdulo de la
velocidad angular ( )
L = R
Qu es la velocidad angular ( )?
Es una magnitud fsica vectorial que
en radianes (rad)
expresa la medida de la rapidez de
R en metro (m) cambio del desplazamiento angular.

L en metro (m)

Movimiento Circunferencial Uniforme


(M.C.U.)

Es aquel movimiento donde una partcula


describe una trayectoria circunferencial,
experimentando en intervalos de
tiempos iguales, recorridos lineales
iguales y adems el radio vector barre
ngulos iguales.

Si la es constante, el mdulo de esta


velocidad se evala as:
t=0

/6 =
/6 t
rad. T = 3s
t = 1s 6 Unidad:
t = 2s
radian rad

Considerando (t) el tiempo transcurrido segundo s
y el ngulo barrido, tenemos del
grfico: : Angulo barrido
: Rapidez angular

O
FSICA
Como forma prctica para indicar la La velocidad lineal o velocidad
direccin de la velocidad angular se tangencial (VT) es constante en el
utiliza la regla de la mano derecha, la M.C.U.?
cual consiste en girar los 4 dedos juntos,
menos el pulgar en el sentido del No!, porque su direccin cambia
movimiento; luego de ello el dedo pulgar continuamente, por tal motivo en ste
indica la direccin de la velocidad movimiento existe aceleracin,
angular ( ), tal como se muestra en la denominada aceleracin centrpeta a cp ( )
figura.

Como en cada instante el mvil gira en


un mismo sentido y en cada segundo el Qu mide la aceleracin centrpeta
radio vector barre un ngulo constante, (a ) ?
cp
entonces en el M.C.U. la velocidad
angular es constante ( ) (tanto en Mide la rapidez del cambio de la
valor como en direccin) direccin de la velocidad tangencial cuyo
mdulo se determina para cada instante
En el M.C.U. qu ocurre con la rapidez
mediante:
lineal o rapidez tangencial (VT)?
Debido a que en intervalos de tiempos
iguales los ngulos barridos son iguales,
entonces las longitudes de arco son
VT2 unidad
iguales (LAB = LBC); por ello la rapidez acp = ; acp = 2 R
lineal es constante (VT) R m / s2

VT
C t = 2s

A
R
t = Os
R

VT

VT
B
t = 1s

Pero : L =R ....(**)

R
Reemp. (**) en (*): VT =
t

VT = R Relacin entre y VT

O
FSICA
y la direccin de la a cp en todo instante
est dirigida hacia el centro de la
circunferencia. Es decir:

acp
acp

VT VT

O
FSICA

Es una rama de la Mecnica, cuyo La interaccin mecnica puede


objetivo es analizar las condiciones que efectuarse entre cuerpos en contacto
deben de reunir un conjunto de fuerzas directo, as como entre cuerpos
que actan sobre un cuerpo o sistema separados.
para que lo mantenga en equilibrio.
Qu es una fuerza?
A qu llamamos interaccin? Veamos, en el ejemplo anterior, si
Para entender este concepto analicemos quisiramos saber con que intensidad
el siguiente caso: interactan los cuerpos entonces
usaremos una magnitud vectorial
Se lanza una pelota para que golpee al denominada Fuerza (F).
bloque, en reposo.
La fuerza tiene como unidad de medida
en el Sistema Internacional (S.I.) el
Newton (N).
Reposo
Observacin:
El movimiento mecnico de un cuerpo es
consecuencia de la interaccin con otros
cuerpos.
Segn sea la naturaleza de las
interacciones, las fuerzas se clasifican
La esfera en:
impacta en
el bloque 1. Fuerzas Gravitacionales
Tienen como origen o causa a la
masa de los cuerpos y son
Luego del golpe, el bloque que se siempre de atraccin. Por ejemplo
encontraba en reposo adquiere el peso.
movimiento mientras que el movimiento
de la pelota es frenado. 2. Fuerzas Electromagnticas
Tienen como origen a las cargas
elctricas de los cuerpos en reposo
o en movimiento.
F2 F1 Las fuerzas son elctricas si las
cargas elctricas estn en reposo,
y sern magnticas si las cargas
estn en movimiento.

3. Fuerzas Nucleares.
Interaccin Estas fuerzas unen los protones y
los neutrones en el ncleo atmico
De esto podemos deducir que cuando un y es de corto alcance.
cuerpo acta sobre otro, puede modificar
su estado mecnico. 4. Fuerzas Dbiles:
Estn fundamentalmente asociadas
A esta accin mutua entre dos cuerpos a la descomposicin de ncleos
se denomina interaccin. radiactivos.

O
FSICA
Las fuerzas que con frecuencia 2. Fuerza de Tensin (T)
usaremos en esttica estn Se manifiesta en las cuerdas,
comprendidas entre las dos usadas para colgar o suspender
primeras de la clasificacin. cuerpos en el aire, para jalar
cuerpos, etc.
FUERZAS USUALES:

1. Fuerza de Gravedad (Fg)


Llamada tambin fuerza
gravitacional, es aquella con la T
cual se atraen dos cuerpos en el
universo, esto se debe a la T
interaccin gravitatoria entre los
cuerpos.

Por ejemplo, si soltamos una


piedra, notaremos que sta cae
dirigindose hacia la tierra. De La fuerza de tensin tiene la
esto deducimos que la tierra atrae misma direccin de la cuerda
a la piedra (lo jala hacia su centro) sobre la que acta.
ejercindole una fuerza a la que Para una cuerda ideal (de masa
llamaremos Fuerza de Gravedad. despreciable), el modulo de la
tensin es el mismo en cualquier
V=0 punto de la cuerda.

g Ejemplo: Una caja de 3 kg es


sostenida mediante una cuerda tal
m como se muestra. Grafique la
Fg fuerza de tensin y determine su
mdulo (g = 10 m/s)

Solucin.

m : masa del cuerpo


T
g : aceleracin de la gravedad

Cuando el cuerpo est prximo a


la superficie terrestre, el valor de
la fuerza de gravedad se calcula
as: Fg = 40N
Fg = m.g

La fuerza de gravedad se grafica


vertical y hacia abajo, en un punto
llamado centro de gravedad (C.G.)
el cual, para cuerpos homogneos
coincide con su centro geomtrico.

O
FSICA
Dado que la caja no cae, entonces Experimentalmente se demostr
concluimos que la fuerza hacia que:
arriba y hacia abajo deben ser A mayor x, mayor Fe
igual mdulo; luego: A menor x, menor Fe

T = 40N Fe
= cte = K
x
3. Fuerza Normal (FN) Fe = KX
Llamada tambin fuerza de
contacto, es una fuerza de
reaccin que se manifiesta K = Constante elstica del resorte
siempre que haya contacto entre (N/m; N/cm)
dos superficies. X = Elongacin del resorte
Lo = Longitud natural del resorte
La lnea de accin de sta fuerza (cuando no est deformado)
es perpendicular a las superficies
de contacto. Nota: el valor de K depende del
material del resorte y de su
longitud natural.

FN FN 5. Fuerza de Rozamiento o de
Friccin (fr)
Seguramente alguna vez usted
habr intentado arrastrar un
bloque de cierto material, y habr
notado que no resbale.

FN
V=0
El bloque no
4. Fuerza Elstica (Fe)
Es una fuerza interna que se resbala
manifiesta en un cuerpo elstico
(Resorte, liga) cuando es
deformado por estiramiento o Esto se debe a que tanto la
compresin. superficie del bloque como el piso
presentan asperezas (rugosidades)
Por ejemplo, suspendemos un y por ello se manifiesta una
bloque de un resorte. oposicin al deslizamiento del
bloque, surgiendo as una fuerza
que recibe el nombre de fuerza
de rozamiento.
Lo En el ejemplo:
T
Fe
X fr

FN

O
FSICA

FN : fuerza normal Luego:


R : Reaccin del piso sobre el fsmax = s . FN
bloque

Luego: Donde:

s : Coeficiente de rozamiento
R = f r + FN
2 2
esttico (Adimensional)

Nota: Adems:
Cuando un bloque resbala o
intenta resbalar sobre una s = tg
superficie, la fuerza total (R) sobre
el cuerpo es inclinada respecto de Donde:
la superficie de contacto y para
facilitar el anlisis se descompone : Angulo mximo que se puede
en una fuerza normal (FN) y una inclinar la superficie de modo que
de rozamiento (fr). el bloque an no deslice.

CASOS PARTICULARES 2. Fuerza de Rozamiento Cintico


(fc)
1. Fuerza de Rozamiento Esttico Esta fuerza se manifiesta cuando
(fs) las superficies en contacto deslizan
Esta fuerza se manifiesta cuando una respecto de la otra. Su valor
las superficies intentan resbalar es prcticamente constante.
pero no lo logran.

Por ejemplo; si analizamos al fc = c . FN


bloque apoyado sobre el plano
inclinado rugoso: V
Aumentamos el
ngulo de inclinacin
fc FN
Inicialmente
V=0 V=0

fs fs
c = Coeficiente de rozamiento
FN FN cintico (adimensional)
>

Nota:
Entre dos superficies en contacto
El bloque aumenta su tendencia a existen dos coeficientes de
resbalar luego, tambin aumenta rozamiento (s y c) de modo
fs de modo que en algn que: s > c.
momento el bloque estar a punto
de deslizar (Movimiento
inminente). En este instante, la
fuerza de rozamiento esttico
alcanza su valor mximo (fsmx)

O
FSICA
DIAGRAMA DE CUERPO LIBRE
(D.C.L.)

Llamado tambin Diagrama de Fuerzas


Fg
es aquel donde se grafica todas las
fuerzas que actan sobre un cuerpo o
sistema. Para efectuar un D.C.L. tenga FNA
en cuenta lo siguiente:

1. Asle el cuerpo del sistema.


2. Grafique la fuerza de gravedad
3. Si el cuerpo est suspendido de FB
cuerdas, grafique la tensin.
4. Si el cuerpo est en contacto
con alguna superficie, grafique En este caso, por facilidad de
la fuerza normal (FN) por cada anlisis, es conveniente en la
contacto. articulacin B descomponer la
5. Si el cuerpo est en equilibrio y reaccin en dos, una componente
solamente acta 3 fuerzas, horizontal FBx y otra vertical
stas deben ser concurrentes, FBy. As:
necesariamente.

Ejemplos: A
* Efecte el D.C.L. de la esfera Fg
mostrada.
FNA

T B FBx

FB
FBy
FN

Equilibrio de Traslacin
Es cuando un cuerpo se encuentra en
Fg reposo o movindose con velocidad
constante, es decir sin aceleracin.
* Efecte el D.C.L. de la barra
Luego:
Liso
A Equilibrio de * Reposo
Traslacin * M.R.U.

Primera Condicin de Equilibrio


Articulacin Si un cuerpo se encuentra en equilibrio
de traslacin y sobre el acta un
B conjunto de fuerzas, se cumplir que:

FR = F = 0

O
FSICA
2. Hallar el coeficiente de rozamiento
Forma prctica () si el bloque A de 10 kg, est
a punto de deslizar (mB = 7.5 kg;
F () = F () g = 10m/s)

F () = F ()
A
Aplicaciones
1. Halle la fuerza que debe aplicar la
persona para mantener el bloque B
de 10 kg en la posicin mostrada.

Masa de la polea=2 kg; g=10 m/s Solucin:


De la figura observamos que la
fuerza que intenta poner en
movimiento al bloque A, es el peso
del bloque B.

100 N
fsmax

FN
Solucin:
* La fuerza que hace la persona en 75N
el extremo de la cuerda es el
mismo en toda la cuerda.
Esto ocasiona que entre el bloque
T T A y la superficie se manifieste la
fuerza de rozamiento esttico
mximo.

Luego:
fs max = 75N
100N

20N s . FN = 75N
s . 100N = 75N
Fy = 0

2T 120 = 0 s = 0.75
2T = 120

T = 60 N

O
FSICA
Momento de una Fuerza ( MFo )
Anteriormente hemos estudiado el efecto F = 5N

de deformacin de un cuerpo debido a F = 10N


una fuerza. En esta parte analizaremos
el efecto de rotacin causada por dicha
1m 2m
fuerza y las condiciones para el o o
equilibrio de rotacin.

Momento de una fuerza ( MF )


M oF = (10N)(1m) M fo = (5N)(2m)
Es una magnitud vectorial que sirve para
medir la intensidad con que una fuerza M oF = 10N.m M fo = 10N.m
causa o tiende a causar un efecto de
rotacin, sobre un cuerpo, respecto de
un punto o eje de giro. Ejemplo: Calcular el momento de la
fuerza F = 15N
Matemticamente:

F F = 15N
d Lnea de 5m
accin de F

A 37
O
Centro de
giro
Solucin

M Fo = F.d 5m F = 15N

A 37
F : mdulo de la fuerza F
4m
d : distancia o brazo de palanca M FA = F.d
unidad: (N.m)

M FA = (15N)(4m)
Convencin de signos:
(+): sentido de rotacin, antihorario
(-) : sentido de rotacin, horario M FA = +60 N.m

Nota:
Es posible producir un mismo momento Observacin:
de fuerza con una fuerza de mdulo Cuando la lnea de accin de una fuerza
pequeo, cuyo brazo sea grande; y con pasa por el centro de giro, su momento
de fuerza respecto de dicho punto es
una fuerza de mdulo grande pero de cero.
brazo pequeo.

O
FSICA
M RA = (15x 3) + (30 x 2)
A F
M RA = 45 60

M FA = 0 M RA = +15 N.m

Equilibrio de Rotacin: Observe que el momento


Es el estado mecnico en el cual un resultante no es nulo, por lo tanto
cuerpo no gira o lo hace uniformemente. la barra no est en equilibrio de
rotacin.
2 Condicin de Equilibrio: En este caso, la barra gira en
Cuando un cuerpo, sometido a varias sentido antihorario.
fuerzas no gira, se encuentra en
equilibrio de rotacin y se cumple que el Ejemplo: Hallar el momento
momento resultante respecto del centro resultante.
de giro, es nulo.
F1=20N
MR = 0

3m 2m
Forma prctica

M(+) = M(-) A

Ejemplo: F2=12N
Determine si la barra de la figura est en
equilibrio rotacional. Solucin:
M RA = M F1 + M
F2

F1=15N
1m M RA = (20.3) + (12 x 5)
F2=30N

M RA = 0
2m
A
La barra est en equilibrio de rotacin.
Solucin: Hallamos el momento
resultante. Equilibrio Mecnico
Llamado simplemente Equilibrio, es
aquella situacin en la que un cuerpo o
F1 sistema cumple las dos condiciones de
1m
equilibrio: (de traslacin y rotacin)
F2

F = FR = 0
EQUILIBRIO M = MR = 0
2m MECNICO
M = M + MA 2
R F
F1
A A

O
FSICA

CONCEPTOS PREVIOS
Pero al aplicarle una fuerza a la
Inercia: plataforma, esta se pone en
Es una propiedad de todos los cuerpos, movimiento mientras que la persona
por la cual stos tienden a mantener su por inercia se resiste a cambiar su
estado de reposo o de movimiento con movimiento y tiende a mantenerse en
velocidad constante. el mismo lugar.

La inercia que posee un cuerpo puede Segunda Ley de Newton


ser comparada con la de otro por medio
de su MASA, es decir que mientras ms Veamos cul es la condicin que se
masivo sea el cuerpo, mayor ser su debe cumplir para que un cuerpo
inercia. acelere o desacelere.

Cmo se manifiesta la inercia? Del grfico mostrado, el bloque se


mantiene en reposo sobre una
La inercia se manifiesta en los cuerpos superficie horizontal donde la fuerza de
como una resistencia que stos ofrecen gravedad es equilibrada por la reaccin
cuando se les trata de cambiar su del piso.
velocidad.
Fg
Para entender mejor esto, veamos los
siguientes casos: V
V=0
I. Plataforma con la persona encima
de ella avanza con velocidad Fg
constante. R

v Pero si la superficie no estuviese no


existira ninguna fuerza que equilibre a
la fuerza de gravedad, esto provocara
que la esfera caiga aceleradamente
(cada libre).
Cuando choca con el obstculo se
interrumpe el movimiento de la Conclusin:
plataforma pero la persona por
inercia continuar avanzando. Para que un cuerpo acelere (cambie su
velocidad) en l debe presentarse una
II. La plataforma inicialmente est en fuerza resultante no nula la cual
reposo. originara su aceleracin.

La experiencia demuestra que mientras


mayor fuese la fuerza resultante sobre
F el cuerpo mayor ser la aceleracin que
ste adquirir.

O
FSICA
La aceleracin que un cuerpo puede
adquirir es directamente proporcional a mg
la fuerza resultante e inversamente
a
proporcional a su masa.
F=20N

FR
a = FR = m a
m
N

adems: FR y a tienen la misma


Observemos que el bloque se desplaza
direccin.
horizontalmente y en esa direccin slo
hay una fuerza F = 20N, entonces ella
ser la fuerza resultante.
Dinmica Rectilnea
Luego:
Es aquella rama de la dinmica en la
cual el objeto de estudio son aquellos
F =ma
cuerpos que describen trayectorias
20 = 2a a = 10 m/s2
rectilneas.
Reemplazamos en (1):
Ejercicio 1:
Vf = 40 m/s
Sobre el bloque de 2 kg inicialmente en
reposo en la superficie lisa, se aplica
una fuerza horizontal constante cuyo
PROBLEMAS RESUELTOS
mdulo es 20 N; determine su rapidez
cuando han transcurrido 4 s.
1. Un bloque es lanzado con una
V=0 rapidez de 4 m/s en una superficie
F horizontal rugosa, detenindose
luego de 2 segundos. Determine el
coeficiente de rozamiento entre las
superficies en contacto.
(g = 10 m/s2)
Resolucin:
Solucin:
Para hallar la rapidez en t = 4 s, 2s
recordamos Cinemtica: a
4m/s
Vf = V0 + at mg V=0
f

Vf = a(4) ......... (1) A B


FN

Nos falta el valor de la aceleracin y


para calcularlo utilizamos la 2da Ley de Como la superficie es rugosa, sobre el
bloque acta una fuerza de rozamiento
Newton, para lo cual hacemos el D.C.L.
f tal que le va disminuyendo la
sobre el bloque:
O
FSICA
velocidad y por lo tanto le provoca una hasta que M llegue a impactar en
aceleracin negativa. el piso (M=m; g=10m/s2)

Luego: f = m.a. .........(1) a) 0,2 s m


Pero: f = . FN = 4 mg b) 0,5 s 0,4
c) 0,8 s 0,2
d) 1,0 s
En (1): mg = ma a = g ...... (2) e) 1,5 s M

Del M.R.U.V.: 2m

Vf = V0 a t Solucin:
A partir del instante que se liberan los
0 = 4 gt bloques, estos adquieren una
aceleracin.
= 4 = 1
mg
102 5
a
= 0,2 fc m

FN
2. Si el bloque de 60 kg apoyado
sobre la superficie horizontal
rugosa, se le aplica una fuerza
horizontal de 60 N, determine la a
m
aceleracin que adquiere. mg f c
(g = 10 m/s2) a=
2m
mg
a) 3 m/s2 0,7
b) 4 m/s2 0,5 mg c mg
c) 5 m/s2 a=
d) 6 m/s2
2m
e) 8 m/s2
a = 4 m / s2
Solucin:
Luego, analizamos al bloque M el cual
60N a parte del reposo y hasta llegar al piso
F=60N recorre 2 m se trata de un M.R.U.V.
6 Kg
fc
d = V0t + at2
FN M V0=0 2
Sabemos que:
FRES = m.a.
F - FC = m.a. a 2 = 4 t2
2m 2
F - C FN = m.a.
t
60 (0,5)(60) = 6 a

a = 5 m/s2 t = 1s

3. Si el sistema mecnico mostrado es Dinmica Circunferencial


liberado en la posicin mostrada,
determine el tiempo que transcurre

O
FSICA
Es aquella rama de la dinmica en la direccin, por lo tanto el satlite
cual el objeto de estudio son aquellos experimenta aceleracin, la cual debe
cuerpos que describen como trayectoria ser causada por una fuerza resultante
una circunferencia. no nula.

Para comprender esto consideremos el Al observar el D.C.L. notaremos que la


movimiento de un satlite alrededor de fuerza resultante es la fuerza
la tierra. gravitatoria, la cual en todo instante
V apunta al centro de la trayectoria que
describe el satlite (centro de la tierra).

Conclusin:
V
Para que un cuerpo describa un
movimiento circunferencial, ste debe
experimentar una fuerza resultante no
nula dirigida hacia el centro de la
V circunferencia a la que se denomina
FUERZA CENTRPETA (Fcp), la cual
causa una aceleracin dirigida hacia el
centro de la circunferencia denominada
ACELERACIN CENTRPETA (acp).
V
De la 2da Ley de Newton:
Haciendo el diagrama de fuerzas:
FR = m a Fcp = m acp

La aceleracin centrpeta mide el


Fg cambio en la direccin de la velocidad
tangencial en el tiempo.

Matemticamente:
Fg
V2
Fg a cp = = 2 r
r

Donde:
V : rapidez tangencial o lineal (m/s)
: rapidez angular (rad/s)
Fg r : radio de la circunferencia

Luego:
Podemos observar que el satlite mV 2
Fcp =
describe una trayectoria curvilnea r
alrededor de la tierra. Despreciando la
interaccin con los otros planetas,
podramos considerar a la trayectoria Fcp = m 2 r
como una circunferencia; como en la
direccin tangencial no hay fuerzas, la
velocidad se mantiene constante en
mdulo, pero continuamente cambia de

O
FSICA
Observacin:
En un movimiento circunferencial el PROBLEMAS RESUELTOS
segmento que une el centro de la
circunferencia con la partcula barre
ngulos a medida que transcurre el
tiempo; esto lo podemos caracterizar 1. Una esferita atada a una cuerda,
mediante una magnitud escalar suspendida en la forma indicada,
llamada: RAPIDEZ ANGULAR (). gira uniformemente en un plano
horizontal. Si la masa de la esferita
Matemticamente: es de 2 kg determine el mdulo de
la fuerza centrpeta.
(=37 ; g=10m/s2)

V a) 10 N
r b) 12 N

c) 14 N
d) 15 N
t L e) 20 N

Solucin:
VV
Hacemos D.C.L. a la esfera
rad
= Unidad: T Descomponemos
t s T Sen 37
la tensin en el
37 eje radial y eje
Tambin sabemos que a travs del tangencial
trayecto se cumple:

V2 T Sen 37
V = = r
t t

V = .r
20 N

Por lo tanto:
Luego, observamos que la fuerza
centrpeta (FCp) queda determinada por
V 2 (r )
2
a cp = = acp = 2 . r la componente:
r r
T sen 37

Es decir:

FCp = T sen 37 (1)

O
FSICA

Adems, en el eje tangencial: 3. Determine la mxima rapidez que


puede alcanzar un motociclista para
T sen 37 = 20 dar una vuelta completa en una
pista circular de 40 m de radio de
T 4 = 20 T = 25N curvatura. Considere S=0,25;
5 k=0,20. (g=10m/s2)

En (1): Solucin:
FCp = 25 3
5

FCp = 15N

m Mg
2. En la figura se muestra a un bloque = 40
r
de 5 kg que gira en un plano VMX
horizontal con una rapidez angular
constante de 2 rad/s, atada a una fs
cuerda de 2 m. Determine la M
X
tensin en la cuerda. FN
La velocidad ser mxima, en el
a) 20 N instante que est a punto de salir de la
b) 30 N trayectoria circular. En este caso la
c) 40 N fuerza que lo mantiene en su
d) 45 N trayectoria ser la fuerza de rozamiento
e) 50 N esttico mxima fsmx.

Luego:

Solucin: fsmx = FCp

Hacemos D.C.L. al bloque s FN = M V2MX


r

FN s Mg = M V2MX
r
T 2
VMX = s gr

r mg 2
VMX = (0,25)(10)(40)

2
VMX = 10 m / s
Eje radial: T = FCp
T = m 2 r
T = (5) (2)2 (2)

T = 40 N

O
FSICA
4. Si la masa m1 avanza con una
PROBLEMAS PARA aceleracin a. Halle la aceleracin
RESOLVER EN CLASE con que se mueve la masa m3

1 2
1. Sobre un cuerpo inicialmente en
reposo acta, durante 4 s, una
fuerza resultante de 1000 N y
recorre 400 m. Cul es el peso del 3
cuerpo?
(g=10m/s2) a) 2 a b) a c) a/2
d) a/3 e) 3a/2
a) 200 N b) 120 N c) 280 N
d) 160 N e) 100 N 5. Un ascensor de 280 N de peso
desciende en un pozo con
2. En el instante mostrado el sistema movimiento uniforme acelerado. En
parte del reposo. Despus de qu los primeros 10 s recorre 35 m.
tiempo el bloque A llegar a Hallar la tensin del cable del que
tocar el piso? (g=10m/s2); est suspendido el ascensor.
mA=3Kg; mB=2Kg.
a) 260 N b) 220 N c) 230 N
d) 300 N e) 280 N

6. De la parte superior de un plano


a) 2 s inclinado totalmente liso de
b) 3 s longitud 9,8m se deja caer un
cuerpo. Con qu velocidad llega al
c) 4 s B piso en m/s?
d) 5 s A a) 4,9 60
e) 6 s b) 9,8
16 m
c) 12,5
d) 14
e) 7
3. Si las superficies son totalmente
lisas. Determinar la fuerza de 7. Determinar la magnitud de la fuerza
reaccin entre las masas m2 y m3. F constante que se debe aplicar al
(4 m1 = 2 m2 = m3 = 4 Kg) sistema, para que los bloques A y B
de 1 Kg de masa cada uno no tengan
movimiento relativo respecto al carro
C de masa 8 Kg. No hay friccin y
100 N g=10m/s2
40 N m2 m3
m1
A
F C B
a) 35 N b) 45,7 N c) 57 N
d) 65,7 N e) 91,4 N

a) 40 N b) 60 N c) 80 N
d) 100 N e) 20 N

O
FSICA
8. Una cuerda cuelga de una polea y
en sus extremos hay dos masas A 11. Un tranva de masa m = 5
de 2 kg y B de 3 kg. Determinar toneladas, va por una curva de
la tensin en la cuerda (1), radio R = 125 m. Hallar la fuerza
sabiendo que la polea pesa 2 N y con la cual presionan lateralmente
no ofrece friccin. g=10m/s2. las ruedas sobre los rieles cuando
la velocidad del tranva es de 9
km/h.
(1)
a) 10 N
b) 20 N a) 300 N b) 250 N c) 125 N
c) 52 N d) 325 N e) 50 N
d) 48 N
e) 50 N 12. Una masa de 10 kg describe una
trayectoria circular de radio 1 m.
con una velocidad lineal de 10 m/s.
A B Hallar la fuerza en Newton, que la
mantiene en su trayectoria.
9. En la figura, las masas A y B
son de 40 g y 20 g a) 100 b) 1000 c) 500
respectivamente. Si la polea se d) 1500 e) 10
mueve hacia arriba de tal manera
que la masa de 40 g queda 13. Una masa M resbala sobre una
estacionaria sin hacer contacto con semiesfera lisa de radio R. A
el piso. Determinar la aceleracin partir del reposo; para un
de la polea. g=10m/s2. desplazamiento angular , su
velocidad es V, y la fuerza normal
F es N. Entonces:
2
a) 5 m/s
b) 4 m/s2
c) 3 m
d) 2 m/s2
e) 1 m/s2
B A

10. Calcular la medida del ngulo ,


sabiendo que todas las superficies
son lisas y que al resbalar W2 , W1 a) N = Mg b) N = Mg+MV2/2
no se mueve. (W2 = 2 W1) c) N > Mg cos f d) N < Mg cos f
e) N < Mg sen f
W2
14. Qu velocidad mnima ser
W1 necesario darle a un mvil en la
parte superior de su trayectoria, si
est atado a una cuerda al describir
una trayectoria circular vertical, en
m/s? Si: R=4,9m; g=10m/s2.

a) 4 b)5 c) 6 d) 7 e) 8
a) 45 b) 30 c) 15
d) 37 e) 53

O
FSICA

TRABAJO MECNICO

No es la intencin dar una definicin


rigurosa acerca del trabajo mecnico;
por el contrario queremos que se
comprenda las diferencias entre este
tipo de trabajo y anlogos en otros
campos de la vida.
Observe que en cada uno de los casos
Para comprender mejor empezaremos se ha superado una resistencia durante
por dar unos ejemplos: una distancia mediante la accin de una
fuerza; pudiendo de esto concluir:
(a) La esfera cae y aplasta al resorte
venciendo la resistencia interna de La transferencia de movimiento
ste. mecnico de un cuerpo a otro recibe el
nombre de Trabajo Mecnico

Esta transferencia de movimiento


mecnico la cuantificamos por medio de
una magnitud escalar denominada
Cantidad de Trabajo (W), la cual
matemticamente se evala de la
siguiente manera:

W FAB = F. d . Cos
(b) El gas se desplaza levantando el
mbolo superando la resistencia Para F constante
ofrecida por la carga hasta una
determinada distancia, originado Donde:
por la presin interna del gas. W FAB : trabajo desarrollado mediante
la fuerza F para llevar el
bloque desde A hasta B.
: ngulo formado por F y el
desplazamiento

Unidades:
F : Newton (N)
d : metros (m)
W : Nm = Joule (J)

(c) La fuerza de rozamiento esttico Grficamente podemos obtener el


fs evita el deslizamiento de los trabajo mecnico de una fuerza:
pes del atleta y a la vez lo impulsa
hacia adelante; es decir, le Para ello veamos el siguiente ejemplo:
transmite movimiento.

O
FSICA
El coche cambia de posicin debido a la
accin de la fuerza F
F (N)
y

F F F
A
x0 x
d
xm
x0 xf
xf
Luego:
A = W F
X0 Xf

F (N)

F PROBLEMAS RESUELTOS

A 1. Un bloque de 2 kg es elevado con


una fuerza F que produce una
xm aceleracin de 5 m/s2. Determine el
x0 xf trabajo de dicha fuerza, durante los
2 primeros segundos. (g=10m/s2)
A = W F
X0 Xf A = F.d B
Recordemos que:
WAF B = F . d ...... (1)
A : rea debajo de la grfica F vs X F
A : F(xf x0)
2s Observa que no
d conocemos el valor
De esto podemos darnos cuenta que el
de F y tampoco del
rea de esta grfica es numricamente
igual al trabajo que desarrolla la fuerza desplazamiento d
F. A

En general para el caso de una fuerza


Sin embargo, como existe aceleracin,
variable pero que es paralela a la
distancia que avanza el cuerpo: entonces usamos:

Ma=R 2 5 = F 20
F = 30N ...... (2)

Ahora, como el bloque estaba en


reposo (V0 = 0), entonces aplicamos
M.R.U.V. para hallar la distancia d.
d = V . t + at2
2

d = 5 22 d = 10m ...... (3)


2
O
FSICA
Luego, reemplazamos (2) y (3) en (1): Solucin:
WAFB = (30 N)(10m) El trabajo de la fuerza de gravedad no
depende de la trayectoria, slo depende
de la altura entre la posicin inicial y
WAFB = 300J final. Es decir:

2. Un bloque est apoyado sobre una


superficie horizontal rugosa en W AFg B = Fg . h AB
x=0. Si se aplica una fuerza
horizontal que vara en la forma WAFgB = (40 N)(4m)
indicada, determine el trabajo de la
fuerza de rozamiento, si el trabajo
neto hasta x=4m es de 50J. W Fg = 160J
Solucin:
F(N)
PROBLEMAS PARA
25 RESOLVER EN CLASE

15 1. Calcular el trabajo que realiz la


fuerza de 60 N en el tercer segundo
de su movimiento sobre el bloque
de 6 kg, si parti del reposo
4 x(m)
(g = 10 m/s2)
Se trata de una fuerza variable, en este
caso el trabajo de F est dado por el
rea de la grfica. Es decir: Liso
F=60 N
25 + 15
WXF=0X = 4 = A = 4
2
WF = 80J .......... (1)

Luego, por dato: a) 600 J b) 4500 J


WNETO = 50J c) 3000 J d) 1500 J
WF - Wfc = 50J e) 750 J
80J - Wfc = 50J
2. Un pequeo anillo es llevado desde
W fc
= 30J la posicin A hasta B a lo largo
del anillo liso. Calcular el trabajo de
3. Determine el trabajo de la fuerza la fuerza horizontal. F = 10 N
de gravedad sobre el bloque de 4
kg de A hacia B. (g=10m/s2) B
A a) 200 J F
b) 320 J
37
B c) 160 J
10m d) 640 J F
6m e) 120 J 0 R=25m
A

O
FSICA

3. Hallar el trabajo realizado por la 6. Un cuerpo de 5 kg resbala a


friccin, si el bloque de 10 N de velocidad constante sobre un plano
peso es llevado desde A hasta B horizontal donde uk = 0,3,
con velocidad constante (F = 20N) encuentre el trabajo realizado por
la fuerza de rozamiento para un
F desplazamiento de 10 m.

a) 0 J b) 147 J c) 294 J
d) 392 J e) 98 J
A B
5m
7. Un bloque de 10 kg es arrastrado
por la fuerza F = 80 N sobre una
a) 100 J b) 50 J superficie rugosa una distancia de
c) 100 J d) 200 J e) 20 J 10 m. Si el trabajo realizado por la
fuerza de rozamiento es de 240 J.
4. Calcular el trabajo neto sobre el Cul es el valor del ngulo ?
cuerpo. Para un desplazamiento de (g = 10 m/s2)
15 m. sobre la superficie rugosa
(g = 10 m/s2) F
50 N C=0,4
C=0,4
20 N 37
5 Kg.

a) 30 b) 37 c) 45
a) 300 J b) 120 J c) 480 J d) 53 e) 60
d) 180 J e) 120 J
8. Si la fuerza tangencial mantiene su
5. La grfica muestra la fuerza mdulo de 150 N, constante.
aplicada a un cuerpo y su Calcular el trabajo que realiza
correspondiente desplazamiento desde A hasta B (R = 2 m)
(x). Qu trabajo se ha realizado al
trasladar el cuerpo de x1 = 0,3m a F
x2 = 0,6 m?
A
120
F(N)
B
40 0
F
30

x(m)
a) 150 J b) 300 J
0 0,3 0,4 0,5
c) 200 J d) 600 J e) 3000/ J

a) 10 J b) 11,5 J c) 12 J
d) 14,5 J e) 16 J

O
FSICA
9. Un bloque de 8 kg es arrastrado 10 TRABAJO NETO
m aceleradamente a razn de 4
m/s2 mediante una fuerza Viene a ser la suma de los trabajos que
constante F sobre una superficie se han desarrollado por aquellas
horizontal rugosa. Calcular el fuerzas que estn aplicadas al cuerpo,
trabajo neto desarrollado sobre el para esto hay que tener en cuenta los
bloque (g = 10 m/s2) F signos de los trabajos + -.

C - El trabajo sobre un cuerpo ser


positivo cuando se le ponga en
movimiento.
- El trabajo ser negativo cuando
tratemos de detenerlo.
a) 80 J b) 160 J c) 240 J - El trabajo de una fuerza ser nulo
d) 320 J e) Falta conocer F si dicha fuerza es perpendicular a la
trayectoria o desplazamiento.
10. El trabajo desarrollado por la
persona A es WA y el realizado Ejemplo de aplicacin:
por B es WB. Halle el valor Determine el trabajo neto realizado
WA sobre el bloque para un desplazamiento
absoluto , si adems se sabe de 3m. F = 20N; f = 8N
WB
FG
que la persona B aplica una
fuerza igual al mdulo del peso del
bloque. F
f
V=Const
. N
A
B d

Solucin:
Observe que la fuerza de gravedad y la
fuerza normal (N) no desarrollan
trabajo por ser perpendiculares al
a) b) - 1 c) + 1 desplazamiento. Luego:
d) + 2 e) - 2
WN = W F + W f ......... (1)
11. En el grfico (F vs. X) mostrado
determinar el trabajo realizado Pero:
por la fuerza F desde x = 0 hasta
x = 16 m WF es positivo porque est a
F (N) favor del movimiento
Wf es negativo porque est en
contra del movimiento.
37
8 Luego:

WN = (20N 3m) - (8N 3m)


WN = 60J 24J
0 x(m)
WN = 36J
a) 288 J b) 224 J
c) 128 J d) 162 J e) 202 J

O
FSICA
POTENCIA MECNICA La eficiencia nos expresa la razn entre
lo til y lo suministrado a una mquina.
La potencia media es una magnitud
fsica escalar que nos indica la rapidez
con que en promedio se realiza un Potencia til P.u
determinado trabajo mecnico. = =
Potencia entregada P.e.

Potencia = Trabajo realizado


en porcentaje:
tiempo empleado

P.u
Pot = W = . 100 %
t P.e.

Unidades:
ENERGA MECNICA
W : Joule (J)
t : segundo (s) El trmino Energa est relacionado
con las diversas transformaciones que
Pot : Joule = watt (w) se dan en la naturaleza, por ello se
s plantea que en la naturaleza se
presentan diversas formas de energa.
POTENCIA INSTANTNEA
Es aquella que nos indica la rapidez con Nosotros nos centraremos
que se realiza trabajo en un intervalo principalmente a relacionar la energa
de tiempo muy corto. Su valor lo con la capacidad para transmitir
determinamos as: movimiento, es decir para desarrollar
trabajo. Para ello, debemos conocer
algunas de las formas en que se
Pot = F.v.cos presenta la energa.

Energa Cintica de Traslacin (EC)


: ngulo entre F y v
Es la medida escalar del movimiento de
EFICIENCIA O RENDIMIENTO traslacin de un cuerpo o partcula.
MECNICO Esta energa se puede obtener a travs
del trabajo que se efecta para mover
Denotada por ; es un nmero que un cuerpo.
va asociado en la estructura de una
mquina y que usualmente indica la V
calidad de la mquina. Su valor expresa
que fraccin de la potencia absorbida o 1
EC = m v2
entregada al cuerpo es transformada 2
en trabajo til.
m : masa del cuerpo
El trabajo til o potencia de salida de v : rapidez del cuerpo
una mquina nunca es igual a la de
entrada. Estas diferencias se deben en
parte a la friccin, al enfriamiento, al
desgaste, etc.

O
FSICA
Energa Potencial Gravitatoria (EPG) La suma de estas tres formas de
energa recibe el nombre de ENERGA
Es la medida escalar de la interaccin MECNICA (EM). Es decir:
gravitatoria de un cuerpo y la tierra.
Esta energa se almacena en el sistema
cuerpo tierra cuando desarrollamos EM = EC + EPG + EPE
trabajo para separarlos.
La Energa Potencial Gravitatoria
depende de la fuerza de gravedad del Importante:
cuerpo y de la altura medida a partir
del nivel de referencia (NR) en donde la La Energa Mecnica de un cuerpo o
Energa potencial es cero. sistema puede variar ya que por lo
general al analizar un fenmeno fsico
vemos que una forma de Energa se
m g transforma en otra.

h Ejemplo:
EPG = m.g.h.
Suponga que lanza un bloque sobre un
piso spero:

m: masa del cuerpo - En el punto A el bloque tiene


g: aceleracin de la gravedad EM; sin embargo la fuerza de
d: distancia vertical que existe entre el rozamiento cintico fc lo va
C.G. del cuerpo y e N.R. deteniendo hasta que en el punto
B su EM es cero.
Energa Potencial Elstica (EPE)
Es la energa que almacena un cuerpo Luego: La EM no se conserva!
elstico debido al trabajo que se
desarrolla para deformarlo (estirarlo o Conclusin:
comprimirlo). Para el caso particular de
un resorte ideal (de masa despreciable) La Energa mecnica de un cuerpo y/o
se calcula as: sistema se conserva (no cambia de
valor) siempre y cuando las fuerzas no
conservativas no efecten trabajo
mecnico.
Son fuerzas conservativas el peso y la
fuerza elstica.

En general:
FR
EM = - Wfnc
X
El cambio en la Energa Mecnica de un
cuerpo o sistema es numricamente
FD
igual al trabajo desarrollado en l por
las fuerzas que actan en l (sin
1 considerar a la fuerza de gravedad y
E PE = K . x2 elstica).
2

K : constante de rigidez del resorte


x : elongacin del resorte
O
FSICA

PROBLEMAS RESUELTOS 2. Una pequea esfera es lanzada tal


como se muestra. Determine el
mdulo de la componente
horizontal de la velocidad que
1. Tenemos una esfera a 250 m de tendr la esfera cuando pase por B.
altura. Calcular luego de cuntos Desprecie los efectos del aire.
segundos de haberse soltado, su (g=10m/s2)
energa cintica ser igual a su
energa potencial gravitatoria.
Desprecie los efectos del aire.
(g=10m/s2)
g 2,4m
Solucin: B
A
A V0=0
En todo el trayecto Solucin:
h1 t slo acta la fuerza Sabemos que en el punto ms alto de
de gravedad. Por lo la trayectoria, la velocidad es
B tanto, la energa horizontal. Adems, en dicha
250m
mecnica entre A y trayectoria la velocidad horizontal es
B se conserva. constante. Luego:
h
VH B = VD .......... (1)
D VD
Es decir:Ref.
E MA = E MB 8m/s VHB
2,4m B
E PA = E C B + E PB A
Ref.
Pero: E CB = E PB

E PA = 2 E PB E MA = E MD

MgH = 2(Mgh) h = H E CA = E C D + E PD
2
h = 125 m
M VA2 M VD2
Luego, nos damos cuenta que desde A = + Mg h
2 2
hasta B ha descendido tambin h1 =
125 m.
82 VD2
= + 10 ( 2, 4)
Luego, del M.V.C.L. 2 2
gt 2
h1 = V . t + VD = 4 m/s
2
125 = 10 t2
2 En (1):
VHD = 4 m/s
t = 5s

O
FSICA

El Estudio de las oscilaciones mecnicas y/o coseno. En la prctica todo


es importante no solamente por su movimiento armnico es a la vez
aplicacin frecuente a la ingeniera, sino peridico.
porque los resultados obtenidos durante
su estudio tambin pueden ser usados Observaciones:
Analicemos el movimiento de una
para el estudio y aclaracin de los
esferita sujeta mediante un hilo, como
fenmenos oscilatorios en otras ramas de se muestra:
la Fsica, tales como por ejemplo el
estudio de las oscilaciones armnicas que
experimentan los electrones en una
antena de transmisin o el movimiento de La esferita oscila
las molculas en torno a una posicin de en torno de su
equilibrio en una red cristalina o el posicin ms baja
movimiento de las molculas sobre la B
superficie libre de los lquidos luego de A C
una perturbacin. B

Por lo expuesto, el M.A.S. es de suma


1ra: La esfera completa una oscilacin
importancia ya que permite
cuando desarrolla un movimiento
comprender algunos de los
completo, es decir, cuando va del
movimientos oscilatorios ms
extremo A hacia el extremo C y
complejos que se presentan en la
luego retorna al extremo inicial, A.
naturaleza. Antes de entrar a analizar y
describir el M.A.S. conoceremos
A B : Un cuarto de oscilacin
algunos aspectos previos como lo que
es: un movimiento oscilatorio y un A C : Media oscilacin
movimiento peridico.
A C A : Una oscilacin
Movimiento Oscilatorio
Se caracteriza porque el movimiento se
repite, siguiendo la misma trayectoria 2da.: El tiempo que debe transcurrir
en ida y vuelta. Se experimenta un para que se repita nuevamente el
movimiento de vaivn. evento se denomina: Perodo (T).
Por ejemplo, un reloj de pndulo, un
columpio, etc. 3ra.: Un movimiento peridico, no es
necesariamente oscilatorio y un
Movimiento Peridico movimiento oscilatorio no es
Es aquel que se repite regularmente en necesariamente peridico.
intervalos de tiempo iguales.
Por ejemplo, el movimiento rotacional Fuerza Elstica
de la tierra, sus clases en el centro pre, Estas fuerzas se generan cuando se
etc. deforma un cuerpo. Por lo general se
distinguen:
Movimiento Armnico
Es aquel movimiento cuya posicin a) Fuerza Deformadora (FD):
est expresada en trminos de seno
O
FSICA
Es aquella fuerza que produce la
deformacin del cuerpo, siempre
tiene el sentido de la deformacin. Lo alejamos una distancia (A) de su
(X = Lf L0) posicin de equilibrio (P.E), por medio
de una fuerza deformadora (FD).

V=0
FD

Lo
x

FR FD
A

Qu movimiento desarrolla el bloque


Lf al dejar de aplicar la FD?

b) Fuerza Recuperadora (FR):


Se genera en los cuerpos Mov. de ida (T/2)
deformados. Si la deformacin no
supera el lmite elstico, se cumple -A +A
la Ley de Hooke.

FD (D.P.) X V=0 V
FR
FD
K = = cons tan te
X N x M
P.E.
K : constante elstica del resorte
Mov. de vuelta (T/2)
Luego, la fuerza recuperadora est
dada por:
El movimiento se repite cada T
segundos.
FR = -KX
El bloque adquiere movimiento
Qu es un Movimiento Armnico mecnico, debido a la accin de la
Simple? fuerza recuperadora (FR = kx, la cual
disminuye a medida que el bloque se
Es un movimiento oscilatorio, peridico acerca a la P.E.).
en lnea recta.
Por ejemplo, analicemos un bloque en Elementos del M.A.S.
reposo ligado a un resorte:
Posicin de 1. X ; posicin de la partcula
equilibrio respecto de la posicin de equilibrio
P.E. llamada tambin elongacin

liso 2. Amplitud (A): Mxima posicin o


elongacin.

O
FSICA
3. Perodo (T): Es el tiempo utilizado
para dar una vibracin u oscilacin
completa.

4. Frecuencia (f): Es el nmero de


vibraciones completas por unidad
de tiempo.

1
f = Unidad:
T
S-1 = Hertz (Hz)

5. Frecuencia cclica ():

2
= = 2f De t0 = 0 a tf= t, la partcula barre un
T ngulo , y del M.C.U. se tiene que:

=.t
Por qu al M.A.S. se le denomina
armnico?
Ecuacin de la posicin:
Se debe a que su movimiento est
gobernado por funciones armnicas
(seno o coseno). A partir del se deduce que:
ECUACIONES DEL M.A.S.
Para obtener las ecuaciones del M.A.S.
trabajaremos con la proyeccin X = A sen ( t + )
horizontal de una partcula que
experimenta un M.C.U., con el
movimiento del bloque. : Fase Inicial; su valor depende de las
condiciones iniciales (posicin y
velocidad inicial)

Se expresa en rad

Ejemplo:
Sea la ecuacin del movimiento de un
oscilador armnico:

X = 0,2 Sen (t + ) m
4

Determinar su amplitud, la frecuencia


cclica, fase inicial, perodo, frecuencia
de oscilacin y su posicin para el

= instante t = 0,25 s

x
O
t
P.E.

t=t
FSICA
Solucin:

Sabemos que la ecuacin de X(t = 0,25) = 0,2 m


movimiento del M.A.S. es:

X = A sen ( t + ) Es decir, en t = 0,25 s el oscilador se


encuentra 0,2 m a la derecha de la P.E.
Luego, por dato:
Ecuacin de la Velocidad
X = 0,2 sen (t + )
4
V(t) = A Cos (t + )
Comparando ambas ecuaciones
tenemos que:
Esta ecuacin nos permite hallar la
velocidad del mvil en cualquier
* A = 0,2 m = 20 cm Amplitud instante de tiempo.

Tambin:
* = rad/s Frecuencia cclica
V = A2 X2
* = rad Fase inicial
4
Esta ecuacin slo nos permite conocer
* T = 2 = 2 el mdulo de la velocidad conociendo la
posicin del mvil.

De esto se deduce:
T=2s En cada oscilacin
el oscilador emplea VMX = A .......... (en la P.E.)
2s VMN = 0 .......... (en los extremos)

* f= 1 = 1
T 2 Ecuacin de la Aceleracin

En cada segundo a(t) = -2 A Sen (t + )


f = 0,5 s el oscilador desa-
rrolla media
oscilacin Para cualquier instante de tiempo.

* Ahora, en t = 0,25 s su posicin De esto se deduce que:


ser:

X = 0,2 sen ( (0,25) + )m


a(t) = -2 x
4

X = 0,2 sen El signo (-) indica que a y x son de


2 direccin contrarias.
Luego:
1

O
FSICA

a(t) = 2 x Se sabe que:

X = A sen (t + ) . (1)
El dato dice que en cada oscilacin el
* aMX = 2 A .... (en los extremos) bloque recorre 100 cm, pero tambin
podemos deducir que en cada oscilacin
* aMN = 0 .... (en la P.E.) el mvil recorre cuatro veces la
amplitud (A).

El perodo de oscilacin, depende Es decir:


de la amplitud? 100 = 4 A

NO!, depende de la masa y de la A = 25 cm = 0,25 m


rigidez del resorte. El perodo (T) se
evala as: Adems:

k 100
= =
m m 4
T = 2
k
= 5 rad/s
Recuerde que:
Para hallar la fase inicial, evaluamos la
2 ecuacin (1) para t = 0
= = 2 f
T -A = A Sen ( (0) + )

-1 = Sen =
Ejemplo: 2

El bloque de 4 kg que se muestra est


en reposo. De pronto se le desplaza X = 0,25 sen (5 t + )
hacia la izquierda y luego se suelta. 2
Determine la ecuacin de su
movimiento, si en cada oscilacin el
bloque recorre 100 cm. (k = 100 N/cm)
En el M.A.S. La energa mecnica
se conserva?
S! Porque la fuerza que mantiene el
M.A.S. es una fuerza conservativa
(fuerza elstica). La energa mecnica
del sistema masa-resorte de un M.A.S.
P.E. se evala as:
K 2
liso kx 2 mV 2 kA 2 m VMX
EM = + = =
2 2 2 2

en cualquier en un en la
posicin extremo P.E.

Solucin:
O
FSICA
PNDULO SIMPLE
X = 0,4 Sen t +
2 3
Consiste de una masa de dimensiones
muy pequeas, suspendida mediante
Determine el perodo de oscilacin,
un hilo inextensible y de peso
posicin y velocidad inicial.
despreciable de un punto fijo. Al ngulo
que forma el hilo con la vertical en la
Rpta.: ______________
posicin extrema se le denomina
amplitud de la oscilacin.
2. Un oscilador armnico de amplitud
40 cm, es observado inicialmente
en X0 = -20 cm. Si realiza 60
oscilaciones por minuto. Determine
el ngulo de fase inicial; la ecuacin

g del movimiento y la velocidad


L L inicial.

Rpta.: ______________

3. Un oscilador realiza un M.A.S. cuya


ecuacin de movimiento est dado
m
por y = A Sen t m, en forma
6 6
Para el perodo del pndulo simple se vertical.
cumplen las siguientes leyes: En qu instante el oscilador est
A 3
1. Es independiente de la masa. en y = + descendiendo?
2
2. Es independiente de la amplitud, si
esta es pequea ( 5) Rpta.: ______________

3. Es directamente proporcional a la 4. Una partcula que desarrolla un


raz cuadrada de su longitud. M.A.S. tiene una velocidad de 5
cm/s y aceleracin de 10 cm/s2
4. Es inversamente proporcional a la cuando se encuentra en X = 2 cm.
raz cuadrada de la aceleracin de Determine su amplitud.
la gravedad.
Rpta.: ______________

5. Un cuerpo es impulsado desde la


L 1
T = 2 = posicin de equilibrio con una
g f velocidad de 0,4 m/s. Si su
amplitud es 0,08 m. Calcular su

velocidad despus de seg. de
3
PROBLEMAS haber partido.

1. La ecuacin del movimiento de una


partcula con M.A.S. es:
P.E
.

O
FSICA

a) X = 2 Sen 2t +
3
3
Rpta.: ______________ b) X = 7 Sen t
2
3
6. El bloque M = 100 g de la figura c) X = 7 Sen 2t
oscila sin friccin con una amplitud
2
de 3 cm. En el instante que pasa 3
d) X = 7 Sen 2t +
por su posicin de equilibrio, cae 2
verticalmente sobre l una masa
m de 44 g, la cual queda e) X = 2 Sen 2t
adherida. Determine la nueva 3
amplitud de oscilacin.

2. El oscilador armnico, oscila a lo


m largo del eje X. Si la posicin de tal
oscilador vara segn muestra la
K grfica. Qu ecuacin gobierna
M dicho movimiento?

5
a) X = 2 Sen t+
4 4
Rpta.: ______________
5
b) X = 3 Sen t+
4 4
7. Un reloj pndulo es llevado a un 5
c) X = 4 Sen t+
planeta en donde la aceleracin de 4 4
la gravedad es un 10% menor que
en la Tierra. Si la longitud del
5
pndulo es de 20 cm. Cul debe d) X = 5 Sen t+
ser la nueva longitud del pndulo 4 4
para que en ese planeta funcione
correctamente? 5 5
e) X = 4 Sen t+
4 6
Rpta.: ______________
3. El anillo de 0,8 kg se sostiene sobre
una mesa lisa y se sujeta a dos
resortes de constantes K1=30N/m y
ADICIONALES
K2=50N/m. Se empuja el anillo a lo
largo de la lnea que une a los
extremos fijos A y B, y despus se
1. Determine la ecuacin del suelta. Calcular el perodo de
movimiento de un oscilador oscilacin del sistema.
armnico que realiza 120
oscilaciones en 2 minutos. La
amplitud del movimiento es de 7
cm, e inicia su movimiento en el K1 K2
extremo izquierdo.

B
O
A
FSICA


a) s b) s c) 2 s
2


d) s e) s
5 3

O
FSICA

CANTIDAD DE MOVIMIENTO (P) Si se desea obtener la cantidad de


Llamado tambin momentum lineal, es
movimiento de un sistema de partculas
una magnitud que sirve de medida
(PSIST) se suma la cantidad de
vectorial del movimiento mecnico.
movimiento de todos los cuerpos.
Todo cuerpo que tiene velocidad se dice
que es portador de cierta cantidad de
Por ejemplo:
movimiento igual al producto de su
masa y su velocidad.
10m/s
m 5m/s
V 4m/s m
53
M
P (1) (2) (3)

PSIST = P1 + P2 + P3 . (1)
Matemticamente: P=M V
P1 = 2(+4) = +8 Kg m = 8 i Kg m
Unidad: Kg m
S s s

El vector cantidad de movimiento (P) P2 = 5(+5) = +25 Kg m = 25 J Kg m


presenta igual direccin que la s s
velocidad (V). Es decir:
P3 = 2 (Vx + Vy)
P V
Ejemplo:
Hallar la cantidad de movimiento de P3 = 2(6 i + 8 J) = (12 i + 16 J)Kg m
cada una de las esferas. M=2Kg; S
M=5Kg
En (1) :
5m/s 4m/s
PSIST = 8 i + 25 J + 12 i + 16 J
m1 m2
X PSIST = (20 i + 41 J) Kg m
s

P1 = m1 V1 = 2(+5) = + 10 Kg. m En general:


S
P2 = m2 V2 = 5(-4) = -20 Kg. m n

S
PSIST = Pi
i =1
* El signo (+) o (-) indica la direccin

O
FSICA
IMPULSO (I) Para un sistema de partculas:

Magnitud vectorial que caracteriza la


accin de una fuerza en un intervalo de FR V2 V2 V
1
tiempo. En forma ms general, el V1
impulso es una magnitud que mide la V3
V3
transferencia de movimiento entre los
cuerpos.

Matemticamente: IR = PSIST = Pf - Pi

* si la fuerza F es constante.
Si: IR = 0

t
V=0 V Pf = Pi La cantidad de
movimiento se
F F conserva

CHOQUES

Se llama choque o colisin a aquellas


interacciones entre cuerpos cuyo
I = F . t Unidad: N.s. tiempo de duracin es pequeo,
exceptundose en este caso las
explosiones.

Si F vara en mdulo, entonces el


rea debajo de la grfica F - t nos V1 V2
dar el impulso.

F2 rea = I

F1 V1 V2

t1 t2 t

Relacin entre el impulso (I) y la


cantidad de movimiento (P) Durante el choque, los
cuerpos se deforman
I = P

Toda fuerza que causa un impulso


sobre un cuerpo origina en l un
cambio en su cantidad de movimiento.

O
FSICA
Clasificacin de los choques Caso 1: Cuando un cuerpo choca con
A. Choque frontal.- Cuando la lnea una pared:
de movimiento de los cuerpos,
antes y despus del choque, es la Vi
misma.

Vf

B. Choque oblicuo.- Cuando la lnea e = Vf


de movimiento de los cuerpos, vi V f = e Vi
antes y despus del choque son
diferentes.
Caso 2: Cuando dos esferas chocan
(1) frontalmente:
V1 V2

(1)

(2)

(2)

Coeficiente de restitucin
Experimentalmente se percibe que las u1 u2
caractersticas del movimiento despus
del choque depende de las propiedades
elsticas de los cuerpos en interaccin,
de las fuerzas en la deformacin y
recuperacin, etc.; por ello para e = Velocidad relativa despus del choque
caracterizar los diferentes choques Velocidad relativa antes del choque
usamos una cantidad adimensional
llamada Coeficiente de Restitucin
e = VREL. D. CH.
(e).
VREL. A. CH.

OBSERVACIONES:
0e1
1. Si: e = 1; CHOQUE ELSTICO.
No hay deformacin
I recuperador permanente, los cuerpos
e =
I deformador recuperan su forma.
E M A .CH . = E M D.CH .

O
FSICA

2. Si: 0<e<1; CHOQUE INELSTICO. 2. Una esfera de 0,5 kg se lanza con


m
Los cuerpos quedan con cierta 30 J . Determine el impulso de la
s
deformacin permanente fuerza de gravedad sobre la esfera
hasta el instante que desciende con
E M i = E M f + Q LIBERADO 20 m/s. Desprecie la resistencia del
aire. (g=10m/s2)
3. Si: e = 0; CHOQUE PLSTICO.
a) +15 N.S. b) 15 c) +20
Los cuerpos quedan d) 25 e) +25
completamente deformados, no
se produce el rebote, por lo tanto 3. Sobre un bloque en reposo,
despus del choque quedan en apoyado sobre una superficie
reposo o se mueven con igual horizontal se ejerce una fuerza F =
velocidad (juntos) 5 t i donde F est en Newton y t en
segundos. Determine el impulso de
la fuerza sobre t = 2 s hasta t = 10
V1 V2 V s.

a) +20 N.S. b) 240 c) 200


d) -200 e) +140
4. Dos esferas A y B con velocidades
m m
respectivas de 4 y 3
s s
E M i = E M f + Q LIBERADO corresponden a masa de 2 kg y 1
kg. Si estas chocan opuestamente y
en forma frontal; calcule las
PRCTICA velocidades de estas esferas luego
de la colisin inelstica (e = 0,5)

m m
1. Una pelota de jebe de 500 g rebota a) 0,5 y4 b) 1 y 3
s s
en una superficie horizontal tal c) 0,5 y 2 d) 2 y 5
como se muestra. Determine la e) 0,8 y 1,7
rapidez de rebote y el mdulo del
cambio de la cantidad de 5. Una pelota se suelta desde una
movimiento sabiendo que ste es altura de 19,6 m sobre el piso, al
mnimo. impactar rebota hasta alcanzar una
altura mxima de 4,9 m. Calcule el
m m coeficiente de restitucin elstica
a) 14 ; 24 kg
s s entre la pelota y el piso.
b) 14; 20
c) 18; 26 a) 1 b) 0,8 c) 0,6
d) 16; 26 50 m/s d) 0,5 e) 0,2
e) 16; 18
60 6. Un minsculo palillo de longitud L
reposa sobre una mesa lisa, una
14 hormiga, cuya masa es la novena
parte que la del palillo, camina
sobre el palillo desde uno de los

O
FSICA
extremos con una rapidez V con
respecto al palillo. Qu distancia 10. Con una velocidad v e inclinacin
retrocede el palillo hasta el instante una pelota se lanza sobre una
en que la hormiga llega al otro superficie horizontal lisa cuyo
extremo? coeficiente de restitucin es e.
L L Hallar el tiempo adicional en el que
a) L b) c) se puede considerar que la pelota
2 4
deja de rebotar.
L L
d) e)
8 10 Rpta.: t = 2V sen
g(1-e)
7. Un hombre y un muchacho que
pesan 800 N y 400 N 11. En forma frontal una esfera de
respectivamente; estn sobre un masa m con velocidad V choca
piso sin rozamiento. Si despus de con otra idntica, en reposo, sobre
que se empujan uno al otro, el una mesa lisa, siendo e el
hombre se aleja con una velocidad coeficiente de restitucin elstica,
de 0,5 m/s respecto al piso. Qu halle la prdida de energa
distancia los separa luego de 5 mecnica una vez efectuado el
segundos? choque.
Rpta.: P.E. = mV2 (1-e2)
a) 7,5 m b) 96 c) 6 d) 8 e) 10,5 4
8. Al explotar una granada en tres ONDAS MECNICAS
fragmentos iguales resulta que los
fragmentos planarmente con Qu es una onda?
m Son oscilaciones que se propagan en el
velocidades respectivas de 5 i ; espacio y tiempo, desde un lugar del
s
espacio que ha sido perturbado,
m
12 J y V. Encuentre V. conocido como foco.
s
Para la propagacin de una onda
m mecnica es necesaria la existencia de
a) 13 b) 8 c) 7
s un medio?
d) 10 e) 12
Rpta.: S!
9. Una bola de billar choca contra la
banda lisa de la mesa de juego, as Sabemos que las partculas de todo
como detalla el diagrama. Si e es cuerpo sea slido, lquido o gaseoso
el coeficiente de restitucin interactan unos con otros. Por eso si
elstica. Halle el ngulo de una partcula del medio empieza a
rebote. oscilar debido a la interaccin este
movimiento oscilatorio comienza a
propagarse con cierta rapidez en todas
las direcciones.

Una onda no transporta masa, slo


transporta energa y cantidad de
movimiento, las cuales son propiedades
fundamentales de toda onda sea cual
sea su naturaleza.

Rpta.: Debido al movimiento oscilatorio de las


= arc tg (e.tg ) partculas las ondas se clasifican en:
O
FSICA
1
a) Ondas transversales.- Son Donde: f=
T
aquellas en las que las partculas
oscilan perpendicularmente a la La posicin y(x,t) de una partcula
direccin de propagacin. En el situada a x metros del origen de
deslizamiento de unas capas de ondas, en el instante de tiempo t es:
otras en los gases y lquidos no
hace que aparezcan fuerzas de
t x
elasticidad por esta razn en los y ( x , t ) = A Sen 2
gases y en los lquidos no pueden T
propagarse ondas transversales.
Ecuacin de una onda armnica
b) Onda longitudinal.- Son aquellas Donde:
en la que las partculas oscilan (-): Si la onda se propaga a la derecha
paralelamente a la direccin de (+): Si la onda se propaga hacia la
propagacin. En la onda izquierda
longitudinal tiene lugar la La frecuencia de la fuente de las
deformacin por compresin. Las oscilaciones es la misma frecuencia de
fuerzas de elasticidad ligada a esta oscilacin de una partcula del medio y
deformacin se originan tanto en es la misma frecuencia que el de la
los slidos como en los lquidos y onda.
en los gases por eso las ondas Las ondas experimentan fenmenos
longitudinales se pueden propagar como: reflexin, refraccin, difraccin,
en todos los medios. interferencia y polarizacin.

Elementos de una onda: Qu sucede cuando una onda se


Sea una onda armnica: encuentra con la frontera de otro
medio?
Cuando un movimiento ondulatorio llega a
t = 0 t = t una superficie o regin donde cambian las
propiedades del medio en el cual se
propaga, sufre una alteracin y como
resultado, parte de la energa del
A y
movimiento ondulatorio es devuelta al
z mismo medio de donde proceda,
x constituyendo la onda reflejada, y la otra
parte es transmitida al otro medio
constituyendo la onda refractada. El grado
e = v: t de reflexin y transmisin depende de la
elasticidad del segundo medio.
y: Es la posicin de la partcula del
medio oscilante ubicada a x metros FUENTE DE FUENTE DE ONDA
ONDA INCIDENTE REFLEJADO
del origen de onda.
A: Amplitud (ymx) Vi
: Longitud de onda Vr
f: Frecuencia en Hertz (Hz)
MEDIO (1)
MEDIO (2)
Rapidez de propagacin V
R
VR
e
V = = = f FUENTE DE ONDA
t T REFRACTADO

O
FSICA

En donde el rayo incidente, el rayo Si la rapidez en el segundo medio es


reflejado y el rayo refractado estn en menor, entonces la longitud de onda en
un mismo plano. el segundo medio ser tambin menor.

En donde el ngulo de incidencia ( i ) y


el ngulo de reflexin ( r ) son iguales:
1 2
i = r (1>2)

Las rapideces de las ondas son


diferentes en los medios (1) y (2):
V1 V2

Sen i Vmedio La frecuencia de una onda no se altera


=
incidente
cuando se transmite de un medio a
Sen R Vmedio refractado
otro.

Las partculas del medio 2 empiezan a ONDAS ESTACIONARIAS


oscilar debido a que son perturbados
por las partculas de la interfase Es un tipo especial de la interferencia
correspondientes al medio 1, las que se de ondas que resultan de la
comportan como si fueran la fuente de superposicin de 2 movimientos
las oscilaciones y como la frecuencia de ondulatorios producidos por dos focos
la fuente de oscilaciones es la misma que vibran sincrnicamente (con la
que la frecuencia de la onda generada misma frecuencia) y por consiguiente
podemos concluir que: tienen la misma longitud de onda.
Estas interferencias se caracterizan
fmedio(1) = fmedio(2) porque existen puntos llamados nodos
donde la interferencia es siempre con
Concluimos que cuando una onda pasa anulacin mientras que en otros puntos
de un medio a otro su frecuencia llamados vientres la interferencia es
permanece constante. siempre con refuerzo.

Qu ocurrir con su longitud de Los nodos y los vientres ocupan


onda? posiciones fijas, de modo que esta onda
parece no avanzar en el espacio de ah
fmedio(1) = fmedio(2) el nombre de onda estacionaria.
/2 /4
Vmedio(1) = Vmedio(2)
1 2

Es decir la rapidez de la onda es


proporcional a su longitud de onda. N N N

V V V

N: NODO V: VIENTRE

Una caracterstica interesante es que la


distancia entre dos nodos consecutivos
o dos vientres consecutivos es de

O
FSICA
media longitud de onda (/2), mientras
que la distancia entre un nodo y un Donde t es una tensin de la cuenta (N)
vientre es de un cuarto de longitud de y es la densidad lineal de la cuerda.
onda (/4). Reemplazado en obtenemos la
Esto se puede apreciar en la siguiente frecuencia de una onda estacionaria.
ilustracin.
/2 = L
n T
f = ..... ()
2L

Para n = 1 obtendremos
/2 = L/2

1 T
f1 =
2L

/ 2 = L/3 A la cual se le denomina frecuencia


fundamental de la cuerda.

La expresin () es importante porque


en ella se puede ver cuales son los
factores que influyen en la frecuencia
de las ondas estacionarias en una
cuerda vibrante.
En los grficos anteriores se observa
que la longitud de onda estacionaria, Como las cuerdas vibrantes se utilizan
toma valores definidos. en numerosos instrumentos musicales
(piano, guitarra, violn, etc.), el sonido
L L L L
= L , , , , ....... , emitido por una cuerda de esos
2 2 3 4 n instrumentos se controla ajustando la
2L 2L 2L longitud, la tensin o la masa de la
=2L, , , ....... ,
2 3 n cuerda.
Donde n es un nmero entero
V n
Como f = f = V ..... ( )
2L
Es decir:
V V V
f= , 2 , 3 , ..... etc
2L 2L 2L

La rapidez con la cual se propaga una


onda a travs de una cuerda est dada
por:

T
V =

O
FSICA

A QU SE LLAMA FLUIDO? Nota:


Es toda sustancia (lquidos, gases) que La densidad de una sustancia expresada
adopta fcilmente la forma del recipiente en g/c.c., queda expresada en kg/m3 si
que lo contiene, y una de sus se multiplica por 1000.
propiedades ms importantes es la de Ejemplo:
ejercer y transmitir Presin en todas las
direcciones.
* H2O = 1 g/cm3
DENSIDAD () Luego:
Esta magnitud nos indica la cantidad de H2O = (1 x 1000) kg/m3=1000 kg/m3
masa que se halla contenida en la unidad
de volumen de un determinado material. * ACEITE = 0,8 g/cm3 = 800 kg/m3

m
= QU ES LA PRESIN?
v Consideremos dos bloques de concreto
idnticos de 4 kg cada uno, apoyados
sobre nieve tal como se muestra.
Unidades:
g/cm3 ; kg/m3 B

PESO ESPECFICO ()
Esta magnitud mide el peso que posee
A
cada unidad de volumen de un material
determinado.

w
=
V
Unidades:
N/m3
Qu notamos?
Relacin entre y Que el bloque B se hunde ms que el
w m.g m bloque A, pero, Porqu, si en ambos
= = = .g
v v v casos los bloques ejercen la misma fuerza
sobre la superficie?
=.g

O
FSICA
Fg = 40N
Por ejemplo, un lquido puede ejercer
presin sobre las paredes del recipiente
que lo contiene.
Fg = 40N

mg

FN=40N FN=40N
10N 10N
20N 20N
10N
h
PH

Notamos que en el caso B la fuerza de


40N se distribuye sobre una menor
superficie que en el caso del bloque A,
por ello cada unidad de rea de la base F
Sabemos que: P =
en B soporta mayor fuerza, por eso A
experimenta mayor hundimiento.
Luego:
Luego, la presin es una magnitud fsica m g ( V )g
que mide la distribucin de una fuerza PH = =
A A
perpendicular (normal) sobre una
superficie de rea A. A h g
PH =
A
Matemticamente:

FN
P= pH = g h
A

Donde:
Unidad en el S.I.
: Densidad del lquido
g : aceleracin de la gravedad
N
= Pascal ( Pa ) h : profundidad
m2
PRESIN TOTAL (PT)
* 105 Pa = 1 bar Es la suma de las presiones locales
(manomtricas, hidrostticas, etc) y la
EJERCERN PRESIN LOS presin atmosfrica.
LQUIDOS?
Como todo cuerpo sobre la Tierra, los Ejemplo:
lquidos tambin se encuentran sujetos a Halle la presin total en el fondo del
la fuerza de gravedad, por lo tanto, cilindro que contiene agua.
pueden ejercer presin: PRESIN
HIDROSTTICA (PH).
O
FSICA

PROBLEMAS RESUELTOS

1. Se tiene una piscina rectangular de


dimensiones 5m y 10m y contiene
1m agua hasta una profundidad de
2m. Determine la presin
hidrosttica, la fuerza hidrosttica
y la fuerza total en el fondo de
dicha piscina.

Solucin Solucin:
En este caso como el lquido est
expuesto a la atmsfera, debe mos a) Hallamos la PH:
agregarse la presin atmosfrica (Patm). PH = H2O g H

PT = PH + Patm
kg m
PT = gH + Patm PH = 1000 3 10 2 (2m )
kg m N m s
PT = 1000 3 x 10 2 x 1m + 105 2
m s m
N
N N PH = 20000
PT = 10 4 2 + 105 2 m2
m m PH = 2104 Pa
pT = 1,1 x 105 Pa
b) Hallamos la fuerza hidrosttica (FH)
Observaciones: FH = PH A
N
1. La presin hidrosttica depende FH = 2 10 4 2 ( 5m )( 10m )
solamente de la profundidad ms m
no de la forma del recipiente que FH = 106 N
contiene al lquido.
2. Todos los puntos en un mismo c) Hallamos la fuerza total (FT)
lquido ubicados a una misma FT = (PH + Patm) A
N
profundidad soportan igual presin
y la lnea que une dichos puntos se
N
FT = 2 10 4 2 + 10 5 2
m m
( 50m )
2

llama ISOBARA. 6
FT = 6 10 N

Reflexiona
Es lo mismo calcular la fuerza
ISBARA hidrosttica sobre la base del
recipiente que sobre la pared
A B PA = PB vertical?
PA < PC

O
FSICA
PRINCIPIO DE PASCAL Una aplicacin prctica de este principio
es la Prensa Hidrulica.
Qu establece el principio de
Pascal?
Todo fluido transmite sin alteracin la
F1
presin ejercida sobre l a todas las A1
partculas del mismo y en todas
direcciones. A2

Po
Por ejemplo:
A F2 Po

Po Po Po
Po Po
Po

P2

P1 Esta mquina basa su funcionamiento en


el Principio de Pascal. Al aplicar una
fuerza sobre uno de los pistones, sta se
transmitir al otro en mayor valor.

En la grfica, cuando, sobre el pistn de


rea A1 se ejerce una fuerza F1, el
P3
lquido transmite una presin adicional:
Si ejercemos sobre el mbolo una fuerza
externa: F1
Po = ..........(1)
Fext A1
Luego, sobre el pistn de rea A2 el
lquido le ejerce una fuerza adicional F2
de modo que:
P2 + P

P1 + P F2 = (Po) (A2) ........ (2)

Reemplazamos (1) en (2):

F A
P3 + P F2 = 1 A 2 F2 = F1 2
A1 A1

Sabemos que: Observacin


F
P=
A Como A2 > A1; entonces F2 > F1; esto
significa que la prensa hidrulica
Luego, notamos que la presin ejercida multiplica la fuerza.
(P), se transmiti en todas las
direcciones.
O
FSICA
Las maquinas hidrulicas como los frenos F
+ Patm = PH + Patm
hidrulicos, gatos hidrulicos, ascensores A
hidrulicos, etc. Estn basados en el De donde:
principio de pascal
A2 F
= H 2 O . g . H
A ; se llama: Ventaja Mecnica. A
1

Problema de Aplicacin: Luego:


La base del mbolo de una bomba F = A . H2O gH
impelente es un crculo de dimetro
D 2
Dcm. Qu fuerza en Newton es preciso
F= 4 ( 10 )
3
( 10 ) H
ejercer sobre dicho mbolo para elevar el 4 10
agua a una altura de H metros (g = 10
m/s)?
D2 H
F=
Solucin 4
Po
PRINCIPIO DE ARQUMEDES

F H Qu establece el Principio de
Po A Arqumedes?
Todo cuerpo sumergido parcial o
x y totalmente en un fluido, experimenta la
accin de una fuerza perpendicular a la
superficie libre del lquido y hacia arriba,
denominada: Fuerza de Empuje
H2O Hidrosttico (E).

La fuerza de empuje acta en el centro


de gravedad de la parte sumergida.
La presin ejercida en x se
debe la fuerza F que Supongamos un cilindro homogneo
buscamos. sumergido en un lquido de densidad L
Como el dimetro es D cm; tal como se muestra:
D
en metros ser:
100
Luego:
2 F1
D D2 2 h1
A= = m
4 100 4 10 4

Ahora uniendo x e y obtenemos una


h2
Isbara, es decir: F3 F4

Px = Py

O
FSICA

E = mliq. desalojado . g
Como ya sabemos, un lquido presiona
sobre el fondo y contra las paredes del
recipiente, y si en l introducimos un
cuerpo cualesquiera, ste tambin estar INDICA EL
DINAMMETRO VALOR DE LA
sometido a dicha presin. TENSION

En consecuencia, observamos que el T


lquido ejerce presin sobre las paredes
del cilindro causando las fuerzas que se E + T = mg
muestra, de tal forma que:
E
mg E = mg - T
Horizontalmente:

F3 = F4 FRx = O
T : Peso aparente del cuerpo

Verticalmente:
Observacin
Como P2 > P1 F2 > F1
Cuando un cuerpo est sumergido en dos
o ms lquidos no miscibles y de diferente
Luego, existe una fuerza resultante: (F2
densidad, experimenta la accin de un
F1) a la cual se denomina empuje
empuje resultante.
hidrosttico (E).
E = F2 F1
E = P2A P1A
E = (P2 P1) A
E = L g (h2 h1)A
A

B ET = EA + EB + EC
E = L . g . Vsum
C

Donde:
Vsum : Volumen sumergido

Experimentalmente, Arqumedes
PROBLEMAS RESUELTOS
comprob que el valor del empuje es
igual al peso del lquido desalojado.
1. Una pieza de metal pesa 1800N en
el aire y 1400N cuando est
sumergida en agua. Halle la
Lquido
desalojado densidad del metal.

Solucin
E
O
FSICA
Recordemos que:

E = peso real peso aparente Solucin:


Trazamos la isbara (por el punto (2)
E = 1800N 1400N = 400N

Adems, sabemos que: E = L g Vs

H2O . g . Vsum = 400N A


kg m
10 3 3
x 10 2 x Vsum = 400 N
m s

Vsum = 4 x 10-2 m3 ........ (1)


1 2 ISBARA
Para hallar la densidad del cuerpo (c)

mc
c = ( v c = v sum )
vc
Sobre (1) presiona el gas encerrado a y
61 cm de Hg. Luego:
w
g w 1800 N P1 = PHg + PA ..... (1)
c = = =
v sum g.v sum m
10.4 x 10 2 2 x m 3
s Sobre (2) solamente acta la atmsfera,
luego:
c = 4500 kg/m3
P2 = Patm............ (2)

(1) = (2) PHg + PA = Patm
c = 4,5 g/c.c. PA = Patm - PHg
PA = 76 cmHg 61 cm Hg
2. Halle la presin del gas encerrado
en el recipiente A
pA = 15 cm Hg

Nota:
Patm <> 76 cm Hg
A
3. Un oso polar que pesa 550 kg flota
sobre un trozo de hielo, conforme
el hielo se derrite. Cul ser el
61 cm volumen mnimo de hielo a fin de
que el oso polar no se moje las
Hg garras?
Densidad del agua salada:1,03
gcc.

O
FSICA
Densidad del hielo: 0,92 g/cc

Solucin PRCTICA DIRIGIDA


El volumen del hielo ser mnimo cuando
las garras del oso estn a punto de 1. Si por la rama izquierda del tubo
mojarse. en U de seccin constante, se
vierte una columna de 40 cm de
E = WH + Wo un lquido x y el nivel de agua en
la rama derecha se eleva a 10 cm.
Wo
Qu densidad tiene el lquido x?

a) 0,2 g/cm3
b) 0,7
H2O
c) 0,3
d) 0,5
e) 0,8

2. Un cilindro flota verticalmente en


agua con la quinta parte de su
volumen emergido, un bloque de
igual masa es colocado encima del
cilindro, entonces el nivel del agua
cubre a ras del bloque. Qu
E W HIELO densidad tiene el bloque?

a) 0,3 g/cm3 b) 0,4


c) 0,5 d) 0,75
L g VH = H g VH + Wo e) 0,2

g VH (L - H) = Wo 3. Un bloque tiene un peso de 50N en


el aire, pero en el agua su peso es
10 x VH (1030 - 920) = 5500 20N. Determine el volumen del
bloque
550 (H2O = 104 N/m3).
VH = VH = 5m 3
110
a) 3 m b) 3 cm3 c) 3 dm3
d) 2,5 cm3 e) N.A.

4. Un bloque se coloca sobre un


recipiente lleno de agua y se
observa que desaloja 20 cm3 de
agua, pero cuando se coloca en un
recipiente de lquido desconocido
desaloja 25cm3. Cul es el peso

O
FSICA
especfico del lquido? (el bloque
flota en ambos casos)
(H2O = 104 N/m3)

5. Qu presin hidrosttica soporta


el fondo del recipiente?

40cm Aceite
= 0,8
40 cm Agua

20 cm Mercurio
= 13,6

a) 9920 KN/m
b) 1000 KN/m
c) 99200 N/m
d) 103KN/m
e) N.A.

6. El bloque A tiene de masa 5g y


volumen 6cm3. El bloque B tiene
de masa 250g y tiene 200 cm3 de
volumen. El bloque C tiene masa
3000g y 3000 cm3 de volumen.
Cul de los tres llega primero al
fondo?

B C
A
a) A
b) B
c) C
d) ByC
e) N.A. AGUA

O
FSICA

Tiene como objetivo conocer una serie de


fenmenos en los cuales las sustancias Observa que la esfera tiene slo energa
(en virtud a ciertas propiedades que cintica respecto a la superficie, entonces
posee) experimentan cambios de tiene energa mecnica.
temperatura; cambios en su estado
fsico, cambios en sus dimensiones
geomtricas cuando intercambia energa
en forma de calor con otros cuerpos.
V=O
Comentario
Hasta ahora slo nos interesaba estudiar
a los cuerpos que cambiaban de posicin m
y rapidez, es decir en mecnica
analizamos la constante transformacin
que experimentaba la energa cintica en Al chocar con la pared dicha esfera se
por ejemplo energa potencial detiene, es decir su energa cintica es
gravitatoria, ahora entendemos como la cero. Entonces, la esfera no tiene energa
energa mecnica se transforma en otro mecnica respecto al piso.
tipo de energa.
Qu ocurri con la energa
El estudio de los fenmenos trmicos mecnica de la esfera?
nos permitir responder a las siguientes
preguntas: Recuerdas que la energa no se crea ni se
Qu ocurre con la naftalina al ser dejada destruye, slo experimenta cambios,
al aire libre?, Qu ocurre si mezclamos entonces es lgico pensar que la energa
dos sustancias a diferentes mecnica se transforma en otro tipo de
temperaturas?, Porqu existe una energa que ocasionan nuevos cambios
separacin entre los riele de un tren? para nuestro entender, por ejemplo el
Consideremos una pequea esfera de hecho que la esfera est deformada y se
plomo deslizndose sobre una superficie encuentre ligeramente ms caliente tiene
horizontal lisa. que estar relacionada con esta
transformacin de energa, para
comprender esto nos hacemos la
siguiente pregunta:
v PARED
DE
ACERO
Qu ocurre en el interior de la
m m esfera?

Para ello analicemos en forma prctica


un modelo mecnico.

O
FSICA
EC : Suma de las energas debido al
RESORTE movimiento trmico
V=O
EP : Suma de las energas debido a la
MODELO interaccin elctrica.
MECNICO DE
UN SLIDO Unidad: Joule (J)
Calora (Cal)
MOLCULA

Es posible medir la energa interna


Al interior de la sustancia las molculas de un cuerpo?
se encuentran en constante movimiento Rpta. No, porque en el interior del cuerpo
de vibracin e interaccin, a dichas debido a las constantes interacciones, la
interacciones las representamos con velocidad de las molculas cambian
resortes imaginarios. constantemente y por dicho motivo es
difcil determinar experimentalmente
Debemos mencionar que al movimiento dicha energa interna.
desordenado de un conjunto de Pero, para tener una idea de la situacin
molculas se les denomina MOVIMIENTO energtica en el interior del cuerpo
TRMICO. utilizamos un parmetro macroscpico
denominado temperatura.
Ahora, debido al impacto las molculas
de la esfera experimentan cambios de Qu es Temperatura?
posicin relativa (se acercan o alejan de Es un parmetro macroscpico de un
las otras), variando de esta manera su sistema fsico que nos informa
energa potencial relativa, adems la indirectamente acerca de la situacin
intensidad del movimiento trmico energtica del conjunto de molculas o
aumenta luego del choque, notamos que tomos que forman el sistema fsico. Nos
la energa que hay en el interior de la indica el grado de agitacin molecular
esfera aument y ello se debe a que la que hay en el interior de una sustancia.
energa mecnica se ha transformado y
ha pasado a formar parte del cuerpo. La temperatura y la energa interna estn
relacionados directamente; cuando la
Cmo se denomina a la energa que primera aumenta, la segunda aumenta
posee el conjunto de las molculas tambin y viceversa.
que conforman un cuerpo?
En un gas ideal:
Rpta. Energa Interna

ENERGA INTERNA (U) n


U = Ec
i =1
Es la energa total debido al movimiento
trmico de sus molculas y a la
interaccin entre ellas: 3
U = n. KT
2
U = EC + EP
n : Nmero de partculas
K : constante de Boltzman
O
FSICA
(K = 1,38 x 10-23 J/k) le est transfiriendo cierta cantidad de
energa interna al bloque B y esto
Unidades: S.I. ocurre en forma espontnea; desde la
T: K ; U:J ; K : J/K sustancia de mayor temperatura (A)
hacia el de menor temperatura (B), a
Observacin: esta energa transferida se le denomina
En la vida cotidiana en forma intuitiva calor (Q).
decimos que un cuerpo est Ms
caliente en comparacin con otro cuando Qu es el calor?
tiene mayor temperatura y esto Es aquella energa que se transfiere en
implicar tambin mayor energa forma espontnea de un cuerpo a otro,
interna. debido a la diferencia de temperatura que
entre ellos existe.
Interaccin Trmica: Calor
Qu ocurre cuando ponemos en Cundo cesa la transferencia de
contacto a dos cuerpos o sustancias a energa?
diferentes temperaturas?. Cuando ambas sustancias alcanzan una
misma temperatura llamada
Para esto consideremos dos bloques de Temperatura de Equilibrio Trmico (TE)
un cierto material de modo que .
ToA>ToB.
TfA = TfB = TE
Inicialmente:

El proceso analizado anteriormente


ToA ToB
podemos representarlo de una manera
ms sencilla mediante un DIAGRAMA
A B LINEAL DE TEMPERATURA, como se
muestra:

QG QP
Tf A Tf B

CALOR

AISLANTE
TRMICO ToB TE ToA T(C)
A B
Por conservacin de la energa:

CONDUCTOR
TRMICO
(INMVIL)
QGANADO(B) = QPERDIDO(A)

En general:
Al ponerlos en contacto, observamos que
la temperatura de B, se incrementa, por
lo tanto aumenta su energa interna, por QG = QP
ello podemos concluir que el Bloque A

O
FSICA
Qe : Cantidad de calor ganado (II)
QP : Cantidad de calor perdido.
CANTIDAD DE D.P. MASA DEL
CALOR CUERPO
(SUMINISTRADO)
m
EFECTOS FSICOS PRODUCIDOS Q
POR EL CALOR
T 2
1. Cambio de temperatura de la T 1
Q2
sustancia. m 10 m
2. Cambio de fase (bajo
determinadas condiciones) Q1 To To
3. Cambio de dimensiones
T 1 < T 2
geomtricas de los cuerpos
(Dilatacin). Adems podemos observar que cuanto
mayor cantidad de calor se le suministra
CAMBIO DE TEMPERATURA a la sustancia, mayor ser el cambio en
su temperatura.
Cuando una sustancia gana o pierde calor
experimenta ciertos cambios en su Q D.P. T
temperatura, el cual est relacionado
directamente con las propiedades Luego:
trmicas de la sustancia.
Qs = Ce . m . T
Calor Sensible (Qs). Es la cantidad de
calor que se requiere para que una
sustancia cambie de temperatura. Donde:
Qs : Calor sensible (caloras: cal)
m : masa de la sustancia (g)
T: cambio de temperatura (T)
Veamos el siguiente caso: Ce: Calor especfico (depende del tipo de
sustancia y de la fase que se encuentra).
(I) cal

g. C

Calores especficos ms usados (a la


10Q
m 10 m presin P = 1 atm)
Q
cal
SUSTANCIA Ce .
g. C
Agua Lquida 1
Se desea que ambos recipientes alcancen Agua Slida (Hielo) 0,5
la misma temperatura, entonces se debe Vapor de agua 0,5
transferir MAYOR calor al recipiente que Aluminio 0,215
tiene MAYOR masa. Vidrio 0,2
Cobre (Cu) 0,093
Luego: Plomo (Pb) 0,03
O
FSICA
Cal
Qu significa Ceagua =1 ?
Lquida g C
Respuesta:
Significa que para que 1g de agua lquida
vare su temperatura en 1C se le debe
transferir 1 Cal.
Observacin
Ep>>>Ec EpEc Ec>>>Ep
1 cal = 4,186 J o
1 J = 0,24 caloras
Qu es un cambio de fase?
Es la transformacin fsica que
Qu es una sustancia pura?
experimentan las sustancias homogneas
Es aquella que mantiene una composicin
al ganar o perder cierta cantidad de
qumica homognea ante un suministro
energa trmica.
de calor, es decir no reacciona, no
experimenta disociacin atmica en sus
En los cambios de fase, se modifican las
molculas.
interacciones moleculares, lo cual implica
una variacin de la energa potencial
Se consideran sustancias puras al agua,
intermolecular en las sustancias,
aire seco, el oxgeno, etc.
mantenindose la temperatura constante.
Qu es una Fase?
Los cambios de fase de una sustancia
pura son:
Es aquella estructura fsica que presentan
las sustancias homogneas en
LQUIDO
determinadas condiciones de presin y
N

temperatura.

VA
CI

CO

PO
CA

RI
ND
FI

ZA
N
DI

Una misma sustancia puede estar en fase


E

LI

CI
NS
SI
SO

N
FU

slida, liquida o gaseosa.


ACI

SUBLIMACIN
N

DIRECTA

Veamos: SLIDO GASEOSO


SUBLIMACIN
REGRESIVA

En que condiciones una sustancia


cambia de fase?
A determinados valores de presin y
FASE SLIDA FASE LQUIDA FASE GASEOSA temperatura conocidos como
condiciones de saturacin.

Por ejemplo, el plomo cambia de la fase


slida a la fase lquida a la temperatura
de 325C y a la presin de 1 atm.
CAMBIO DE FASE

To = 20C T = 325 C 325 C 325 C T > 325C


O Slido
Lquido
Pb Pb
Pb

GRAN COHESIN MENOR


MOLECULAR COHESIN MINIMA
COHESIN Y
MOLECULAR
GRAN
RESPECTO A MOVILIDAD
FSICA

CASO II
Cuando suministramos calor (Qs) a la
barra de plomo en primer momento To=325C;P=1ATM
To=325C;P=1ATM
notaremos que la temperatura se
(Pb)
incrementa, esto significa que la energa m Luego m
cintica de las molculas est
aumentando y por lo tanto aumenta la Pb
energa interna (U) del plomo. QT2

En un segundo momento cuando el plomo llega a


una temperatura de 325C, tal temperatura se En el caso I, necesitamos suministrarle
mantiene constante a pesar que se le sigue
suministrando calor observndose que el plomo
mayor calor de transformacin que en el
empieza a derretirse, es decir fusionar. caso II, debido a que en el calor I, la
barra de plomo tiene mayor masa.
Por qu no cambia la temperatura
suministrando calor, cuando se El calor de transformacin (QT) es directamente
proporcional a la masa (m).
encuentra a 325C?
Es porque el calor suministrado es absorbido por
el plomo para romper los enlaces QT
QT Dp m = Cons tan te = L
intermoleculares, separndose las molculas es m
decir el calor suministrado pasa a incrementar la
energa potencial de las molculas ms no a
incrementar la energa cintica por consiguiente la
QT = mL
temperatura aumenta, entonces decimos que el
plomo est cambiando de fase slida a fase
lquida. Donde:
L: calor latente su valor depende de la
Cmo se llama a la cantidad de sustancia y cambio de fase.
calor necesario para que una Cal KCal
sustancia cambie de fase? Unidad: ;
g kg
Se le llama Calor de Transformacin
(QT), para nuestro caso en condiciones de
Por ejemplo:
saturacin (T = 325C, P = 1ATM).
Para el plomo
CASO I

To=325C;P=1ATM
1. Fusinsolidificacin
(T = 325C, P = 1ATM)
To=325C;P=1ATM
(Pb) Luego 2m Cal KCal
Lfusin = Lsolidificacin = 5,95 = 5,95
g Kg
Pb
QT1
O
FSICA
2. Vaporizacin-condensacin 3. Cierta cantidad de aceite
(T = 1750C, P = 1ATM) incrementa su temperatura en
12C cuando se le suministran
Cal KCal 300Cal; si a esta misma cantidad
Lvaporiz= LCondens = 175 = 175
g Kg de aceite le quitamos 200cal de su
Para el agua energa interna En cunto
disminuir su temperatura inicial?
1. Fusin-solidificacin (T = 0C,
P = 1ATM) Rpta. ............................

Cal KCal 4. Cul es la temperatura en la


Lfusin = Lsolidificacin = 80 = 80
g Kg mezcla de 50g de agua a 20C con
50g de agua a 70C. si el
recipiente en el cual se vierten no
2. Vaporizacin condensacin gana ni pierde calor?
(T = 100C, P = 1ATM)
Rpta. ............................
Cal KCal
Lvaporiz= LCondens = 540 = 540 5. Se tiene 5g de hielo a 0C Cul
g Kg
ser su temperatura final si se le
proporcionan 400 caloras?
Que significa para el agua que
Cal
Lfusin = Lsolidif = 80 ? Rpta. ............................
g
6. Determine la cantidad de calor
Significa que por cada gramo de necesario para llevar 50g de hielo a
agua le debemos entregar o 10C hasta vapor de agua a
sustraer 80Cal a condiciones de 100C
saturacin para que cambie de (CeHielo = 05,cal/gC)
fase.
Rpta. ............................
PRACTICA DIRIGIDA
7. Un recipiente de una masa
1. Se observa que 200g de aceite, despreciable contiene 500g de
descienden su temperatura en 7C agua a 80C Cul debe ser la
cuando piden 0,7 Kcal Cul es el cantidad de hielo a 20C que se
calor especfico del aceite? debe colocar en el agua para que
la temperatura final sea 50C (Dar
Rpta. ............................ una respuesta aproximada)?

2. Se tiene su calormetro de cobre de Rpta. ............................


300g (Cecu = 0,19 cal/gC) Cul
es el equivalente en agua de dicho 8. Halle la capacidad calorfica de una
calormetro? sustancia si al entregrsele 0,3 Kcal
eleva su temperatura desde 15
Rpta. ............................ hasta 35C

O
FSICA
a) 10 cal/C b) 15cal/C increment la temperatura del segundo
c) 25 cal/C d) 30 cal/C cubo?
e) 50 cal/C
a) 10C b) 20C c) 30C
9. Se muestra la curva del d) 40C e) 50C
calentamiento de una sustancia
desconocida, si la muestra es de 13. Se tiene el grfico temperatura-calor,
suministrado para una muestra de 6g de
50g Cul es la capacidad calorfica cierto material, se pide el calor latente de
especfica? fusin.
TC
T(C)
90

40

20 20 140 Q(cal)
200 380 500
-10
Q(Kcal) -20

0,1 0,15
a) 10 Cal/g b) 15 Cal/g
a) 0,1 cal/gC c) 20 Cal/g d) 25Cal/g
b) 0,05 cal/gC e) 30 Cal/g
c) 0,15 cal/gC
d) 0,2 cal/gC 14. En un recipiente de capacidad calorfica
e) 0,5 cal/gCC despreciable se tiene un bloque de hielo
de 2,2Kg a 0C. Calcular a que
10. Si el equivalente en agua de un temperatura se debe poner en contacto
calormetro es 300g. Hallar el valor de su con el hielo, una bola de fierro de 8 Kg de
masa si el material del cual esta masa, para lograr derretir el hielo en
construido tiene una capacidad calorfica forma exacta (CeFE=0,11 Cal/gr)
especfica de 0,75 cal/gC?
a) 150C b) 170C
a) 400g b) 200 c) 800 d) 300 e) c) 200C d) 225C
500 e) 252C

11. En un recipiente de capacidad calorfica 15. En un calormetro de capacidad calorfica


despreciable se mezclan 70g de aceite a nula se introducen 500g de agua a 0C,
50C con mg del mismo aceite pero a 100g de hielo a 0C y 200g de vapor de
10C obtenindose una temperatura final agua a 100C. Hallar la masa de vapor en
de 35C. Hallar m. el equilibrio, aproximadamente.

a) 45g b) 42 g c) 40 a) 74g b) 78g c) 72g d) 70g e) 76g


d) 36 e) 30
16. Se tiene 20g de hielo a 0C
12. Dos cubos del mismo material se ponen Cunto trabajo se debe efectuar
en contacto, uno a 100C y el otro de para fundirlo completamente?
10C. Si sus aristas son e y 2e
respectivamente. En cuanto se
a) 6688J b) 6954J
O
FSICA
c) 5972J d) 4866J
e) 7220J

O
FSICA

INTRODUCCIN dilatan (expanden) al aumentar de


Sabemos que todo cuerpo est temperatura.
VF
constituido por molculas que se
encuentran en constante movimiento e
interaccin. Para describir tal LO, T o
comportamiento se utiliza en forma
prctica el modelo mecnico-molecular,
en el cual las molculas en constante
movimiento estn ligadas entre s por x
LF, T F Vo
resortes microscpicos que
continuamente se deforman, indicando Considerando las dimensiones de los
esto la interaccin. cuerpos, la dilatacin trmica puede ser:

1 Lineal. De una sola dimensin

Lo
BARRA METLICA To
L
TF
MODELO
MECNICO LF
MOLECULAR
Se cumple:

Qu sucede si la temperatura de la L
=
barra se va incrementando? LoT
Sus molculas van incrementando sus
oscilaciones, lo que permite que la : Coeficiente de Dilatacin Lineal
distancia relativa entre ellas se
incremente y como consecuencia, las
dimensiones de la barra empiezan a L = Lo . T
incrementarse (expandirse). En
conclusin: al aumentar la temperatura,
la barra se dilata (expande). LF = Lo (1 + T)

* Qu es la Dilatacin Trmica? 2 Superficial: De dos dimensiones


Es aquel fenmeno fsico que
experimentan los cuerpos cuando la TO TF
separacin relativa entre sus molculas
se incrementa, debido a incrementos de
temperatura.
Salvo excepciones, las sustancias en
todas sus formas, slido, lquido y gas se AO
AF
O
FSICA

EJEMPLOS DE APLICACIN
Se cumple:
Calcular las longitudes en cm de una
A varilla de latn y una varilla de hierro
=
A O T para que tengan una diferencia de
longitud constante de 5 cm a todas las
temperaturas. Los coeficientes de
: Coeficiente de Dilatacin Superficial. dilatacin lineal del latn y del hierro son:
0,000018C-1 y 0,000012C-1
Luego: Respectivamente.

AF = Ao (1 + . T) Solucin
Para que la diferencia de longitudes sea
( = 2) la misma a cualquier temperatura,
debern experimentar ambas varillas
3 Volumtrico: De tres igual cambio en sus longitudes; es decir,
dimensiones: si ambas aumentan o disminuyen su
longitud en la misma medida, la
TF diferencia de sus longitudes ser siempre
TO la misma.

Luego:
RO LL
RF
5 cm
VO
VF
LH
Se cumple:
LH = LL
V LH . H . T = LL.L . T
= LH . 1,2 x 10-5 = LL . 1,8 . 10-5
Vo.T
3
LH = L L (LH>LL)
: Coeficiente de Dilatacin Volumtrico 2

Por condicin:
Luego: LH LL = 5cm
3
L L L L = 5 cm
VF = Vo (1 + T) 2

( = 3) LL = 10 cm; LH = 15 cm

O
FSICA
TERMODINMICA

Qu estudia la termodinmica? CONCEPTOS PRELIMINARES


El intercambio de energa entre sistemas
que interactan trmicamente. En 1. Sistema Termodinmico
nuestro caso, un sistema sera un gas Porcin de materia que separemos
ideal, otro sistema sera el recipiente que imaginariamente, del medio
lo contienen y otros sistemas seran las externo a ella y la cual interacciona
sustancias que rodean al gas ideal. con su medio ambiente y como
consecuencia de la cual se da una
El bloque es un transferencia de calor.
sistema
2. Sustancia de Trabajo
Sustancia empleada como medio
de transporte del calor as como de
intermediario en la transformacin
El gas ideal de calor en trabajo. Usualmente es
es un sistema un gas.

3. Energa Interna (U)


Energa de un cuerpo la cual est
relacionada con el movimiento
Los gases ideales tienen energa trmico de las molculas que lo
potencial? forman.
No, porque a nivel molecular la Si no hay cambio de fase, la
separacin relativa entre las molculas es energa interna es una funcin de
muy grande, lo que significa que las la temperatura absoluta por lo que
interacciones entre ellas son el cambio de energa interna solo
despreciable. depende de la temperatura del
Como las molculas estn en constante estado final y la del estado inicial
movimiento, significa que la energa pero no de la forma como se ha
asociada a un gas ideal es cintica, pasado de estado inicial al final.
luego:
4. Proceso termodinmico
U Gas = E CINETICA Sucesin de estados por los cuales
ideal de las molculas se hace pasar un sistema con la
finalidad de transformar calor en
trabajo.
Si la temperatura de un gas ideal se El estado de un sistema esta
incrementa, sus molculas presentan determinado por el conjunto de
mayor rapidez (V) y por lo tanto mayor propiedades que posee en un
energa cintica, lo que significa mayor momento dado. Estas propiedades
energa interna. se determinan por ciertas
magnitudes, que determinan el
comportamiento del sistema,
denominadas variables de estado.

O
FSICA
5. Ciclo Termodinmico entrega o sustrae a un sistema es igual al
Es una sucesin de procesos la trabajo realizado por o sobre el sistema
cual permite evolucionar a un mas el cambio correspondiente de
sistema de estado inicial (I) hacia energa interna (U).
un estado final (F) y volver al
inicial de manera que durante la Q=W+U
realizacin del ciclo parte del calor
suministrado se convierte en
trabajo. CALORES ESPECFICOS DE LOS
GASES
Como el sistema vuelve a su El calor necesario para elevar la
estado inicial se tiene que el temperatura de un gas depende de como
cambio neto de energa interna es se halle confinado. Por ejemplo si el
nulo y el trabajo neto. La suma de volumen se mantiene constante el calor
los trabajos realizados en cada uno recibido por el gas se convierte
de los procesos. El trabajo neto se totalmente en energa interna elevando
representa por el rea encerrada por lo tanto la temperatura. Debido a
por el ciclo en el plano P.V. esto para un gas se distinguen 2 calores
especficos:

P V : Calor especfico a volumen constante


PROCESO F
P : Calor especfico a presin constante.

Para el caso de gases es usual emplear el


I
nmero de moles en vez de la masa,
AREA = W razn por la cual se define el calor
especfico molar:
V
Vi VF
Cantidad de calor(Q)
C=
P CICLO [( N de moles(n)]T
I
Cumplindose que C = Me
( M : masa molar)
Para un gas dado se cumple:
F
V (1) Cp > C v (2) Cp = Cv + R
(3) Coeficiente adiabtico ()

PRIMERA LEY DE LA p
=C = >1
p
TERMODINMICA v
Cv
En todo proceso termodinmico se
Gases Monoatmicos: = 5/3
cumple que la cantidad de calor que se
Gases Diatmicos: = 7/5
O
FSICA

Cmo podemos variar la energa TRABAJO REALIZADO POR UN GAS


interna de un gas ideal? IDEAL
Variando su temperatura, lo cual se logra
suministrndole o extrayndole energa. Cuando un gas confinado en un
recipiente experimenta un proceso de
CASOS: expansin o compresin desarrolla o
a. Trasfirindole energa en forma de consume respectivamente un trabajo el
calor cual depende de la forma como vare la
presin y volumen del gas, es decir del
proceso realizado. Para cualquier proceso
el trabajo queda representado por el rea
FGas encerrado por la grfica del proceso en el
FGas plano P-V y el eje de los volmenes,
tenindose los casos:
P Expansin P Compresin
VF>Vi
F VF<Vi
I
x
QEntrega al gas
I F
W = Area (+) W = Area (-)

Se cumple: V V
Vi VF VF Vi

Q Entrega = U Experiment a + WRe aliza PROCESO TERMODINMICOS


al gas el gas gas
SIMPLES:

(1) Iscoro (V = Const.) (2) Isobrico (P = Const)


(1 Ley de la Termodinmica)
P F P

b. Trasfirindole energa, mediante


trabajo realizado. I Po F

W
V V
v Vi VF

W=0 W = Po (VF - Vi)

F F
(3) Isotrmico (T=Const.) (4) Adiabtico (Q=0)

P I P I
x
Qlibera el gas
F F
W W
Se cumple: V V
Vi VF Vi VF

E su min istra = U Del + Q Libera


gas e lg as V
WF W=2.3PiViLog F W= PF VF Pi Vi
Vi 1

O
FSICA
OBSERVACIONES:
Solucin
1. Como el cambio de energa interna Proceso isotrmico: T = 0
solo depende del estado final e Entonces: Q12 = W 12
inicial, siempre se puede relacionar
con el cambio de energa interna
en un proceso isocoro entre las Luego: W = A
mismas temperaturas:
40 + P1
W= KPa.(2 0,5)m3
2
U = QV = C V n (TF - Ti)
40 + P1
90 KJ = . 1,5 KJ
2
2. La isoterma (en el plano P-V) es
una curva simtrica respecto a la
bisectriz del primer cuadrante. P1 = 80 Kpa

3. La adiabtica es una curva ms


inclinada que la isoterma, es decir, MQUINAS TRMICAS Y LA
su pendiente vara ms SEGUNDA LEY DE LA
rpidamente. TERMODINMICA

Qu es una mquina trmica?


Ejemplo de Aplicacin
En el proceso indicado, las temperaturas Es un dispositivo que convierte energa
en el estado (1) y (2) son iguales. Si el trmica en otras formas tiles de energa
calor transferido en el proceso es 90 kJ; como la energa elctrica y mecnica.
calcular la presin en el estado 1,
sabiendo que la presin en el estado 2, Dispositivo diseado con la finalidad de
es 40 kPa. trasformar calor en trabajo, para lo cual
la mquina sigue un ciclo termodinmico.

Cul es la funcin de una mquina


trmica?

Que una sustancia de trabajo recorra un


(KPa) proceso cclico durante el cual:
P
(1) 1. Se absorbe calor de una fuente a
P1
alta temperatura
(2)
40 2. La mquina realiza un trabajo, y

3. Libera calor a una fuente de


o 0,5 2 V(m3) temperatura ms baja.

O
FSICA
En una mquina de vapor, como ejemplo
de un proceso cclico, el agua es la
sustancia de trabajo.

Toda mquina trmica se puede


representar por el esquema:

Segunda Ley de Termodinmica


Fuente T 1 : Alta
Como se ha visto, la primera ley es una
Q1 aplicacin de la conservacin de la
energa, pero no afirma nada respecto al
Mquina Wtil curso que toman los acontecimientos en
Trmica el universo. Se conserva la energa
cuando cae una piedra y su energa
Q2 potencial gravitatoria se transforma en
cintica. Pero al chocar la piedra con el
Sumidero T 2 : Baja suelo y al llegar al reposo, su energa
cintica se transforma en energa
trmica.
O tambin
Sin embargo, una piedra que se
encuentra en reposo sobre el suelo nunca
Foco caliente Foco fro o cambia la energa trmica de ella y de la
Q1 Mquina Q2
o reservorio
Trmica
sumidero de vecindad en energa cintica y sale
de calor (T 1) calor (T 2)
disparada hacia arriba. La primera ley no
W
excluye esta posibilidad ya que este
proceso inverso tambin conserva la
energa. Pero tal proceso no ocurre.
Donde se tiene que el trabajo neto:
Hay otros procesos en el universo que no
estn excluidos por la primera ley que no
W Q 1 Q2 ocurren. Por ejemplo, en forma
espontnea el calor fluye de un cuerpo
Donde la desigualdad caracteriza las caliente a otro, fro pero no
mquinas reales y la igualdad a las espontneamente del cuerpo fro al
perfectas o ideales. caliente. Esto nos indica que en la
naturaleza los procesos se presentan en
Eficiencia Trmica (n) una sola direccin en forma espontnea;
La eficiencia de una mquina trmica (E) la segunda ley ha sido formulada en
se obtiene mediante la relacin entre el varias formas, todas ellas equivalentes.
trabajo realizado y la energa recibida del Una de las ms antiguas establece:
foco caliente.
El calor fluye espontneamente de un
objeto caliente a otro fro y no a la
WUTIL Q1 Q 2 Q inversa. En virtud de esto, es imposible
n MAQ = = = 1 2
Q1 Q1 Q1 que en un proceso cclico se transfiera

O
FSICA
calor de un cuerpo de baja temperatura a Est constituido por dos procesos
un cuerpo de alta temperatura a menos isotrmicos y dos adiabticos.
que se efecte un trabajo externo sobre
el sistema que efecta el ciclo.

CONCLUSIONES DE LA SEGUNDA
P
LEY DE LA TERMODINMICA
Q1
A
1. Es imposible tomar calor de un B
recipiente y convertirlo W T1
completamente en trabajo sin que
D T2
efecten otros cambios en el C
sistema o en sus alrededores. Q2

2. Es imposible para cualquier


V
proceso tener como nico A B: proceso isotrmico
resultado la transferencia de calor B C: proceso adiabtico
desde un cuerpo fro a uno C D: proceso isotrmico
caliente. D A: proceso adiabtico

Ejemplos de Aplicacin Cuando una mquina trmica trabaja con


En una mquina trmica que funciona este ciclo, obtiene un trabajo neto
segn el ciclo de Carnot, el calor mximo, con una cantidad de calor
rechazado por el foco fro equivale a la suministrada a la sustancia de trabajo. Se
cuarta parte del calor que absorbe la observa que en este ciclo U = 0. La
mquina. Si la temperatura del foco fro eficiencia mxima que se logra en este
es 7C, calcular la temperatura del foco ciclo se determina por:
caliente.

Solucin Q2 T
n max = 1 = 1 2
Por dato: Q1 T1
Q
Q2 = 1 ; y T2 = 280K
4
Adems:
Luego:
Q1 Wneto = Q1 Q2
Q1
= 4
T1 280
T1 = 1120 K PRACTICA DIRIGIDA

T1 = 847C 1. Una sustancia desconocida de = 4


x10-2C-1, tiene una densidad de
Ciclo de Carnot 10g/cc a 20C. Determine la
Ciclo terico que le permite a una densidad de tal sustancia, cuando
mquina ideal transformar la mayor haya alcanzado una temperatura
cantidad de calor en trabajo, es decir, es de 120C.
el ciclo de mxima eficiencia. Rpta. ............................
O
FSICA
x 103 cal. Determine el trabajo
2. Indique lo correcto con respecto al realizado por la mquina durante
comportamiento de un sistema el ciclo
termodinmico en el diagrama Rpta. ............................
mostrado.
P(Pa)

d c

a b

V(m3)

a) ab: el proceso es isotrmico


b) ab : el proceso es iscoro
c) abc: no se desarrolla
trabajo
d) cd: el trabajo es de
expansin
e) en el ciclo, el trabajo es
negativo
Rpta. ............................

3. El gas en un recipiente de
capacidad calorfica despreciable
se le transfiere 600J. Si dicho gas
desarrolla un trabajo de 400J,
determine en cunto vari su
energa interna.
Rpta. ............................

4. Una mquina de vapor tiene una


caldera que opera a 500K. El
calor cambia el agua a vapor, el
cual mueve un pistn. La
temperatura de escape es la del
aire exterior aproximadamente
300K. Determine la mxima
eficiencia trmica de sta mquina
de vapor.
Rpta. ............................

5. Una mquina trmica trabaja con


un ciclo de carnot entre 227C y
127C durante el cual absorbe 2,5
O
FSICA

OBJETIVO:
Conocer la carga elctrica y algunos - 1 milicoulomb: 1 mc = 10-3c
fenmenos relacionados con ella. -6
- 1 microcoulomb: 1 uc = 10 c
-9
- 1 nanocoulomb: 1 mc = 10 c
Carga elctrica

* Cantidad de carga del electrn y


protn.
qe = -1,6.10-19c
N qp = -1,6.10
-19
c
P P e
N
PROPIEDADES DE LA CARGA
ELCTRICA

1. Cuantizacin de la Carga

A la propiedad que presentan los q * cuerpo electrizado


electrones y protones y que nos *
permite explicar su atraccin y/o
repulsin le llamamos CARGA
ELECTRICA q = n |qe|

Por convencin al electrn se le asocia = # de electrones ganados o


carga negativa y al protn positiva perdidos.

* Un cuerpo se electriza cuando gana o 2. Conservacin de la carga


pierde electrones. En un sistema elctricamente
aislado.
Si gana electrones Si pierde electrones
(exceso de e) (defecto de e )
q inicio = q final

---- Ejem:
- - - + + + Se tiene 2 esferas idntica una
- - + + + + + + electrizada con q = 8 c y la otra
- - - - - + + no electrizada, si se ponen en
contacto determine el # de
electrones transferidos.
Se electriza Se electriza
Negativamente Positivamente
INICIO CONTACTO FINAL
* La carga elctrica (q Q) se (1) (2) (1) (2) (1) (2)
expresa en COULOMB (C).
q = 8c q=0 e- q q

O
FSICA
1. Conservacin de la carga
qinicio = qfinal Campo Elctrico
Entre partculas elctricas cmo es
posible la fuerza de atraccin o repulsin?
8c + 0 = 2q q = 4c

2. Cuantizacin de la carga

q = n qe Q q
+ +
4.10
-6
= n x 1,6 x 10
-19 Fe Fe

n = 25.1012 e

Leyes de Electrosttica Esta es posible porque a cada cuerpo se


le asocia un medio denominado CAMPO
1. Ley Cualitativa ELECTRICO.

El campo elctrico es materia no


sustancial que se asocia a todo cuerpo
electrizado la cual trasmite la
interacciones elctricas.

- + + + Cmo representamos el campo


Atraccin! Repulsin! elctrico asociados a cuerpos
electrizados?
2. Ley Cuntitativa (ley de coulomb)
Para ello faraday idea las Lneas de
Fuerza o Lnea de Campo Elctrico,
q1 q2 colocando cargas de prueba q en el
Fe ... Fe
+ - campo que se analiza.

d + Fe
qo
Fe +
K q1 q 2 +
Fe = qo Lineas
d2 + Salientes
qo Fe

Donde: K Constante elctrica


Para el aire o vaco K 9 x 109 N
2 + qo
m /c2 Fe
+ -
Fe qo
* Para otro medio Lineas
Fe
+ qo Ingresantes
K vaco
Kmedio =

: Permitividad dielctrica del medio


(1)
O
FSICA
Cmo caracterizamos en cada punto el OBSERVACIONES
campo elctrico debido a la Fe que
transmite? 1. La E no depende de la qo

Para ello usamos una magnitud


vectorial denominado Intensidad de _
Campo Elctrico ( E ), cuyo valor EA
Q dA
expresa la Fe que transmite el campo
A
elctrico por unidad de carga +
dB _
Matemticamente B EB

dB>dA
FeA Fe//E
EA =
qo EB < EA

2. El Vector E es tangente a la
lnea de fuerza y tiene la
Unidad N/C
misma orientacin.

qo Fe Lnea de Fuerza EA
Q
dA +
+ EA A
A
EB
B

EA EB

3. Cuando las lneas de fuerza


* Si qo es (+) la E Fe
estn ms juntas el campo
tienen la misma direccin.
elctrico es ms intenso.

* Si qo es (-) la E Fe
tienen diferente direccin.
EA

A EB
Pero:
B
KQ q KQ
Fe = E = d2
d2

* EB > EA

O
FSICA
4. Las lneas de fuerza es Dp a la
larga de la partcula que la 7. Cuando las lneas de Fuerza
genera. son //, se tiene el Campo
Elctrico Homogneo o
Uniforme, donde la E
permanece constante.
+ +-
+ - ++ - + -
+ + Fe = EIqI -
- + - + + +- + -
+ EB
-
+ + -
+ -
+ -
+ -
+ Fe = KIqoI - -
+ -
5. El nmero de lneas de fuerza
es Dp a la carga de la Ec
partcula que la genera
Energa Potencial Elctrica
Q 2Q (Upe)

Q q Vo = O Liso
1 ^
+ + + + distante

* Al inicio estn en Reposo Ec=0


6. Las lneas de fuerza nunca se
v
cortan porque en un punto se Q q
2
tiene un solo valor de E ; + +
se produce la Superposicin
de Campos Elctricos. * Al cortar la cuerda la esferita q
tiene Energa Cintica.

La Energa Cintica aparece debido al


Q Q TRABAJO MECANICO que realiza el
+ -
Campo elctrico y ello es porque al
inicio hay energa al que denominamos
Energa Potencial Elctrico (Upe)

KQq
U PE = con su signo
d
Q 2Q
+ -
* Upe (+) Repulsin
* Upe (-) Atraccin

O
FSICA
Ejem : * Se observa que se almacena
Dos esferitas electrizadas con -4uc - Upe y que al analizarlo por
6uc estn separados a una gran unidad de carga qo se obtiene
distancia, determine Ud. el W que se
debe realizar para que estn separados Upe W fe A
12 cm, desprecie efectos gravitatorios. = = VA Potencial Elctrico
qo qo
i)
El V es una caracterstica escalar del
campo elctrico debido a la energa que
- + almacena.

con signo
KQ
dmx Pero : UPE = KQqo/d VA =
d

* No tienen Upe porque dmx; por Voltio


medio de una Fext se les junta
pero el WFext sirve para que los OBSERVACIN
campos elctricos interactan.
1 El V no depende de qo

A
- - dA
Q
+ dB
12 cm
B
Wfetx = Upe

9 x10 9 x (4 x10 6 )(6 x10 6 ) dA > dB


Wfetx = VA < VB
12 x10 2
2 Para un sistema de partculas el
Wfext = 1,8J
Vp es la suma escalar.

Potencial Elctrico (V)


+ q3
Veamos que sucede al colocar a qo q2
dentro del campo elctrico de Q - d3
d2

Q q1 d1
qo P
A Fe +
+
Vp = Vp1 + Vp2 + Vp3
dA
* Considerar el signo de la carga.

O
FSICA
5 Para trasladar lentamente se
3 Aquellos puntos donde el emplea un agente externo.
Potencial elctrico tiene un solo
valor se denomina SUPERFICIE
EQUIPOTENCIAL.

C B
+ A
B
Fext

+
Wneto = 0

A Wfe = - Wfext

D 6 En un Campo Elctrico Uniforme:

VA = VB E = Cte
VB = VD A
VA VC
VA > VB B

4 A qo se puede trasladar entre dos Fe


puntos de un Campo Elctrico.
C qo

VA = V C VB=VD
V A > VB
Fe B
+ A
WfeCB = qo (VC - VB) ..... (1)

Como:
dAB Fe|qo| = cte WfeCB = E |qo|dCB....(2)

Luego: (1) = (2)

WfeAB = WAfe. - WBfe. * VC VB = E.d

WfeAB = qoVA - qo VB V

V = E. d // E // d
WfeAB = qo (VA - VB)

Diferencia de Potencial Intensidad de


Elctrico Campo Elctrico
UNIFORME
O
FSICA
Ejem : CAPACIDAD EL
Si el potencial elctrico en A es 90v, LECTRICA (C)
determine la diferencia de potencial
elctrico entre A y B ( VAB ) y el Es una propiedad de la cual gozan los cuerpos
conductores que indica la variacin de su potencial
trabajo que realiza el campo para ante la ganancia o prdida de carga elctrica.
trasladar a q o = +2uC entre A y B.
Q +
A + +
+ + V
d = 0,2 m + +
+ +

d = 0,6m
B
Sol : Q C
C= = faradio = F
V V
FE
Se pide VAB WAB 1F = 10-6F

CAPACIDAD ELCTRICA PARA UNA


q o ( VA - VB ).... (1) ESFERA CON DUCTORIA
Q +
+ +
KQ R
(*) VA = dA
- 90v = K ( Q ) KQ = -18 + +
0 ,2
+ +
KQ 18 +
(+) VB = VB = VB = 30V
dB 0,6

VAB = -60V

C = 4 o . R
A

3r * La capacidad elctrica depende


de las caractersticas geomtricas
+ qo Fext
del conductor.
fe
CONDENSADOR:
Es aquel dispositivo constituido de dos
conductores separados cierta distancia
V = -90v y ambos cargados con cargas del
mismo valor pero de signos contrarios.
B Smbolo:
r = 0,2 m V = -30v

WfeAB = 2.10-6 x-60J


WfeAB = -12.10-5J * Condensador de placas paralelas

O
FSICA
V = V1 + V2 + V3
Vo ^
Q
Para dos condensadores:
d
-Q C1 xC 2
C1 + C 2

Paralelo
A
Co = Eo C1
d
C2
Eo = 8,85 x 10-12 f/m

* Si est lleno de una sustancia C3


aislante (dielctrico)
CE

CE = q1 + q2 + q3
Z1
A
C = K o q = q1 + q2 + q3
d V1 = V2 = V3 = V

PRACTICA DIRIGIDA
El condensador almacena carga y por
lo tanto almacena energa 1. Dos cargas separadas a cierta distancia se
El dielctrico aumenta la capacidad repelen con una fuerza de 200N. si una
del condensador si est conectado a carga se duplica, la otra se cuadruplica y
la nueva distancia es el doble de la
la batera. anterior. Con qu nueva fuerza se
Si est desconectado de la batera su repelen?
capacidad se conserva pero disminuye a) 100N b) 200N c) 400N
su potencial. d) 500Ne) 250N

2. Si: Q1 = 4Q2 Calcular a que distancia


Vo
V= respecto de Q1 se debe colocar una carga
K tal que la fuerza resultante en sta sea
nula.
Asociacin de Condensadores: +Q1 +Q 2

Serie:
C1 C2 C3 CE
3m
a) 1m b) 1,2m c) 1,5m
d) 2m e) 2,5m

q1 = q2 = q3 = q
O
FSICA

Es aquella parte de la electricidad que INTENSIDAD DE LA CORRIENTE


estudia a las cargas elctricas en ELCTRICA (I)
movimiento y los fenmenos que Para provocar la aparicin del campo E,
producen. dentro del conductor, se debe colocar
en los extremos de ste, potenciales
CORRIENTE ELCTRICA. diferentes, ya que el campo seala
Es sabido que en los conductores hacia donde decrece el potencial y las
(metales) existen cargas libres, que se cargas libres positivas se movern en
mueven caticamente debido a la aqul sentido.
agitacin trmica. Para que estas
cargas se muevan ordenadamente es La corriente elctrica en los
necesaria la presencia de un campo conductores circula de lugares de
elctrico que los impulse, en este caso mayor a lugares de menor potencial y
se dir que circula una corriente para que halla corriente debe existir
elctrica a travs del conductor. diferencia de potencial en los extremos
del conductor.
En la realidad las cargas libres en los
conductores son electrones (carga La intensidad de la corriente I nos
negativa) que se movern sentido indica la cantidad de carga que
contrario al campo E, sin embargo, es atraviesa la seccin recta del conductor
un hecho experimental que el en la unidad de tiempo.
movimiento de una carga negativa en
un sentido, es equivalente al Plano
movimiento de una carga positiva del Perpendicular al
Conductor
mismo valor en sentido contrario. x
E
+ + + +
Basndonos en lo anterior supondremos VA + +
de ahora en adelante que la corriente VB
est constituda por cargas positivas,
movindose en el sentido del campo E, Seccin Recta del
esta es la llamada corriente Conductor
convencional.
VA > V B

Q
E I=
t
Donde:
Q = Cantidad de carga que atraviesa la
Corriente Corriente seccin recta del conductor.
Electrnica Real convencional
t = tiempo transcurrido.

UNIDAD: S.I
1 coulomb/segundo = 1 amperio.

O
FSICA
DIFERENCIA DE POTENCIAL Y Nota: las pilas reales tienen resistencia
FUERZA ELECTROMOTRIZ () () interna, que se coloca en serie con la
fuerza electromotriz.
1. Fuerza electromotriz
Es la energa que cada unidad de
carga elctrica gana al atravesar una
fuente de energa elctrica en un R
sentido de (-) a (+) E

ENERGA RESISTENCIA ELCTRICA (R)


= Las cargas al circular a travs del
CARGA
conductor, colisionan con los tomos de
ste debido a lo cual el material se
2. Diferencia de Potencial
opone al paso de la corriente, una
Es la energa que invierte la
medida de dicha oposicin es la
unidad de carga elctrica al desplazarse
resistencia elctrica.
de un punto a otro en el recorrido que
realiza. Se le conoce con el nombre de
Los llamados buenos conductores
cada de tensin.
poseen una resistencia elctrica
. Terminal Positivo pequea y los malos conductores
(AISLANTES) tienen una resistencia
elctrica muy grande.

Experimentalmente se comprueba que


+ la resistencia de un conductor
E Pila Batera homogneo de seccin constante es
-
proporcional a su longitud e
inversamente proporcional a su seccin
transversal.
. Terminal de Menor Potencial
L
Smbolo de las resistencias

UNIDAD: 1 joule/coulomb = 1 voltio.


Analicemos el circuito ms simple que se A
. R
.
puede obtener formado por una batera y
una resistencia en serie, comparmoslo con
su simil mecnico: R L
La persona hace las veces de batera ya R = . L/A
que la persona entrega energa a las R 1/ A
esferas al levantarlas, el rozamiento que
consume la energa entregada reemplazara
a la resistencia del circuito, donde las Donde es una constante del material
esferas representan las cargas que
que constituye al conductor, llamado
constituyen la corriente. A la energa por
unidad de carga que entrega la persona se
resistividad del material.
le conoce como diferencia de potencial.
LEY DE OHM.
x Para materiales metlicos
(conductores) la corriente que los
atraviesa es directamente proporcional
+
E R
a la diferencia de potencial conectada
- en sus extremos. La constante de
proporcionalidad se denomina

O
FSICA
Resistencia Elctrica, del conductor, EFECTO JOULE:
esta Ley fue descubierta Las cargas que forman la corriente al
experimentalmente por el fsico alemn atravesar los conductores van
GEORG SIMON OHM (1789 - 1854). colisionando con los tomos del
material, los tomos al ser golpeados
Se cumple: vibrarn con mayor intensidad con lo
I VAB VAB/I = constante cual el conductor aumenta su
temperatura (se calienta), hasta emitir
calor, este fenmeno se denomina
VAB/I = R = VAB = RI
EFECTO JOULE.

I P = VAB . I

Econsumida
= (R.I) .I Econs = R.I 2 . t en joules
t
R E cons = Q t segundos
R ohmios
I Amperios

.V . AB
pero:
1 joule = 0.24 caloras

Donde: VAB = diferencia de potencial = Q = 0.24 RI2t caloras


VA VB = cada de tensin
I = Intensidad de la corriente
R = resistencia del conductor ASOCIACIN DE RESISTENCIAS:

Se define de lo anterior la unidad I. EN SERIE


M.K.S. de resistencia: En este caso las resistencias se
1 OHMIO = 1 = Voltio/Amperio. conectan una a continuacin de otra, de
tal manera que el voltaje total
POTENCIA ELCTRICA conectado en los terminales V se
Para que las cargas que forman la reparte en cada resistencia en V1, V2,
corriente atraviesan un dispositivo V3
elctrico se realiza un trabajo en cierto
intervalo de tiempo, con lo cual en el Tambin hay que observar que no se
dispositivo elctrico se consumir acumula carga en las resistencias por lo
potencia. cual las corrientes en cada elemento
WAB deben ser la misma; aquella resistencia
Sabemos que: P= que remplaza a las anteriores
t
produciendo el mismo efecto es la
I
llamada RESISTENCIA EQUIVALENTE
A B
(RE)
qVAB q
P= = VAB P= VAB.I R1 R2 R3
RE
t t

Para conocer la potencia consumida en I1 I2 I3 IE


vatios, se debe tener la diferencia de
potencial entre los terminales en voltios .V.
y la corriente que circula en Amperios.
VATIO = VOLTIO x AMPERIO .V .
O
FSICA
CARACTERSTICAS que esta sea lo ms pequea posible
para que el circuito no sea alterado
1. I1 = I2 = I3 = IE 2. V=V1+V2+V3 prcticamente. + -
3. REIE = RII1+R2I2+R3I3

RE = R1+R2+R3
R
II. EN PARALELO
En esta ocasin las resistencias se I A
conectan teniendo terminales comunes,
de lo cual se desprende que todos los
elementos recibirn el mismo voltaje, y Si deseamos medir la diferencia de
la corriente total se repartir en cada potencial entre los extremos de una
resistencia, la resistencia equivalente resistencia, debemos colocar un
es aquella que recibiendo el mismo VOLTMETRO en paralelo con la
voltaje soporta la misma corriente resistencia, la corriente que se dirige a
total. la resistencia se bifurca penetrando
parte de la corriente al voltmetro, la
resistencia interna del voltmetro debe
ser lo mximo posible para que a travs
. de l no pase corriente y el circuito no
I1 R2 I2 R3 I3
.
V Req IE
V
se altere.
+ -

CARACTERSTICAS
1. V1 = V2= V3 = V R
2. V/RE = V1/R1 + V2/R2 + V3/R3
I
1/RE = 1/R1 +1/R2+ 1/R3

V
INSTRUMENTOS ELCTRICOS DE PUENTE DE WHEATSTONE
MEDICIN Este montaje se utiliza muy a menudo
Todo aparato destinado a detectar la para efectuar medidas rpidas y
presencia de corriente elctrica en un precisas de resistencias.
alambre conductor se denomina Fue inventado en 1843 por el fsico
GALVANMETRO, de acuerdo a su ingls CHARLES WHEATSTONE.
escala de medida se puede hablar de c
ampermetro, miliampermetro o
microampermetro. R1 R2

Para medir la corriente que circula por R3


I1 I2
un hilo el ampermetro debe colocarse a b
E
en serie para que toda la corriente que
deseamos medir pase por el aparato. I4 I3
Como el ampermetro tiene una cierta R4 R3
resistencia interna es conveniente
d
O
FSICA

Para poder hallar una de las b


b

resistencias, se busca una relacin tal


que en R3 no circule corriente (I = 0), x
es decir Va = Vb. R1 R2

Se cumple: Y

Z
Vca = Vcb Vad = Vbd a c a c
R3

R1I1 = R2I2 R4I1 = R3I2 R 1R 2 R 2R 3


x= y=
R1 + R 2 + R 3 R1 + R 2 + R 3
Dividiendo las ecuaciones:
R 1R 3
R1 R 2 z=
= R1 + R 2 + R 3
R4 R3
R1R3=R2R4 SUSTITUCIN ESTRELLA - DELTA
R2
Cuando se cumple esta relacin se dice
que el punto est balanceando, y en R5
Rx Ry
no circula corriente.
R1
PUENTE WHEATSTONE Rz R3
MODIFICADO:

RxRy + RxRz + RyRz


R1 =
R Rx
Ry

V
Alambre de seccin
RxRy + RxRz + RyRz
Regla R2 =
graduada recta y resistividad "" Rz
L2
L1
R1 = KL1 R2 = KL2 PROBLEMAS PROPUESTOS

1. Hallar la intensidad de corriente


Luego: que circula por un alambre sometido a
RR2 = Rx R1 una diferencia de potencial de 420 voltios,
R siendo su longitud 1km y su seccin
Rx = Rx 2 cuadrada es de lado igual a 3mm.
R1
( = 1.4 x 10-5 - m)
L
Rx = R 2
L1 a) 0.14 A
b) 0.27 A
c) 0.18 A
SUSTITUCIN DELTA ESTRELLA d) 0.21 A
e) 0.30 A
Un circuito DELTA formado por R1, R2,
R3 puede ser reemplazando por un 2. Hallar la corriente que circula por
circuito ESTRELLA equivalente, formado un calentador elctrico de 20,
por X, Y, Z tal que se cumple: para que en 10min caliente 432
O
FSICA
grs de agua desde 20C hasta 5. Hallar la corriente por la
80C resistencia de 2

a) 1.47A 3 2
b) 2.66 A
c) 3A
d) 4.16 A 3 6v
e) 5A 3
4

3. Hallar la resistencia equivalente 3


entre a y b a) 3A
b) 2A
a c) 1.2 A
d) 1.71 A
3 e) 0.85 A
9
6. La corriente I en el circuito es

6 18 1 1
c .
a)
b)
2
1.5
. b
c) 0.66
6v 6v
1
d) 8
I
e) 36

4. Calcular lo que marca el


ampermetro, si V = 20 voltios. a) 0 A b) 2 A c) 3 A
d) 4 A e) 6 A
2
7. El voltmetro v de la figura,
indica 117 voltios y el
ampermetro A 0.13 amperios.
La resistencia del voltmetro es
A 2 V 9000 ohmios y la del
ampermetro 0.015 ohmios.
4 Cul es el valor de la resistencia
R?
V

a) 20 amp b) 10 amp
c) 15 amp d) 8 amp R
e) 5 amp
A
a) 106
b) 105
c) 104
d) 103
e) n.a.

O
FSICA
8. La corriente I mostrada en el circuito 12. En el circuito mostrado, hallar
es igual a: Rx, si VAB = 0, R1 = 10, R2 =
5 y R3 = 15
A
1 a) 3.34 R1 Rx
1
1 b) 7.5
1 c) 30 2
1 1 d) 28
I
e) 20 R2 R3
B
3v + V -

a) 0.0A b) 0.5 c)1.0A 13. Hallar la resistencia equivalente


d) +1.0A e) +3.0A vista desde A- B
3
9. Calcular el sentido y la intensidad de a) R
5
la corriente elctrica b
7
b) R
40
5
100v
50v c) 2.5R
R1 R R2
4
300v d) R
5
30 e) 1.5R
A B
5 2R R
a) 2 A : Horario
b) 4 A : Antihorario 14. Hallar el calor disipado en la
c) 2 A: Antihorario unidad de tiempo por la
d) 4 A: Horario resistencia de 3
e) n.a.

10. Doscientas bombillas iguales de 300


de resistencia c/u estn
. 2 2

conectadas en paralelo a una fuente


de 100 voltios y resistencia interna 4 3
120V 6
de 0.5. La potencia desprendida en
cada bombilla es:

a) 75 W
d) 50W
b) 37.5 W
e) 18.75W
c)125W . a) 36 b) 24 c) 72 d) 54 e) n.a.
11. Determinar la resistencia
15. Un motor elctrico absorbe 15A a
equivalente visto desde x e y, si 110V. Hallar el costo de
todas vales 1.5
x . funcionamiento mensual, si trabaja
durante 8 horas diarias y cada KW
a
Hr consumido vale 8.5 soles (Tomar
mes de 30 das)
a) S/. 3000
b b) S/. 3300
.
y c)
d)
S/. 3225
S/. 3366
a) 6 b) 5 c) 4 d) 3 e) n.a. e) S/. 2320

O
FSICA
16. En el circuito mostrado hallar I1
2 2 a) 1.45 hr
b) 2.54 hr
8v 24v
c) 3.73 hr
d) 4.17 hr
e) 5.29 hr
16v
1 2
20. Un alambre de cobre tiene una
resistencia de 9, si se le estira
I1 hasta que su longitud se
a) 1 A b) 2 A c) 3 A quintuplique. Hallar la corriente
d) 4 A e) 5 A. que circula por esta ltima
resistencia, si se le aplica a sus
17. Si un foco es conectado a una fuente extremos una diferencia de
elctrica de 220 voltios, la intensidad potencial de 675 voltios.
de la corriente a travs de l es
0.5A. Cul ser la intensidad de la a) 1 amp
corriente si se conectan 3 focos b) 2 amp
iguales al primero, en serie y a una
c) 3 amp
fuente de 1320 voltios?
d) 4 amp
a) 0.5 A e) N.A.
b) 0.75 A
c) 1A 21. Mediante una batera de 36
d) 1.25 A voltios se desea hacer funcionar
e) N.A. normalmente una lmpara
diseada para trabajar con 6v y
18. Dos lmparas que indican 60W 120V y 0.5A. Para ello se debe colocar
40W-120V respectivamente, estn en serie con la lmpara una
conectadas en serie a una lnea de 120V, resistencia de R ohmios y P
que potencia se disipa en las 2 lmparas,
vatios, donde valores correctos
en stas condiciones?
debern ser:
a) 320 vatios a) 12 , 3 W
b) 160 vatios b) 72 , 18 W
c) 144 vatios
c) 58 , 12 W
d) 24 vatios
e) 32 vatios
d) 60 , 15 W
e) 36 , 40 W
19. Al cabo de que tiempo despus
de cerrar el interruptor hervir el 22. Una pila se conecta a una
agua que inicialmente estaba a resistencia de 4 . Luego Se
80C, siendo su volumen de 3 reemplaza esta por otra de 9.
lts. Si ambas resistencias disipan la
10v
misma potencia Cul es la
resistencia interna de la pila?
7
a) 2
b) 4
c) 6
6 d) 8
Agua
e) 10

O
FSICA

Tiene como objetivo principal el estudio N


de las propiedades de los imanes y sus DIPOLO
interacciones mutuas. E MAGNTICO
S
Se denomina imn a toda sustancia que
es capaz de atraer al hierro o cuerpos
formados de hierro, a esta propiedad ACCIONES ENTRE LOS POLOS
de los imanes se le denomina MAGNTICOS
magnetismo.

En todo imn se distingue las siguientes FUERZA DE ATRACCIN


regiones:
F F
a) Polos. Es la regin en la cual se N S
concentran las propiedades
magnticas del imn en el caso
de un imn en forma de barra los FUERZA DE REPULSIN
polos se encuentra ubicados en
sus extremos.
F1 F1
N N
b) Zona Neutra. Es la regin que
presenta muy poco o ninguna
propiedad magntica.
CAMPO MAGNTICO
* Imn: Partes
HIERRO Se denomina as a la modificacin de las
POLO
propiedades del espacio que rodea a un
POLO ZONA
NEUTRA imn. El campo magntico trasmite las
acciones entre los polos magnticos y se
suele caracterizar por una cantidad
PROPIEDADES
vectorial denominada vector induccin
1) Orientacin de un Imn
magntica o vector campo magntico (B).

Todo campo magntico al actuar sobre


un imn ejerce sobre los polos de este
POLO POLO fuerzas de direcciones opuestas lo cual
NORTE SUR produce un torque el cual tiende a
orientar al imn en forma paralela al
NORTE SUR
GEOG GEOG campo magntico.

2) Inseparabilidad de los polos * Transmite las acciones entre los


polos magnticos
N S N S N S
* Induccin magntica ( B )

Unidad:

O
FSICA
S.I. Tesla (T)

EXPERIMENTO DE OERSTED
* PROPIEDAD
OERSTED descubri que al acercar un
F imn a un conductor recorrido por una
B corriente el imn experimentaba
B fuerzas que tendan a orientar al imn
F F
S N
en forma perpendicular al conductor.
OERSTED adems determin que el
F sentido del Imn depender del sentido
de la corriente.

El campo magntico al igual que el Adems, intensidad con la cual gira el


campo elctrico tambin se suele imn depende de la intensidad de
representar por lneas de fuerzas las corriente.
cuales presentan las siguientes
caractersticas:

1. Por cada punto del campo


magntico pasa una y solo
una lnea de fuerza. I
2. El vector induccin magntico I
es siempre tangente a la lnea
de fuerza en cada uno de sus
puntos.
3. Las lneas de fuerza se I I
orientan del polo norte al polo
sur por el exterior del imn y
del polo sur al norte por el Toda corriente produce un
interior del mismo. campo magntico.
4. La separacin entre las lneas B (D.P.) I
de fuerza es inversamente Todo campo magntico ejerce
proporcional al valor del fuerzas sobre cargas en
campo magntico de la regin movimiento.
considerada.
EFECTOS DE LOS CAMPOS
* Lneas de fuerza del Campo MAGNTICOS
Magntico
A) FUERZA SOBRE UNA CARGA MVIL
Todo campo magntico ejerce
B2
B1 sobre una carga en movimiento una
2 fuerza la cual presenta las siguientes
caractersticas.
F
B

B3

+
q
V
O
FSICA
(3) Movimiento de una carga en un
campo magntico uniforme
B Si V B M.C.U

x x x x x x


- x
V x
q w
V x
x x
R
x x x
F
F x x
q+ V

1) Depende de la direccin del x x x x x x

movimiento
2) Mdulo Donde: FMAG = FCP

F = |q| V B. Sen mV 2
|q| V B =
R
De donde:
|q| B.R. = mV q B R = mV
Si V B FMAX = q V B
Pero:
Si V// B FMIN = 0
V = w. R
3) F V y F B
qB
w=
4) Sentido, depende del signo de la m
carga.

Observacin: (4) Si V no es perpendicular a B, el


movimiento es helicoidal
(1) Unidad del Campo Magntico
Movimiento Helicoidal
F N
B = MAX T es la =
qV m
C.
s
F
N
T es la =
A.m V COS

B
(2) Como F B F V
V SEN
F no realiza trabajo
F no altera el valor de la V
velocidad, nicamente su
direccin.

O
FSICA
B) FUERZA SOBRE UNA CORRIENTE Presenta las siguientes caractersticas:
RECTILINEA
1) Dependen de la forma
Todo campo magntico ejerce una geomtrica del conductor que
fuerza sobre una corriente la cual es recorrido por la corriente.
depende de la forma del conductor que
es recorrido por la corriente as como el 2) El valor del campo magntico
campo magntico cumplindose en siempre es d.p. a la
particular que dicha fuerza es intensidad de corriente.
directamente proporcional a la
intensidad de la corriente. 3) El campo magntico tambin
depende del medio que rodea
Para el caso particular del campo al conductor que es recorrido
magntico uniforme y una corriente por la corriente.
rectilinia se cumple Q
El campo magntico se representa por
lneas de fuerzas cerradas razn por la
B
F cual se suele denominar lneas de
induccin las cuales rodean al
conductor que es recorrido por la
I corriente.
B
EL VECTOR
Induccin magntica siempre es
tangente a las lneas de induccin en
L cada uno de los puntos coincidiendo su
sentido con la orientacin de las lneas
de induccin.

La orientacin de las lneas de


induccin se obtiene mediante la
1) F = I L B Sen aplicacin de la regla de la mano
F = (BIL) Sen derecha o regla del saco corcho.
2) F conductor
. .
F B . .
.I .
3) Sentido: Basta conocer el sentido
convencional de la corriente.

* Adems
Si I B FMAX = BIL B
Si I//B FMIN = O

CAMPO MAGNTICO DE CORRIENTE B


Las leyes que permiten calcular los
. .
campos magnticos debido a corrientes .I .
son bastante complicadas pudiendo . .
reducir a partir de filas el campo
magntico producido por una corriente
en un punto.

O
FSICA
* Algunos campos magnticos N de espiras o vueltas
1) Corriente Rectilnea Infinita Si L>> dimensiones transversales del
solenoide y las espiras estn muy
juntas.

a) El campo magntico se
I I ...
r xxx
concentra en el interior
b) B centro = 2B extremo
c) El campo en el centro es
B uniforme y tiene un valor

Lneas de induccin: o NI
Circunferencia B=
L
0 I o = 4 x 10-7
B=
2 r N = N de espiras
L = Longitud del Solenoide
En el vaco
T.m. Siendo o la permeabilidad magntica
o = 4 x 10-2 del vaco
A
I = Ampere;
PROBLEMAS PROPUESTOS
R=n
B = Tesla (T)
1. Una partcula de carga 2e + se
mueve en un campo magntico
2) Corriente Circular
uniforme de 0,2T siguiendo una
trayectoria circular con un
perodo de 2 x 10-7 s. La masa
B de la partcula es
I a) 3,2 x 10-27 kg
I b) 6,4 x 10-27 kg
c) 1,6 x 10-27 kg
V r
r d) 4,8 x 10-27 kg
e) 2,4 x 10-27 kg
I
2. Un electrn con rapidez de 106
En el centro
m/s ingresa en una regin donde
existe campo magntico. Si la
o II
B= trayectoria es circular y tiene un
2R radio de 10 cm, la magnitud del
campo magntico ser
3) Solenoide (me = 9,11 x 10-31 kg)
L
a) 56,9 x 10-6 T
b) 56,9 x 10-8 T
c) 0,57 x 10-6 T
d) 5 x 10-6 T
e) 5 x 10-7 T

Y
Li: Longitud del solenoide
O
FSICA
3. Por un conductor rectilneo muy 6. En el centro de una espira de 12
largo circula una corriente de 2A. cm de dimetro hay un campo
A una distancia de 4 cm del magntico de 2T producida por la
conductor la magnitud del campo corriente elctrica que circula por
magntico B es ella. La corriente en la espira es

a) 2 x 10-5 T a) 6 x 105 A
b) 4 x 10-5 T 6
c) 10-5 T b) x 105 A

d) 5 x10-5 T c) 3 x 105 A
e) 3 x 10-5 T 3
d) x 105 A
4. Dos conductores separados una
distancia de 126 cm conducen e) 6 x 10 A
corriente de 10 A cada uno en
direcciones opuestas. La 7. Un electrn entra
magnitud del campo magntico perpendicularmente a la regin
en el punto P es de un campo magntico de 0,2T.
El tiempo que tarda en dar una
I vuelta es (me = 9,11 x 10-31kg)

a) 5,7 x 10-12 S
P b) 5,7 1012 S
8cm 8cm c) 57 x 10-12 S
d) 57 x 1012 S
e) 17,9 10-11 S
I
16 cm 8. En la figura, de que magnitud es
el campo magntico B para que
a) 5,2 x 10-5T b) 2T la carga q+ siga una trayectoria
c) 5 x 10-5T d) 0 T rectilnea horizontal? (Los
e) 3,2 x 10-3 T campos elctricos y magnticos
son uniformes)
5. Un alambre conductor rectilneo
por donde circula una corriente E = 18 N/C
de 5A es perpendicular a un
campo magntico de 3,4T. La
fuerza por unidad de longitud es
v = 10 m/s
a) 17N/m b) 1,7 N/m
c) 3,4 N/m d) 27 N/m q+ B
e) 34 N/m

a) 18 T b) 12 T c) 1,2 T
d) 1,8 T e) 2 T

O
FSICA
9. En la figura, la barra conductora 11. Dos alambres paralelos conducen
tiene largo L, masa m siendo corrientes en sentido opuesto,
su resistencia R. Los rieles son repelindose con una fuerza F1.
lisos y de resistencia despreciable Al duplicar las corrientes y la
y la fuente tiene una fuerza distancia de separacin, la fuerza
electromotriz V. Hallar el ngulo F2 ser:
de equilibrio de la barra.
a) 2F1 b) F1
c) 4F1 d) 8F1
e) 0,5F1
B
12. Un electrn describe un crculo
m de radio R1 con una velocidad
angular W1, dentro de un campo
magntico B1. Si el campo
HORIZONTAL
magntico se duplicase, entonces
RIEL son verdaderas.

I. Su velocidad angular se
V duplica
II. Su radio se duplica
III. Su radio no se altera.
VLB
a) Arc Sen a) I, II b) I, III c) I
mgR d) II e) III
mgR
b) Arc Cos 13. Se tienen tres vectores
VLB
perpendiculares entre si. Una
mgR carga positiva q se mueve con
c) Arc Tg
VLB velocidad v = ai , en un campo
Vmg uniforme B = b J
d) Arc Sen
LBR La fuerza magntica sobre la
Vmg carga es: (considerar a y b
e) Arc Cos
LBR positivos; los vectores i , J, K
son de mdulo unitario y
10. En el vaco una carga q gira adimensionales).
circularmente en una trayectoria
de radio R con una velocidad
lineal V. Hallar la induccin
magntica que genera la carga K
en el centro de sus trayectorias

u o qv u o qv
a) b)
R2 2R 2 J
u qv u qv
c) o 2 d) o 2
2R 4R
u qv i
e) 2o 2
R

O
FSICA
15. Se tiene un conductor
a) Cero infinitamente largo y rectilneo
llevando una corriente de 3A tal
b) Ab k como se muestra en la figura.
Cul ser el valor de B en el
c) qab k
punto P. si Cos = 3/4?
d) qab k

e) ab k

14. Cul ser el flujo magntico en


el casquete A hemisfrico
mostrado. Si el campo magntico
5 cm
B es constante ?
P
I
B

R
a) 2.6 x 10-8 T
b) 2 x 10-5 T
c) 2 x 10-7 T

d) 6 7 / 7 x 10-5T

A e) 1.6 x 10-5T

a) Faltan datos 16. Un electrn con velocidad 3.2 x


104 m/s entra en un campo
b) 2 RB
magntico uniforme
c) Cero perpendicular y describe un
crculo de radio 91mm. Cul es
BR 2 la magnitud del campo
d)
2 magntico? (qe = 1.6 x 10-19C;
me = 9.1 x 10-31 kg)
e) BR
a) 1.0 x 10-6 Wb/m
b) 2.0 x 10-6 Wb/m
c) 4.0 x 10-6 Wb/m
d) 8.5 x 10-6 Wb/m
e) 2.5 x 10-6 Wb/m

O
FSICA

Se denomina as aquel fenmeno el 3. A mayor velocidad relativa le


cual consiste en la generacin de una corresponde una corriente
corriente elctrica o una fuerza inducida de mayor intensidad.
electromotriz o voltaje a partir de un
campo magntico variable.

(Conductor) (Inductor
* Corriente
)
Inducida (I 1)
Campo Campo
Magntico Elctrico
Variable Variable
* Fem (Voltaje) V
Induccin (E )

EXPERIMENTO DE FARADAY
Este experimento se basa en hacer CONCLUSIN GENERAL
pasar una imn de propiedades Existe una corriente inducida y una
magnticas muy intensas a travs de fuerza electromotriz inducida si vara el
una bobina la cual se encuentra nmero de lneas de fuerza del
conectada a un galvanmetro, el cual inducido.
permite la medida de la corriente. Al
imn que genera el campo se denomina FLUJO MAGNTICO
inductor y a la bobina en la cual se Es una magnitud escalar la cual
establece la corriente el inducido. determina el nmero de lneas de
fuerza del campo magntico que
Despus de muchos experimentos atraviesan (Lneas de Induccin) de una
Faraday lleg a las siguientes superficie dada.
conclusiones.
El flujo magntico a travs de una
1. Se genera una corriente superficie se obtiene multiplicando la
inducida siempre y cuando componente del campo magntico
exista un movimiento relativo perpendicular a la superficie con el rea
entre el inductor e inducido. de dicha superficie.

2. El sentido de la corriente Observacin:


inducida depende del polo
magntico que se acerque o 1. La normal se traza a una sola
se aleje del inducido, de las caras de la superficie.
invirtindose el sentido de la
corriente al invertirse el 2. El flujo magntico puede ser
sentido del movimiento positivo o negativo
relativo. En particular el dependiendo del ngulo
acercar un polo norte es formado entre la normal y la
equivalente a alejar un polo direccin del campo
sur. magntico.

O
FSICA
3. Debido a que las lneas de
fuerza del campo magntico LEY DE FARADAY - HENRY
son lneas cerradas se tiene La fuerza electromotriz inducida en un
que el flujo magntico a circuito es proporcional a la rapidez con
travs de cualquier superficie la cual vara el flujo magntico a travs
cerrada es igual a cero. de dicho circuito.

NORMAL B
(N)

i =
t

S Unidad:
weber
Voltio:
segundo
= B . A. Cos
i
= BN . S

Donde: * Si el circuito est formado por N


espiras el efecto se hace N veces
BN = B.Cos
mayor.
Es la componente del campo
perpendicular a la superficie (en la
direccin de la normal)

Unidad:

WEBER (Wb) = T.m


MAXWELL (Mx) = Gs.cm
Z

1 Wb = 108 Mx

* CASOS PARTICULARES

N
B N

B X

B

i = -N
= BS =O = -B.S t

Donde es la variacin de flujo en 1 espira

O
FSICA
es decir la corriente o el voltaje se
LEY DE LENZ expresan con la ayuda de las funciones
Esta ley establece una relacin entre el seno o coseno.
campo magntico del inductor y el Para toda corriente alterna se tienen las
campo magntico que genera la siguientes caractersticas:
corriente inducida. Esta ley establece
que: 1. AMPLITUD
Toda fuerza electromotriz inducida en Es el valor mximo de la corriente o
un circuito cerrado genera una voltaje alterno.
corriente cuyo campo magntico se
opone a la causa que produce la f.e.m. 2. PERIODO
inducida. Es el tiempo al cabo del cual la
corriente o voltaje a dado una
* CASOS POSIBLES oscilacin completa y ha tomado todos
los valores positivos y negativos
1. Aumento del flujo permitidos.
Bo (Campo Inductor)
3. FRECUENCIA
Indica el nmero de veces que se repite
la oscilacin, tambin se le suele definir
como la inversa del perodo. En el caso
del Per la frecuencia es de 60Hz.
I
V = Vo Sen (wT)

B1
(Campo Inducido) Vo : Valor Pico
W : Frecuencia Angular
T : Perodo
2. Reduccin del flujo f: Frecuencia

B1 B1 Donde:
B0
2 1
T= =
W f

I * En particular

V
+

I (t)
CORRIENTE ALTERNA - R
Se denomina as a toda corriente o
voltaje que vara peridicamente en V( t )
valor y direccin. Una de las variaciones I(t) =
ms usuales es la variacin armnica, R

O
FSICA
1. Un ncleo de hierro o de un
I = Io Sen (wt) material magntico cuya funcin
es la de concentrar el campo
Donde: magntico en su interior.
Vo
Io = 2. Dos arroyamientos los cuales se
R
emplean uno para recibir el
voltaje que se desea modificar y
VALORES EFICACES dos para suministrar el voltaje
modificado. Al primer
Se denomina as a los valores de una arroyamiento se le denomina
primario y al segundo
corriente o voltaje continuo los cuales
producen el mismo efecto que una secundario.
corriente o voltaje alterno para un
mismo intervalo de tiempo.
Is
Ip
Q Q
V(t) V(ef)
+ +
Vp Vs
I (t) R R
2
3
- - I EF
1

1) Ncleo de Hierro
Depende la forma como vare V(t) y I(t) 2) Primario
Para una variacin Armnica. 3) Secundario

Vp = - Np
t
Vo Io Vp Np
VEF = IEF =
2 2 =
Vs Ns

Vs = - Ns
t
Luego se tiene:
Si las prdidas son despreciables

Pp Ps Vp Ip = Vs Is
IoVo
P = IEFVEF =
2 Luego:

Vp Np Is
= =
Vs Ns Ip
TRANSFORMADOR

Se denomina as a todo dispositivos * Entonces


diseado con la finalidad de modificar el
voltaje o la intensidad de corriente Np > Ns
alterna. Un transformador por lo Si Np > Ns Ip < Is
general est constituido por:
Vp > Vs
Si Np > Ns Ip < Is

O
FSICA
Wb Wb
a) 5 b) 10
PROBLEMAS PROPUESTOS s s
Wb Wb
1. Una bobina tiene 20 espiras c) 10-2 d) 10-4
s s
cuadradas de lado 10cm y se
Wb
encuentra perpendicularmente a e) 100
un campo magntico uniforme s
Wb 3. Un imn se mueve con rapidez
de magnitud B=2 . Si la constante hacia un electroimn,
m2
como muestra la figura. Indicar
bobina efecta un giro de 90
la verdad (V) o falsedad (F) de
respecto al campo, entonces la
las siguientes proposiciones
variacin del flujo magntico es
(N = vector normal al plano)
I) La corriente en R es de b
hacia a
II) El imn ser atrado por el
electroiman
III) El sentido de la corriente es
de a hacia b y el imn es
N
B repelido.
V
N S

a) = 0,4Wb b) = 0 R
c) = 40 Wb d) = 2Wb
e) = 0,2 Wb a b
a) VFV b) FFF c) VVV
2. Una barra metlica SP de 10 cm
d) FVF e) FFV
de longitud se mueve sobre un
riel metlico con una rapidez de
4. El flujo magntico a travs de
5 cm/s, como muestra la figura,
una espira en una bobina de 50
entonces la variacin del flujo
espiras, vara como muestra el
magntico por segundo es
grfico adjunto. Entonces la
Wb magnitud de la f.e.m. inducida en
B = 2 2
m la bobina entre 0,5 y 1 s es:

(Wb)
B 4
. . . . . s . . . . . . .
. . . . . . . . . . . .
. . . . . . . . . . . .
. . . . . . . . . . . . 2
. . . . . . . V
. . . . .
. . . . . . . . . . . .
. . . . . . . . . . . . t(s)
0,5 1
. . . . . P. . . . . . .
a) 200V b) 50V c) 2V
d) 0 e) 150V

O
FSICA
5. Un imn cae libremente y se II) La violacin de la ley de Lenz
acerca a una bobina, como conduce a la violacin de la
muestra la figura. Para el caso en ley de conservacin de la
que el imn an no atraviesa la energa.
bobina y observando la bobina III) En una central hidroelctrica,
desde la posicin del imn, la corriente elctrica que se
indicar la verdad (V) o falsedad produce bsicamente por la
(F) de las siguientes aplicacin de la ley de
proposiciones induccin de Faraday

I) Se induce una corriente en la


bobina en sentido antihorario a) FFF b) FVF c) VFV
II) Se induce una corriente en el d) VVF e) VVV
sentido horario
III) No se induce corriente en la 8. Un equipo de rayos x requiere un
bobina. voltaje de 30000V para
funcionar. Se dispone de un
voltaje de 200V y de un
N
transformador de 300 espiras en
el primario, entonces el nmero
de espiras en el secundario es
S
a) 45000 b) 10000 c) 2000
d) 30000 e) 50000

9. Un alambre recto de cobre de 2m


de longitud se mueve con
velocidad V en un plano
a) VFV b) VFF c) FFV perpendicular a un campo
d) FVF e) FFF magntico uniforme de 0,7
Wbm-2 los extremos se conectan
6. Una bobina de 100 espiras est a una resistencia de 3. Calcular
situada perpendicularmente a un la intensidad de la corriente para
campo magntico uniforme. Si el v = 3m/s
rea de las espiras son de 20
cm y el campo magntico vara
de 0 a 0,5 T en 0,1s, determinar Resistencia Despreciable
la magnitud de la f.em. inducida
x x x x x x
x x
x x x x x x
a) 1V b) 2V c) 0,5V x x
x x x x x x
d) 10V e) 20V x x
x x x x x x
x V x
7. Indicar la verdad (V) o falsedad
x x x x xR=
x 3x 2m
x x x x
(f) de las siguientes x x x x
x x x x
proposiciones x x x x
x x x x
I) Desde el punto de vista de los x x x x
x
principios fsicos, se puede
afirmar que un motor elctrico es
un dispositivo inverso a la de un a) 1,4 A b) 2,8 A c) 0,7 A
generador elctrico. d) 2,1 A e) 6,9 A

O
FSICA
10. Con respecto a los principios del
electromagnetismo a) I, II b) I, III
c) I, II, III d) II, III
I. Toda corriente elctrica genera e) III
un campo magntico.
II. Slo las corrientes variables 12. En la espira rectangular
producen un campo magntico. conductora, determinar el sentido
III. Todo campo magntico que de la corriente inducida. La
pasa a travs de una espira, espira desciende con una
genera en ella una corriente velocidad V y el cable
inducida. conductor infinito est en reposo.

Indicar las afirmaciones


verdaderas: V

a) I, II b) II, III
c) I d) I, III i1
e) II i2

11. El imn mostrado tiene


movimiento vertical de bajada y
subida el tramo h
Cuando baja el ampermetro A de I
cero central, indica una deflexin
hacia la derecha (horario) a) Como i1
b) Como i2
I. Cuando sube el imn la c) No circula corriente inducida
deflexin ser hacia la d) En cualquier sentido
izquierda (antihorario) e) N.A.
II. Si se invierte los polos del
imn, al bajarlo la aguja 13. En la figura, se tiene un anillo
deflexionar hacia la conductor de radio R y a lo
izquierda. largo de su eje un alambre
III. Si baja con velocidad conductor infinitamente largo por
constante, no hay deflexin. el cual fluye una corriente I cuyo
valor est aumentando.
Que afirmaciones son Determinar en que sentido fluye
verdaderas: la corriente inducida en el anillo?

S
2h I1
N R
I

h 1 0 1
2 2
A
I2

O
FSICA
a) Como i1 15. Un anillo circular de alambre de
b) Como i2 10cm de radio se coloca con su
c) No circula corriente inducida normal haciendo un ngulo de
d) En cualquier sentido 30 con la direccin de un campo
e) N.A. magntico uniforme de 5000 Gs.
El anillo se hace bambolear de
14. Un conductor de longitud L y manera que su normal gire
masa m puede deslizarse por un alrededor de la direccin del
par de guas metlicas verticales campo a razn de 120 RPM, el
conectadas a una resistencia R, ngulo entre la normal y la
como se indica en la figura. La direccin del campo no se altera
friccin, y la resistencia del por este proceso.
conductor y de las guas son Qu fuerza electromotriz
despreciables. Hay un campo aparece en el circuito?
magntico uniforme y horizontal
del mdulo B normal al plano de 2 3 2
la pgina y dirigido hacia afuera. a) v b) v
100 100
Cul es el valor de la velocidad
estacionaria final de cada bajo la 3 3
c) v d) v
accin de la gravedad? 1600 100

R e) N.A.

mgR mg mg
a) b) c)
2BL BLR B2 L2

BLR
d) e) N.A.
g

O
FSICA

Consideremos una simple antena origen a un campo elctrico variable,


formada por dos barras metlicas M y N etc. La asociacin de un campo
conectadas, como indica la figura, a un magntico y un campo elctrico, ambos
oscilador de alta frecuencia. Como el oscilantes, es la condicin necesaria
circuito est abierto, la corriente fluir para que se engendren ondas
slo un instante, hasta que las dos electromagnticas capaces de
barras quedan cargadas. Cada vez que propagarse por el espacio libre. El
se invierte la polaridad se produce un dipolo oscilante irradia energa en
breve flujo de corriente en direccin forma de ondas electromagnticas. En
opuesta. Este dispositivo es un dipolo todo punto, del espacio que recibe la
oscilante con cargas opuestas en sus radiacin hay un campo elctrico y otro
extremos que cambian continuamente magntico perpendiculares entre s y en
de signo con la misma frecuencia que el ngulo recto con la direccin de
oscilador al cual est conectado. propagacin.

M La radiacin es transversal. En el caso


+ del dipolo oscilante, el vector del campo
B elctrico radiado est siempre en el
mismo plano que el eje del dipolo y la
radiacin se dice que est polarizada en
el plano. Se verifica que en el vaco la
velocidad de propagacin est dada
por:
Oscilador

1
C= = 3 x 108 m/s
o o
B
-
N
La ecuacin de la onda puede ser
Las cargas elctricas aceleradas representada como:
producen alrededor de la barra un
campo magntico variable. Pero, como
sabemos, un campo magntico variable t x
E = Eo SEN 2 , o tambin
produce un campo elctrico capaz de T
inducir corrientes en los conductores.
Fue Maxwell quien, investigando estas
t x
relaciones entre campos magnticos y B = Bo SEN 2
magnticos, lleg a la conclusin de T
que un campo elctrico variable, incluso
en el espacio donde no hay corrientes
de conduccin, produce un campo
magntico oscilante.
De este modo, alrededor del dipolo, el
campo elctrico alterno produce un
campo magntico oscilante, el cual da
O
FSICA

y Se propagan con una velocidad que


depende del tipo de onda y de la
densidad del medio.
E (campo elctrico)

C
Se propagan necesariamente en un
medio material.
Velocidad de
propagacin
x
B (direccin de Se caracterizan por la variacin regular
(campo propagacin)
magntico)
de una sola magnitud, que puede ser
por ejemplo, la amplitud de la
z partculas vibrantes (ondas en una
cuerda) o la densidad del medio
En una onda electromagntica plana, (ondas sonoras).
las magnitudes del campo elctrico y
magntico estn relacionadas por: Transportan energa y cantidad de
E=CB movimiento.

De donde se concluye que los campos Se reflejan, se refractan y presentan


oscilan en fase, es decir cuando uno de fenmenos de difraccin o interferencia.
ellos es mximo el otro tambin se
hace mximo. ONDAS ELECTROMAGNTICAS
Son siempre transversales.
ENERGA DE UNA ONDA ELECTROMAGNETICA
En una onda electromagntica, al igual Se propagan siempre con la velocidad
que en una onda elstica, lo que se de la luz.
propaga es la energa del campo
electromagntico. Puede demostrarse Se propagan a travs del vaco.
que la energa que pasa, en la unidad
de tiempo, a travs de la unidad de Se caracterizan por la variacin regular
rea dispuesta perpendicularmente a la de dos magnitudes, el campo elctrico
direccin de propagacin, o sea, la y el campo magntico.
intensidad de la onda electromagntica,
es Transportan energa y cantidad de
E movimiento.
I = o EB = o E = o E/c
C
Se reflejan, se retractan y presentan
fenmenos de difraccin e interferencia.
Expresada en W/m
EL ESPECTRO DE LA RADIACIN
A continuacin se muestra para ELECTROMAGNTICA
comparacin las analogas y diferencias Las ondas de las diversas regiones del
que existen entre las ondas mecnicas espectro eletromagntico poseen
y las electromagnticas. propiedades semejantes, pero
diferentes en longitud de onda,
ANALOGAS Y DIFERENCIAS ENTRE
frecuencia y mtodo de produccin. En
LAS ONDAS MECNICAS Y LAS
ELECTROMAGNTICAS la figura se resumen las distintas
ONDA MECNICAS radiaciones del espectro y los intervalos
Pueden ser longitudinales (por ejemplo de frecuencia y longitud de onda que
ondas del sonido) y transversales les corresponden. La frecuencia
(ondas en una cuerda). superior 1021 Hz (longitud de onda 10-13

O
FSICA
m, corresponden a los rayos gamma La relacin entre longitudes de onda,
ms energticos, y la inferior 104 Hz y frecuencia del espectro, f, viene dada
(longitud de onda 104 m) a las ondas por la ecuacin = c/f, en donde c es
de la radio de muy baja frecuencia. la velocidad de la luz en el vaco. As,
Las ondas de la radio se engendran por por ejemplo, la longitud de onda de las
medio de circuitos elctricos oscilantes. ondas de radio transmitidas por una
Segn su frecuencia, se clasifican en estacin que opera a una frecuencia de
radiofrecuencia (RF) y microondas. 600 kHz (6 x 105 s-1) es
Entre las primeras estn las ondas
ordinarias de la radio, FM, televisin c 3 x 108 m / s
(VHF y UHF) radiotelefona, etc. Entre = = = 500m
f 6 x 105 s 1
las microondas estn las ondas de
radar.
ESPECTRO VISIBLE
Estas ondas constituyen lo que llaman
Para engendrar radiaciones con
luz, y se producen como resultado de
frecuencia superior a la regin de
ciertos ajustes internos en el
microondas no son tiles los mtodos
movimiento de los electrodos en
electrnicos, emplendose en su lugar
tomos y molculas. Segn su longitud
radiaciones atmicas. En el intervalo de
de onda o frecuencia, la luz produce en
frecuencia comprendido entre las
nuestra retina diferentes sensaciones,
microondas y la radiacin visible estn
que llamamos Colores.
los rayos infrarrojos o radiacin
trmica.
En la TABLA 2 se indica la relacin
entre el color, la longitud de onda y la
La luz visible es radiacin
frecuencia de la luz.
electromagntica en el intervalo de
frecuencia de 4 x 1014 Hz a 7.5 x 1014
Debido a la relacin entre el color y la
Hz, correspondiente a longitud de onda
longitud de onda o la frecuencia, una
comprendidas entre 750 y 400 nm
onda luminosa de longitud o frecuencia
(1nm = 10-9 m). A frecuencia todava
bien definida se llama MONOCROMTICA
mayores est la radiacin ultravioleta
(MONO: uno; CROMO: color)
(8 x 1014 a 3 x 1017 Hz).

Estas ondas son producidas TABLA 2


artificialmente por medio de descargas
elctricas en los tomos y molculas. El COLOR (m) f(HZ)
sol es una fuente poderosa de radiacin Violeta 3.90-4.55 x 10-7 7.70 6.59 x 1014
ultravioleta que interacciona con los Azul 4.55-4.92 x 10-7 6.59 6.10 x 1014
tomos de la atmsfera superior,
Verde 4.92-5.77 x 10-7 6.10 5.20 x 1014
produciendo un gran nmero de iones.
Por esta razn se denomina ionosfera. Amarillo 5.77-5.97 x 10-7 5.20 5.06 x 1014
Naranja 5.98-6.22 x 10-7 5.03 4.82 x 1014
Los rayos X se extienden en el intervalo
de frecuencia 3 x 1017 a 5 x 1019 Hz. Rojo 6.22-7.80 x 10-7 4.82 3.84 x 1014
Se producen en las capas ms internas
de los tomos. Por ltimo, los rayos
gamma ocupan la zona del espectro La luz en medios homogneos se
electromagntico de mayor frecuencia y propaga rectilneamente, por lo tanto
son de origen nuclear. podemos utilizar el concepto de rayo
luminoso, que nos indicar la direccin
de propagacin de la luz.
O
FSICA

REFLEXIN DE LA LUZ 2. REFLEXIN IRREGULAR O


DIFUSA
Es el cambio de direccin que Se presenta en superficies rugosas,
experimenta la luz al incidir sobre un verificndose que rayos de luz que
medio que no permite su propagacin. inciden paralelamente se reflejarn en
direcciones arbitrarias.

R
i
SE
FA

ESPEJO
R
TE

Son superficies pulimentadas, en las


IN

cuales existe reflexin regular.

RI = rayo incidente ESPEJO PLANO


RR = rayo reflejado Son superficies planas, pulimentadas
N = recta normal a la superficie donde en base a las leyes de la
i = ngulo de incidencia reflexin se obtienen imgenes que
R = ngulo de reflexin cumplen las siguientes caractersticas:
P = plano de incidencia
a) El tamao de la imagen (I) es
LEYES: siempre igual al tamao del
1. El rayo incidente, la normal y el objeto (O)
rayo reflejado son siempre b) La ubicacin del objeto y su
coplanares. imagen es siempre simtrica
2. i =R al espejo ( = -i)
c) La imagen es virtual y
TIPOS DE REFLEXIN derecha.

1. REFLEXIN REGULAR O
ESPECULAR Zona virtual(-) Zona real (+)
Este tipo de reflexin se presenta en
superficie pulimentadas, verificndose
que los rayos de luz que inciden
paralelamente se reflejarn tambin
paralelamente.

i o

ESPEJOS ESFRICOS
Son casquetes de esfera pequeos con
un abertura angular menor o igual a 5
tal que una de sus caras est

O
FSICA
pulimentada, y permite obtener d) Cuando el objeto se ubica en
imgenes reales o virtuales. le centro de curvatura (C), la
imagen es real, invertida y de
TIPOS DE ESPEJOS ESFRICOS igual tamao que el objeto y
ubicada en C.
1. ESPEJO CNCAVO e) Cuando el objeto se ubica
Son aquellos cuya cara pulimentada ms all de C, la imagen es
est en el interior. real, invertida y de menor
tamao que el objeto, ubicada
entre F y c.

2. ESPEJO CONVEXO
Son aquellos cuya cara pulimentada
est en el exterior en estos espejos las
caractersticas de la imagen son
l

nicas, siempre es virtual derecha y de


a
foc

menor tamao, que el objeto, ubicada


yo
Ra

entre F y V.

l
ca
fo
yo
Ra
C = Centro de Curvatura
F = foco
V = vrtice
xx = eje principal
= Distancia del objeto
i = distancia imgen
f = VF = Distancia focal
ECUACIN DE DESCARTES
R
f=
2 1 1 1
= +
f i
r = Radio de curvatura

CARACTERSTICAS
ECUACIN DEL AUMENTO (A):
a) Cuando el objeto se ubica
I i
entre V y F, la imagen es A= =
virtual, derecha y de mayor O
tamao que el objeto.
b) Cuando el objeto se ubica en CUADRO DE SIGNOS
el foco (F) no se forma f i A o II
imagen ya que los rayos + Espejo Siempre Imagen Imagen
reflejados salen paralelos. Cncavo Real derecha
c) Cuando el objeto se ubica - Espejo Nunca Imagen Imagen
entre F y C, la imagen es real, Convexo Virtual Invertida
invertida y de mayor tamao
que el objeto ubicada ms all
de C.
O
FSICA
NDICE DE REFRACCIN (n)

Es una cantidad adimensional que mide


la densidad ptica del medio i
transparente, se define como la
relacin de la velocidad de la luz en el
vaco (c) a la velocidad de la luz en
dicho medio (v).

c of o
n= = = RI = rayo incidente
v f Rr = rayo refractado
Ya que al pasar de un medio a otro la N = recta normal a la superficie
frecuencia de la luz no se altera por que i = ngulo de incidencia
el nmero de longitudes de onda que r = ngulo de refraccin
llegan a la interfase en la unidad de P = plano de incidencia
tiempo, es igual al nmero de
longitudes de onda que se transmite al LEYES
otro medio. 1. El rayo incidente, la normal y
el rayo refractado son siempre
o = longitud de onda de la luz en el coplanares.
vaco 2. n1 SEN i = n2 SEN r -----------
= longitud de onda en el medio. - LEY DE SNELL

TABLA 3 En base a la ley de SNELL se deduce


que cuando la luz pasa de un medio
SUSTANCIA INDICE DE menos denso a otro ms denso el rayo
REFRACCIN refractado se acerca a la normal, es
AGUA (25C) 1.33 = 4/3 decir n1 < n2 i > r.
ALCOHOL (20C) 1.36
VIDRIO (CROWN) 1.52 Adems si la luz pasa del medio ms
HIELO 1.31 denso al menos denso el rayo
VIDRIO FLINT 1.65 refractado se aleja a la normal, decir
AIRE 1.00029 n1 > n2 i < r.
CUARZO 1.57-1.45
ANGULO LIMITE
SODIO 4.22
Es el ngulo de incidencia que permite
DIAMANTE 2.417
un ngulo de refraccin de 90 esto
solamente sucede cuando el haz de luz
REFRACCIN DE LA LUZ
pasa del medio ms denso al menos
Es el cambio de direccin que
experimenta la luz, al pasar de un denso.
medio transparente a otro.
REFLEXIN TOTAL INTERNA
Este fenmeno se produce cuando el
ngulo de incidencia es mayor que el
ngulo lmite; en este caso la luz no
puede pasar al otro medio reflejndose
totalmente.

O
FSICA

L
i > L

Clculo del ngulo lmite (L)

n1 SEN i = n2 SEN r

Menisco
Plano convexo
n1 SEN L = n2 SEN 90 convergente

n2 n 2. LENTES DIVERGENTES O
SEN L = L = ARC SEN 2 NEGATIVAS
n1 n1 Toda lente que sea ms gruesa por sus
bordes que por el centro har que un
haz de rayos paralelos al eje salgan
LENTES divergentes de la lente. El punto F del
cual divergen los rayos al salir de la
Son sustancias transparentes que lente, es el foco principal, como la luz
presentan dos caras donde una por lo no pasa en realidad por ese foco, se
menos debe ser esfrica y permiten dice que es un foco virtual.
obtener imgenes aprovechando el
fenmeno de la refraccin.

TIPOS DE LENTES

1. LENTES CONVERGENTES O
POSITIVAS
Cuando un grupo de rayos luminoso
incide sobre estas lentes paralelamente
a su eje, cada rayo se desva hacia la
parte ms gruesa de la lente; al salir de
esta, convergen hacia un punto F del
eje, llamado foco principal. A la
distancia del centro de la lente al foco
principal se da el nombre de distancia
focal de la lente (f), una lente delgada
tiene dos focos principales uno a cada
lado de la lente y equidistantes de ella.

O
FSICA
ELEMENTOS DE UNA LENTE Los radios se colocan con su signo de
acuerdo a las zonas.

POTENCIA DE UNA LENTE


Esta magnitud es una medida del poder de
convergencia o divergencia de una lente,
por ejemplo para una lente convergente, si
su distancia focal (f) es pequea los rayos
luminosos rpidamente se acercan a
juntarse en el foco por lo tanto la potencia
de la lente es grande, de donde:

l
C1 y C2 son los centros de curvatura de P=
las caras. f
R1 y R2 son los radios de curvatura.
F1 y F2 son los focos principales. f = en metros
O es el centro ptico de la lente P = en dioptras
Xx es el eje principal de la lente.
es la distancia objeto DISTANCIA FOCAL EQUIVALENTE
i es la distancia imagen DE UN CONJUNTO DE LENTES
DELGADAS
F2O = F1O = f Es la distancia focal de la Por ejemplo para el caso de tres lentes
lente. de distancias focales: f1, f2 y f3 la
distancia focal equivalente fE ser:
ECUACIN DE LOS FOCOS
CONJUGADOS
1 1 1 1
1 1 1 = + +
= + f E f1 f 2 f3
f i

ECUACIN DEL AUMENTO


CUADRO DE SIGNOS
F i A o II
II i
A= = + LENTE OBJETO IMAGEN IMAGEN
o CONVERGENTE REAL REAL DERECHA
- LENTE OBJETO IMAGEN IMAGEN
DIVERGENTE VIRTUAL VIRTUAL INVERTIDA
ECUACIN DEL FABRICANTE DE
LENTES: NOTAS
1. Las imgenes virtuales se forman en
l nL 1 1 la interseccin de las prolongaciones
= 1
f n M R1 R 2
de los rayos luminosos, estas
imgenes se pueden ver a simple
vista.
2. Las imgenes reales se forman en la
Donde: interseccin de los rayos reflejados o
nL = Indice de refraccin de la lente. refractados segn sea el caso en un
nM = Indice de refraccin del medio que espejo o lente respectivamente, estas
imgenes no se ven a simple vista, se
rodea a la lente.
necesita una pantalla donde
R1 = Radio de la cara de la lente mas proyectarlas.
cercana al objeto.

Вам также может понравиться